nbde II

¡Supera tus tareas y exámenes ahora con Quizwiz!

:Multiple macule and café-au-lait ?? 1-neurofibromatosis 2-Albright-syndrome 3-both

3-both

claas V glassionomer prep should 1-bevel occlusal wall 2-bevel gingival wall 3-no bevel at all 4-90 degree angel

3-no bevel at all

Value can be remembered by thinking that patients VALUE white teeth

Lighter colors - increased value Darker colors - decreased value

Child show equal numbers of primary teeth and permenant teeth in his mouth (12 primary & 12 permenant) at age of: 9-10) - 7.5 - 8.5 - 9.5 - 10.5

- 8.5***

doctor insist the patient for treatment plan and explain to him the advantages of treatment without telling him about the disadvantages: - Veracity - Autonomy - Hetronomy

- Hetronomy***

. Good sugar replace in high caries - xylitol, sorbitol

xylitol

treatment of ranula ?

Excision

Snaked eye teeth?

Mandibular 1st premolar

The drug-of-choice for the treatment of adrenergically-induced arrhythmias is quinidine. lidocaine. phenytoin. propranolol

"D" because Lidocaine binds to Na channels inhibiting recovery after repolarization while propranolol bind to beta adrenoceptors so it is tx of choice for adrenergically induced arrhythmia.

Which of the following is/are locally delivered antimicrobial(s) agent(s) used to treat infected periodontal pockets? 1. Metronidazole. 2. Chlorhexidine. 3. Doxycycline. 4. Clindamycin. All of the above. (4) only. (2) and (4) (1) and (3) (1) (2) (3)

(1) (2) (3)

The characteristic of resin bonding A) more in dentine than enamel B) more filler C) depend on etching D) more in enamel

) depend on etching

Which of the following are primary reasons for splinting teeth with a fixed prosthesis?: a) To stabilize loose teeth in a favorable occlusal relationship. b) To distribute occlusal forces so periodontically weakened teeth do not loosen c) To prevent a natural unopposed tooth from migrating. d) To prevent maxillary central incisors from separating after closure of diastema. * a,b,c * a,c,d * b,c,d * a,d

* a,d

modiolus (hub of wheel) is a point at the corner of mouth formed byintersection of 8 muscles-

- depressor angulii oris -levator anguli oris -risorius - orbicularis oris -buccinator -zygomatic major -quadratus labii superioris -quadratus ladii inferioris

ideal occlusion in the adult population of the unites states is found a. often 80-90% b. 50-60% of the time c. 20-30% of the time d. very rarely(0-5%)-

. 20-30% of the time

Head and neck radiation therapy causes 1. Hypersensitivity 2. Trismus

. Trismus

implant vertical bone loss amount the first year? implant vertical bone loss amount after the first year?

0.02mm

A recently-introduced local anesthetic agent is claimed by the manufacturer to be several times as potent as procaine. The product is available in 0.05% buffered aqueous solution in 1.8 ml. cartridge. The maximum amount recommended for dental anesthesia over a 4-hour period is 30 mg. This amount is contained in approximately how many cartridges a. 1-9 b. 10-18 c. 19-27 d. 28-36 e. Greater than 36

0.05*10*1.8= 0.9 30mg/0.9 = 33.33 mg Answer is D

What happens to the length in the mandibular arch? 6-18 years old

1 mm decrease due to incisor uprighting

patient has PFM crowns and amalgam restoration, apply: 1.23 % acidulated phosohate flouride stannous flouride 5% sodium flouride 1% neutral sodium fluoride

1%neutral sodium flouride

First indication of cavernous sinus thrombosis 1) blurred vision 2) peri orbital edema 3) headache

1) headache

Tensile strength is good in 1-Polycarboxylate 2-Zinc Phosphate

1-Polycarboxylate

Cancer of the other parts of the body metastases most commonly to? 1-Tongue 2-Lip 3-Floor of the mouth 4-Gingiva

1-Tongue

The tooth preparation for a porcelain veneer must have a 1. rough surface. 2. space for the veneer material. 3. definite finish line. 4. margin at least 1mm supragingivally. A. (1) (2) (3) B. (1) and (3) C. (2) and (4) D. (4) only E. All of the above. . A patient with pain, fever and unilateral parotid swelling following a general anesthetic most likely has A. Mumps. B. sialolithiasis. C. acute bacterial sialadenitis. D. Sjögren's syndrome. E. sarcoidosis The most objective measurement of successful scaling and root planing is 1. reduction of pocket depth. 2. root smoothness. 3. absence of plaque. 4. absence of bleeding upon probing. 5. increased sulcular fluid flow. A. 1,5 B. 2,4 C. 1,4,5 D. 1,4

1-a 2-c 3-c

Q1 histologically the loss of rete peg is a sign of 1 pemphigus 2 linchen planus 3 syphills 4 pemphigoid Q2 best reson for RPD over fixed partial denture 1 hygiene 2 coopertion 3 esthetic Q3 where is gold directed on MO onlay spruce 1 gingival floor 2 pulpal florr 3 occlusal florr 4 faces pulpal axial line angle Q4 GREATEST appositional groth occure 1 chin 2 ant boarder of ramus 3 post boarder of ramus Q5 when do you do serial ext 1 space deticiency in mand ant region 2 space deficiency in mand post region 3 space deficiency in maxillary ant region 4 space deficiency in max post region Q6 periodontal exam of pt referred for endo treatment 1 there is inward flow of fluid 2 there is outward flow of fluid there is no fluid Q7 when there is deep pocket with juvenile perio leaset effective treatment 1 root planning 2 occlusal adjustment 3 perio surgery 4 antiobitic treatment Q8 how far should implants placed from one anthor 1 2mm 2 4mm 3 5mm 4 7mm Q9 what is impression matrial with best dimension stablity 24 afer taking impression 1 PVS 2 reversible colloid 3 irreversible colloid Q10 complication of uprighting molar 1 move distally and extrudes 2 class 2 molar relationship develop 3 class 3 molar reletionship develop Q11 which of the following effect common to meperidine,diazepam,pentobarbital 1 amnesia and skeletal muscle relaxation 2 anticonvulsant and hypnotic 3 analgesia and relief of anxiety Q12 in office bleaching changes the shade through all except 1 dehydration 2 etching tooth 3 oxidation of colorant 4 surface demineralization Q13 lateral bennet shift is most likely to effected by centric occlusion 1 mesial distal step incline 2 facial lingual steep incline

1-ans2 2- ans 3 3-I think 3 -- spru should be attached to thickest portion 4-ans3 5-ans-1 6- 7-Answer 2. 8-2 9-ans-1 10-ans-1 11-ans-2 12-ans-4 13-ans2

After injection of LA pt starts to wheezing due to 1-asthma 2-COPD

1-asthma

during hinge movement which anatomical structure are related for pure rotation 1-condyle and disc 2-glenoid fossa 3-condyle 4-disc fossa and condyle

1-condyle and disc

the location of finish line in abutment tooth is determined by?- c- Length of clinical crown d- Type of planned restoration How to verify the success of ttt of apical periodontitis?-20 a- decrease in the size of radiolucency b- Fistula closure c- The tooth becomes asymptomatic d- Negative pulp tes

1-d 2-a

pt turn pale after LA injection what causes this 1-epinephrine 2-cradiovascular effect of epi 3-CNV of epi 4-anesthesia

1-epinephrine

antibodies found in the epithelium , acanthosis, bullae 1-pemphigus 2-pemphigoid

1-pemphigus

Removing cusp affects retention form T/F Increasing intercuspal space affects resistance form T/F Marginal ridges help with resistance form T/F Loss of marginal ridge affects both resistance and retention T/F

1-removing cusps, which is a resistance factor,will in turn reducing surface area for retention of crown..so it could be true.. Not sure.. 2- T (by altering arc of displacement) 3,4- T (a dome shaped tooth preparation, By not creating marginal ridges, will offer less resistance to forces of displacement; also less retentive cuz reduced surface area of tooth for prosthesis)

x ray for subcondylar fracture x ray for disc of tmj

1-subcondylarfracture- ct, pano disk -hard tissue evaluation-pano softtissue evaluation-arthrography

The outline form of upper maxillary molar access opening is Triangular, The base of this triangle is directed toward: 1. Buccal. 2. Palatal. 3. Mesial. 4. Distal.

1. Buccal.

Safe position in emergency? A.Trendelenburg B.Upright C.Semi upright D.Supine

1. Emergency- Conscious patient-- Supine position Unconscious patient-- Recovery position (lateral recumbent or 3 quarters of d body in prone position) 2. Vasovagal syncope/hypotension/shock patients-- Trendelenburg position 3. Pregnant pt.-- Left lateral position

Mucoceles the best treatment is: 1. Excision. ( Surgical excision )** 2. Leave it. 3. Marspuilization. 4. Cauterization.

1. Excision. ( Surgical excision )

Transminase is increased in 1. Hepatitis 2. SABE 3. MI 4. Jaundice

1. Hepatitis

Sorting of patient of trauma of head injury on basis of need of care and reliability of resources ? 1. Triage 2. Emergency care 3. Definitive care 4. Hospital care

1. Triage

Classical pattern of extraction in Class II camouflage is: a) Extraction of upper 1st premolars and lower canines. b) Extraction of upper 1st premolars and lower 2nd premolars. c) Extraction of all 2nd premolars. d) Extraction of upper 2nd premolars and lower 1st premolars. e) Extraction of all 1st molars

1. U4s, L5s —> severe cl II 2. 4s —> bimax, crowding in ant, not severe cl II 3. U4s —> molar cl II, canine cl I, reduce OJ

Amount of periapical bone deposition per month, following a successful endo treatment?

1.2 sq.mm per month

gingival index/perio index flaws?

1.Since no caliberated probe or essentially radiographs are used when performing the PI examination, the results tend to underestimate the true level of periodontal disease, especially early bone loss in a population. 2. The number of periodontal pockets without obvious supragingival calculus is also underestimated in the periodontal index. 3.It doesn't indicate the degree of periodontal tissue destruction. 4.Modern understanding has shown the PI to be invalid because it does not include evaluation of Clinical Attachment Loss (CAL), grades all pockets of 3 mm or more equally and scores gingivitis and Periodontitis on the same weighted scale.

Pt with yellowish discoloration after RCT due to: 1.leaving GP at pulp chamber 2.incompelet depridment of canal 3.Hemorrhage from pulp 4.1+3

1.leaving GP at pulp chamber

The purpose of relining a distal saddle of a removable partial denture is to improve 1. tissue adaptation. 2. occlusion. 3. function. 4. fit of the framework.

123

What is the proper order for treatment planning? 1- emergency care 2- reevaluation 3- disease control 4- maintenance care 5- definitive treatment

13254

191. difference between acetaminophen and aspirin a. asthma patient 192. which sedative medication cause dissociation from the world a. ketamine 198. dental phobia is hard to eliminate because it is a. self reinforcing b. become habit c. patient can not seen 200. best scale for gingival index a. ratio b. nominal c. interval 201. most of dental fear are from a. patients experiences b. family and friends 203 Studer-Weber syndrome a. mandibular retro b. midface ecto c. maxillary prog 204. patients #8 and #9 PFM is little darker than adjacent teeth but is clinically acceptable, what is treatment you would propose except one a. redo PFM b. indirect resin bond to c. porcelain bond to 208. #29,30,31 all need DO amalgam, which one do first 211. for beveling gingival margin, what not to use a. hatches b. gingival margin trimmer c. thin taper bur 214. preload of implant is comparable t what force a. torque b. compressive 215. top of the implant is what mm from adjacent CEJ a. 2-3mm b. 4-5mm c. 7-8mm d. 5-6mm 216. keratosis happen where in the mouth a. palate b. buccal mucosa c. floor of mouth d. upper lip 218. what is vestibuloplasty a. adding the bone to the chin b. adding the soft tissue inter occlusal for better denture fitting 224. systemic fluoride are least effective on a. proximal surface b. smooth surface c. root surface d. pits and fissure 225 When finishing the occlusal portion of a posterior composite restoration, the dentist should carefully a. eliminate contacts in the fossa b. avoid altering the centric contact on enamel c. develop centric contacts on cavosurface margins 230 A patient who has Parkinsons disease is being treated with levodopa. Which of the following characterizes this drugs central mechanism of action a. it replenishes a deficiency of dopamine b. it increases concentrations of norepinephrine c. it stimulates specific L-dopa receptors d. it acts through a direct serotonergic action 233 What is Bennett angle a. it is the angle that is formed by the non-working condyle and the sagittal plane during lateral movement b. it is the angle that is formed by the condyle and the horizontal plane during protrusive movements. c. It is an difference in condylar inclination between protrusive and lateral movements d. It is the difference between in the condylar and incisal inclinations. 235 an adult patient presents with multiple, soft nodules and with macular pigmentation of the skkin. Which of the following BEST represents this condition a. lipomatosis b. neurofibromatosis c. metastatic malignant melanoma d. polyostotic fibrous dysplasia e. bifid rib-basal cell carcinoma syndrome 238 a 5 years old child has a posterior unilateral crossbite that is accompanied by a functional shift of the mandible. When should this crossbite be corrected a. immediately without waiting for the eruption of permanents first molar b. when all the primary teeth have exfoliated c. after the permanent first molars have fully erupted d. when the child is approximately 9 years old 243 which of the following agents is used for HIV infection a. amantadine b. acyclovir c. zidovudine d. ribavirin e. isoniazid 245 which of the following is the MOST common postoperative problem associated with mandibular sagittal-split osteotomies a. infection b. TMJ pain c. Periodontal defects d. Devitalization of teeth e. Neurosensory disturbances 252 Initial instrumentation for the biomechanical preparation of a vital tooth should begin at the canal orifice and should end at the a. radiographic apex b. cementodentinal junction c. cementoenamel junction d. cement pulpal junction 255 an x-radiation dose of 4Gy administered locally to a patients arm is MOST likely to cause a. erythema b. bone marrow death c. carcinoma of the skin d. acute radiation syndrome 256 biologic changes in the aging patient affect both pharmacokinetics and pharmacodynamics of drugs, which in turn, are manifested as each of the following EXCEPT one, which one is this EXCEPTION a. decreased renal excretion b. increased plasma protein binding c. increased half lives of some drugs d. decreased volume of distribution and sequestration of drug in body fat 262 A patient has pain over the left pre-auricular area, this patient can open approximately 45 mm and has a pop-and-click in the joint area. The MOST likely diagnosis is a. myofacials pain dysfunction syndrome b. internal derangement with reduction c. auriculotemporal syndrome d. coronoid hyperplasia 264 in which stage of tooth development does tetracycline discoloration occur a. apposition b. proliferation c. mineralization d. histodifferentiation 267 for a population, the research divides the number of disease cases by the number of people. By so doing, this investigator will have calculated which of the following rates a. incidence b. odds ratio c. prevalence d. specificity 269 an apically displaced flap is generally impossible in which of the following areas a. mandibular facial b. mandibular lingual c. maxillary facial d. maxillary palatal 271 a mandibular first molar crown is severely decayed due to caries. Each of the following procedures can be used to remove this tooth EXCEPT one. Which one is this EXCEPTION a. using a cowhorn forceps b. removing the thin lingual cortical plate of bone c. elevating the root sections with an east-west elevator d. reflecting a modest buccal flap and removing the buccal bone 281 what is the method of HMO payment? Capitation 293 which of the following is used in restored a small portion of a mandible a. silastic b. tantalum c. homogenous bone d. autogenous bone 295 smooth muscle relaxation is caused by which of the following drugs a. prazosin b. atropine c. theophylline d. amphetamine 309 how long after eating is the PH in the mouth significantly lower a. 10-30 min b. 1-2 hours c. 2-4 min d. 3-6 hours e. 10-15 min 310 all f the following cause vasoconstriction except a. epinephrine b. norepinephrine c. phenylephrine d. levonor defrin 314 opioid agonists act by a. stimulating GABAergic neuron b. increase pain threshold c. acting as Mu receptor agonists 320 CHIP a. use in perio diagnosis only b. use in children 321 postural hypotension is a common complaint of patients who take antihypertensive agents because many of these agents interfere with the a. sympathetic control of vascular reflexes b. release of acetylcholine in the ganglia c. epinephrine release from the adrenal medulla d. parasympathetic control of vascular resistance e. neuromuscular transmission in skeletal muscles

191-true 192 -True 198-a 201 -B 215- A 243- C 245- E 271- B or d 293- D 295 -C 310 -D 314- C or b 321- A 208-31 211-b 224-d 238-a 245-d 264-c 309-e 200- 204-a 216-a or b 218-b 230-a 233-a 235-b 252-c 267-c

best way to determine platelet fxn a. platelet count b. bleeding time c. PTT d. INR QUESTION: What aspirin affects? Extrinsic, intrinsic, bleeding time, common pathway? QUESTION: What determines the bleeding time? Intrinsic, extrinsic, platelet adherence, common pathway

1q- bleeding time 2q- common pathway 3q-platelate adherence

For PM,the distance from the margin of the mesial and distal wall to the proximal surface must not be less than____________mm. For Molars,the distance from the margin of the mesial and distal wall to the proximal surface must not be less than____________mm.

1q-1.6mm 2q- 2mm

During casting procedure the wax pattern should be at what distance from the end of casting ring.... A 3 mm B 6 C 10 D 2 the thinest portion of wax pattern should be placed in th ? deepest portion of wax pattern against the ring of support opposite to the direction of rotation of casting

1q-6mm 2q-A

Which of the following conditions is most likely to result in new Periosteal bone formation A. Fibrous dysplasia B. pagets disease C. Chronic osteomyelitis D. Hyperparathyroidism radiographically there is a widening of the periodontal ligament space around all the teeth. what is your initial diagnosis? A. earliest sign of osteosarcoma B. scleroderma C. hyperparathyroidism D. fibrous dysplasia 12. Traumatic bone cyst - associated with? a Paget's, b fibrous Dysplasia, C fibro-osseous Dysplasia Picture 14 y/o right man expansion and RCT on #30, key is what causes bone expansion and tooth extruded? A-Osteosarcoma, B-fibrous dysplasia

1q-B 2q-A 3q-b(Simple bone cyst accompanied by florid cemento-osseous dysplasia (FCOD). 4q-a?

Plain retraction cord how long time to retract gingival from tooth surface 2min 4 min Retraction cord how long to keep it a sulcus 2 min 4 min After removal retraction cord how long does it takr to stay in it is place until it returns to its place

1q-b 2q- 5minutes 3q-1minute

the degree of leukocytes infiltration of the junctional epithelium is dependent of the volume of inflamed connective tissue T/f Thus, this process may occur in gingiva with significant signs of clinical inflammation t/f

1q-false 2q-true The degree of leukocyte infiltration of the junctional epithelium is independent of the volume of inflamed connective tissue; thus, this process may occur in gingiva with only slight signs of clinical inflammation.61

Primary endodontic infection have _____ organism Unsuccessful endo treatment have ______ organisms

1q-gm negative 2q- E. feacalis

Most common c/r fracture Most common horizontal Most common vertical least fracture

1q-max central 2q-Max ant 3q- mand 2 molar then first and then max premolars 4q-mandi premolar, canines

Cleft Lip surgery closure is done at 2-3 months T/f cleft palate surgery closure is done at 8-9 yrs before eruption of canine t/f Soft palate closure is at 18-24 months t/f Hard palate closure is at 4-5 yrs t/f

1q-true 2q-false(performed between 12 and 16 years of age, after eruption of the canine.) 3q-soft palate closure at 9-12 months of age 4q-true

1q-How do you differentiate between an endo/perio lesion? 2q-Best prognosis Perio endo Endo perio

1q-vitality test(thermal) 2q- endo perio

Bur used in endodontic procedure and their number?

2 and 4 diamnond bur Transmetal bur Carbide 2 and 4 bur Endo z bur Gates gidden drill

Porcelain laminate veener fixed by resin show black margin in 2 days cause 2 weeks cause 2 months cause Options are Amine Resin wear off Porcelain break Microlekage

2 day -porcelain break 2week-amine 2month-microlekage

which of the following conditions should an emergency occlusal separator should be constructed for 1 degenerative arthritis 2 myositis 3 hemarthrosis 4 capsular fibrosis 5 unilateral condylar hyperplasia

2 or 5

how to distinguish between external versus internal resorption

2 radiographs are taken one perpendicular and one mesial. in case of external resorption ,the lesion will shift from the pulp canal.

Incision of undisplaced flap start : 1) just apical to the base of the pocket 2) just coronal to the base of the pocket

2) just coronal to the base of the pocket

rays with high intensity : 1.xrays 2.gamma rays 3.uv rays 4.beta rays

2-From highest to lowest is Gamma,X-rays,ultraviolet,visible,infrared,least is radio

Q2:The condylar guidance is increased from 20 to 45 degrees,what do you do . 1- Decrease the compensatory 2-Increase the compensatory curve

2-Increase the compensatory curve

when pt bite and detach it's painful 1-vertical fracture 2-cracked tooth

2-cracked tooth

:What is the cause of wheezing 1-inspiration difficulty 2-expiration difficulty

2-expiration difficulty

what is the reason inflamed tissue are hard to anesthetize 1-anesthesia is unstable 2-ionized 3-lipid soluble

2-ionized

n gingivoectomy External bevel is put to tooth apical to what !! 1. Crest of bone 2. JE 3. CT 4. JE

2-performed APICAL to JE that severs CT attachment down to osseous crest.

Green stick # are 1. Open # 2. Closed # 3. Comminuted Open 4. Comminuted closed

2. Closed #

Caries progression in children more rapid than adult due to: 1. Difference in PH. 2. Generalized dentine sclerosis by age* 3. Increasing in organic content of tubular dentine by age

2. Generalized dentine sclerosis by age

A teenager (11 yo) presents with an intruded maxillary lateral incisor (#7). What is the next step of treatment? 1.Extract and Implant 2. Splint for 2 weeks and follow up 3.Splint for 2-3 weeks and follow up

2. Splint for 2 weeks and follow up

The most common neoplasm of the parotid gland is: 1. Warthin's tumor. 2. benign mixed tumor. 3. monomorphic adenoma. 4. adenoid cystic carcinoma. 5. mucoepidermoid carcinoma.

2. benign mixed tumor.

marginal leakage of amalgam restoration is 1. related to new phases formed 2. decreased as the restoration ages 3.severe throughout the life of the restoration 4. less after initial placement than at 6 months

2. decreased as the restoration ages

In determining the need for extraction of a tooth due to periodontal disease: 1. the pocket depth is more important than the attachment loss. 2. the attachment loss is more important than the attachment remaining. 3. the mobility is more important than the attachment loss. 4. the attachment remaining is more important than the attachment lost. 5. the pocket depth is more important than the amount of recession.

2. the attachment loss is more important than the attachment remaining.

In young children with avulsed and replanted permanent teeth that have open apices, the blood supply is usually regained within

20 days

What is the dosage of cortisone that should be given every day for two weeks in 2 years in order to suspect adrenal crisis and you might need hydrocortisone supplements before surgery? a. 20 b. 200 c. 10 d. 1

20 is the answer. The general recommendation for patients on steroids is if the patient took 20 mg of cortisone for 2 weeks in the past 2 years, we have to give cortisone supplement to avoid adrenal crisis under stressful procedure. Some Guidelines: • People on small doses (5 mg prednisone/day) will have suppression when they have been on the regimen for a month. • People taking the equivalence of 100 mg cortisol/day (20-30 mg prednisone/day) will have ab- normal cortical function in a week. • Short-term therapy (1-3 days) of even high-dose steroids will not alter adrenal cortical func- tion. • A person who has been on suppressive doses of steroids will take as long as a year to regain full adrenal cortical function.

28. To drain submandibular abscess: a. lntraorally through the mylohyoid muscles b. Extraorally at the lower border of the mandible. c. Extraorally under the chin. d. Extraorally at the most purulent site. 66. Patient suffering from a submandibular gland abscess. Dentist made a stab incision and is fixing a rubber drain to evacuate the pus. The drain is sutured to: a. Intra-oral b. From angle of the mandible c. Between myloid muscle d. All the above 64. For continous evacuation of pus in a mandibular space infection, the incision and drainage will be done in: a. The most necrotic part of the abscess b. The most bottom of the abscess c. Intra oral d. All the above

28-d,66-b,64-b

Pin in amalgam A. 1 mm from dej B.2 mm from dej

2mm into dentin and 2 mm in amalgam vertically 1 mm away dej horizontal

Benzodiazepines Metabolism

3 members that undergo Extrahepatic metabolism - Outside The Liver Oxazepam Temazepam Lorazepam PS : These do not form active metabolites

Extrusion of canine what flap technique is used except: 1)Envelope flap 2) Semilunar flap 3) Apically repositioning flap

3) Apically repositioning flap

1.What has ultimate effect on the thickness of epithelium of free Gingival graft. 1.Recipient epithelial tissue 2.Donor epithelial tissue 3.Donor CT 4.recipient CT q2.Epithelium of graft is attained from? 1.Recipient epithelial tissue 2.Donor epithelial tissue 3.Donor CT 4.recipient CT q3.source of Epithelium of graft is from? 1.Recipient epithelial tissue 2.Donor epithelial tissue 3.Donor CT 4.recipient CT q4.type of epithelium of graft is from? 1.Recipient epithelial tissue 2.Donor epithelial tissue 3.Donor CT 4.recipient CT

3,2,2,4

Which treatment has the least successful long term prognosis on a deep carious lesion on #3 1. Direct pulp capping 2. Indirect pulp capping 3. Pulpectomy 4. Pulpotomy

3- Least desirable in permanent teeth: pulpotomy and pulpotomy in permanent teeth leads to calcification. Least desirable in primary teeth: direct pulp capping

How long must pain last to be considered chronic? 1-2 3-6 7-12 12+ months

3-6months

gingivectomy contraindicated 1-suprabony pocket 2-fibromatic gingiva 3-bottom of pocket is apical to mucogingival junction

3-bottom of pocket is apical to mucogingival junction

minimal undercut on max tuberosity what would you do 1-remove undercut 2-reline 3-cover with denture base

3-cover with denture base

:when doing restoration prep, which bur type produces smoothest surface ? 1-cross-sectional fissured bur 2-plain sectioned fissured bur 3-diamond stone

3-diamond stone

:very short clinic crown, you prep for metal crown and it is lacking facio lingual resistance you would do 1-facial groove 2-lingual groove 3-proximal groove

3-proximal groove

The PH of Ca (OH) is equal to. 1-5.5 2. 7.5. 3. 12.5. 4. 19.5.

3. 12.5.

Most epidemiologic studies indicate that gingivitis in children is relatively common. A strong positive association between specific nutritional deficiencies and the presence of periodontal disease in children and adults has been demonstrated. 1. both r true 2. both r false 3. 1st true, 2nd false 4. 1st false , 2nd true

3. 1st true, 2nd false

What is not a benefit of a 3⁄4 crown over a FGC? 1. Better chance to do pulp testing 2. Less gingival irritation 3. Better retention 4. More esthetic

3. Better retention

For a lesion in enamel that has remineralized, what most likely is true? 1.The enamel has smaller hydroxyapatite crystals than the surrounding enamel 2.The remineralized enamel is softer than the surrounding enamel 3.The remineralized enamel is darker than the surrounding enamel 4.The remineralized enamel is rough and cavitated

3.The remineralized enamel is darker than the surrounding enamel

:How do you decrease the width of artificial teeth.? 1.Deepen the facial line angle proximally and increase the interproximal embrasure 2.Deepen the facial line angle proximally and decrease interproximal embrasure 3.take the facial line angle labially and increase the interproximal embrasure 4.take the facial line angle labially and decrease the interproximal embrasure

3.take the facial line angle labially and increase the interproximal embrasure

Pocket depth of 5mm and 2mm from CEJ and gingival margin what is the attachment loss

3mm example -Calculating CAL in the presence of Recession of the Gingival Margin When recession of the gingival margin is present, the CAL is calculated by adding the probing depth to the gingival margin level. Example: Probing depth measurement: 4 mm Gingival margin level: +2 mm* Clinical attachment loss: 6 mm * = 2 mm of tissue needs to be added for the gingival margin to be at its normal level.

Which units are missing on Williams probe?

4 and 6

What causes root caries? 1) diabetes 2) gingivitis 3) attrition 4) abrasion

4) abrasion

you prep a tooth for composite filling and it goes below enamel toward cementum, you should 1-resin composite 2-amalgam 3-glass ionomer 4-RMGI

4-RMGI

purpose of acid etch to enamel 1-chemical bone 2-remove moisture 3-remove collagen 4-none

4-none ( inc surface area, wetting, better mechanical lock, clean surface)

best xray to see zygomatic arch 1-ct 2-panorama 3-water's 4-submentovertex

4-submentovertex

For a surgical extraction, what does not contribute to developing post-operative pain? 1.High-speed drill 2. Low speed low torque drill 3. Sharp burs 4. High-speed drill with water spray

4. High-speed drill with water spray

2. which cyst is associate with anterior cyst w/ crown 3. common cyst is lower anterior teeth and teeth is vital periapical cemato dysplasia 4. know different between amelogenesis imperfecta and dentinogenesis imperfecta 5. which of the following represents the basic constituent of the most root canal sealer a. zinc oxide b. zinc stearate c. polyvinyl resin d. polycarboxylate e. zinc oxyphosphate 6. aging of the pulp is evidenced by an increase in a. vascularly b. cellular elements c. fibrous elements d. pulp stones 7. in the normal dental pulp, which of the following histologic features is least likely to appear a. cell-free zone of weil b. palisade odontoblastic layer c. lymphocytes and plasma cells d. undifferentiated mesenchymal cells 8 the most commonly found salivary gland tumor is a. adenocystic carcinoma b. pleomorphic adenoma c. muco epidermoid carcinoma 9 the action f the Hawley appliance is mainly a. intrusion b. tipping c. bodily movement 10 a light force applied to the periodontal ligament during orthodontic treatment is considered a. intermittent b. direct c. continuous d. indirect 11 the fluoride concentration in most dentifrices range from a. 1-5 ppm b. 900-1500ppm c. 450-700ppm d. 4000-6000ppm 12 in a full upper denture the post palatal seal is determined by a. the technician b. the depth of the vibration line c. 2-3mm 13 how is scrap amalgam is stored a. under water b. under sulfide c. glycerin 14 the leas likely situation for a carcinoma to occur in the oral cavity is a. floor of the mouth b. alveolar ridge c. lateral border of the tongue 15 histologically, the loss of the rete peg often is a sign of a. pemphigus b. lichen planus c. pemphigoid d. syphills 16 which of the following represents the predominate type cell type in crevicular epithelium a. mast cell b. PMN c. macrophage d. lymphocyte e. plasma cell 17 which of the organisms are involved with periodontal disease a. P. gingivilits b. E. species c. C. rectus d. Bacteroid e. all of the above 18 each of the following has been associated with gastric limitation, except a. acetaminophen b. alcohol c. ibuprofen d. indomethacin 19 the most common reason for fracture of an amalgam in class 2 pedo molar toth a. insufficient deth b. saliva contamination during condensation d. line angle too sharp 20 the best reason for RPD over fixed partial denture a. hygiene b. cooperation c. esthetic 21 where is the gold directed on an MO onlay spruce a, faces pulpal axial line angle b. occlusal floor c. pulpal floor d. gingival floor 22 which injection post the greatest risk for a hematoma a. PSA b. MSA c. mandibular block 23 why is the surgical stent required for an immediate denture a. to give an idea of the anatomy of the region b. prevent hematoma c. to determine occlusion 24 the least likely fracture site in the mandible will be a. coronoid b. condyle c. body of mandible d. ramus e. symphysis 25 which tooth will the matrix band be a problem with when placing a two surface amalgam a. mesial on maxillary first molar b. distal on maxillary first premolar c. mesial on maxillary second premolar d. distal on mandibular first molar 26 what is pulpectomy a. extirpating pulp chamber and canal completely b. partial instrumentation of the canal c. complete cleaning and shaping 27 the greatest appositional growth occur a. posterior border of the ramus b. anterior part of ramus c. chin 28 when do you do serial extraction a. for space deficiency in mandibular anterior region b. for space deficiency in mandibular posterior region c. for space deficiency in maxillary anterior region b. for space deficiency in maxillary posterior region 29 on what surface of the tooth is there deposition of F a. smooth surface b. pits c. fissures 30 how will treat patient with type 2 furcation a. tissue guided regeneration b. oral hygiene instruction and root planing c. reposition flap surgery 31 thumb sucking can cause all the following except a. deep over bite b. protruded maxillary incisor c. lingual tipping of the mandibular incisors d. anterior open bite how will treat a vital second molar with a 1.5mm exposure on a 12 years old patient a. apexification-ans b. endodontic treatment c. extract d. apicoectomy 32 bonding on a tooth does all of the following except a. chemical bonding b. mechanical bonding c. increase surface area 33 a patient complaint of recent severe pain with percussion of a tooth. The most likely cause is a. acute apical periodontitis b. chronic apical perio c. reversible pulpitis d. irreversible pulpitis 34 direct pulp capping is recommended for primary teeth with a. carious exposure b. mechanical exposure c. calcification in the pulp chamber d. all of the above 35 a calcium hydroxide pulpotomy performed on a young permanent tooth is judged to be succeful when a. the patient is asymptomatic b. the tooth responds to pulp testing c. normal tooth development continues all of the above 36 organism implicated on causing severe spreading abscesses include a. Fusobacterium b Campylobacter c. Enterococci d. Bacteroides 37 the periapical lesion that wound most likely contain bacteria within the lesion is a. an abscess b. a cyst c. a granuloma d. condensing osteitis 38 of the following periapical diagnosis, which most likely contain bacteria within the lesion a. suppurative apical periodontitis b. apical cyst c. chronic apical perio d. acute apical perio 39 a periodontal exam of a patient referred for endodontic treatment a. there is an inward flow of fluid b there is an outward flow of fluid c. there is no fluid 40 you fit a new completed denture and the patient complaint of cheek biting, what would you do a. grind buccal of lower teeth b. grind buccal of upper teeth c. grind lingula of lower teeth d. grind lingula of upper teeth 41 with juvenile periodontitis which teeth are predominantly involved a. first permanent molars and anterior teeth b. all deciduous molars c. all deciduous teeth 42 the optimal amount of tooth reduction on a molar for a metal ceramic crown preparation a. 1.5mm b. 1mm c. 2mm d. 2.5mm e. 3mm 43 which drug is LEAST likely to result in an allergy reaction a. epine b. procaine c. bisulfite d. lidocaine 44 when doing an endo treatment you hit a ledge, what are you going to do a. use a smaller instrument and get beyond the ledge b. fill as far as you have reamed c. use a small round bur and remove the ledge d. continue working gently to eliminate the ledge 45 when treating deep packets in a patient with juvenile perio, the least effective treatment is a. occlusal adjustment b. root planing c. antibiotic treatment d. perio surgery 46 the materials that will produce the best osseous regeneration is a. autograft b. allograft c. alloplastic d. simigraft 47 what is main reason to splint mobile perio involved tooth patients comfort 48 dysplasia is related to which of the following conditions. Please check a. leukemia b. diabetes c. pregnancy d. puberty 49 a patient with new denture can not make the S and TH sound, what is the problem a. extensive vertical overlap b. incisors place too far c. incisors placed too far lingually 50 what composite should ideally be used for a class 5 a. microfil because it polished better b. microfil because it is stronger c. hybrid because it polished better d. hybrid because it is stronger 51 what would you recommended for an 8 years old patient a. this is a normal eruption pattern b. refer for ortho c. refer to oral surgery 52 between which teeth is the primate space found in the mandible a. deciduous canine and first molar 53 what would you warm patient about who is taking birth control pills and requires Penicillian a. penicillin decrease effectiveness of birth control pill b. birth control pill decrease effectiveness of pen c. they may develop allergy 54 after doing RCT, the success can be determine by all the following except a. loss of periapical lucency on a radiograph b. formation of apical scar c. absence of pain d. absence of exudate 55 what is the impression materials with the best dimension stability 24 after taking the impression a. PVS b. reversible colloid c. irreversible colloid 56 what radiograph would you prescribe for a young patient who has no caries and where the molar teeth are contacting each other a. BW and occlusal b. FMS c. pan d. pan and ceph 57 how far should implants be placed from one another a. 3mm b. 4mm c. 5mm d. 7mm 58 what does of F are most effective a. small dose high frequency b. small doses low frequency c. high doses high frequency d. high doses low frequency 59 if a patient has SNA of 82 and a SNB of 87 wht type of malocclusion will be seen a. mandibular protrusion b. maxillary protrusion c. maxillary retrusion d. mandibular retrusion 60 in what part of the month are metastases seen most frequently a. mandible b. lateral border of tongue c. palate d. floor of the month 61 the greatest decrease in radiation to the patient/gonads can be achieved by a. change from D to F speed b. thyroid collar c. filtration d. collimation 62 meperidine overdose is treated with a. naloxone b. amphetamine c. nalbuphine and epine 63 what is the complication of up righting molar a. move distally and extrudes b. a class 3 molar relationship can develop c. class 2 molar relationship can develop 64 what materials will be used for a non vital pulpotomy in a primary tooth? ZOE as root canal filling can not do pulpotomy on a non vital primary tooth 65 a new patient comes in to see you with deep pockets. What will you initially do a. scaling and root planing b. gingivectomy c. WMF 66 when will the BULL rule be utilized with selective grinding a. working side b. balance side c. protrusive movement d. all the above 67 which of the following can be used for topical anesthesia a. lidocaine b. benzocaine 68 collimation a. reduce the size of the beam b. reduces the shape of the beam c. reduces radiation to the patient d. all the above 69 what can make porcelain crown lighter a. value b. chroma c. hue 70 multiple periapical lucency are common in patients with which of the following condition a. dentinal dysplasia b. taurodontia c. germination d. amelogenesis imperfecta e. dentinogenesis imperfecta 71 which of the following is the most common cause of TMJ ankylosis a. trauma b. otitis media c. rheumatic arthritis 72 which of the following are effects common to pentobarbital, diazepam and meperidine a. amnesia and skeletal muscle relaxation b. anticonvulsant and hypnotic c. analgesia and relief of anxiety 73 which of the following is the treatment of choice of a 7 year old child with a non vital permanent first molar a. apexification b. calcium hydroxide pulpotomy c. gutta percha root filling 74 which radiographs would be most effective in localizing a supernumary tooth and its relationship to other teeth a. 2 periapical views at different angles and an occlusal b. a periapical and an occlusal c. a periapical using a long cone d. a pan and a supplemental occlusal 75 the relative position of the maxilla to the cranial base will be determined cephalometrically by the angle a. SNA b. SNB c. Sn-GoGn d. ANB 76 pan showing lucency going inferior over the body of mandible close to the angle. Informed the patient was involved in an accident. Identify the lucency a. pharyngeal airspace b. fracture c. artifact-retake radiograph 77 with mandibular bilateral distal extension RPD, when you place pressure on one sides the opposite side lifts and vice versa, what is the problem a. no indirect retention used b. rests do not fit c. acrylic resin base support 78 a patient with a new denture has a problem pronouncing the F and V sounds. What is the problem a. maxillary incisors placed too far superior b. maxillary incisors placed too far superior c. mandibular incisors placed too far lingually 79 which of the following is the man side effect of bleaching of an endodontically treated tooth a. external cervical resorption b. demineralization of tooth structure c. gingival inflammation 80 which nerve involved with a Le Forte II fracture a. infra orbital b. greater palatine c. nasopalatine d. PSA 81 when treating deep vertical pockets in a patient with juvenile periodontitis, the least effective treatment is a. occlusal adjustment b. root planing c. antibiotic therapy d. periodontal flap surgery 82 sedative drug such as hydroxyzine, meperidine and diazepam are carried in the blood in a. serum b. white blood cells c. red blood cells d. hemoglobin 83 steroid causes a. adrenal gland suppression b. thyroid gland suppression c. parathyroid gland suppression 84 after opening the flap buccally in the maxillary premolar area, how will you suture it a. interrupted b. intermittent c. mattress 85 the materials that will produce the BEST osseous regeneration is a. autograft b. allograft c. alloplast d. simiograft 86 the best combination treating TB a. rifampin and isoniazid b. rifampin and penicillin c. ethambutol and penicillin d. rifampin and streptomycin 87 the distal palatal termination of the maxillary complete denture base by the a. tuberosity b. fovea palatine c. maxillary tori d. vibrating line e. posterior palatal seal 88 the strength of dental investment for gold alloys is dependent upon the amount of a. silica b. carbon c. copper d. gypsum 89 which med cause mydriasis atropine 90 what happen to curve of spee in a patient who has lost tooth #19 and #20 have drifted curve of spee reversed 91 what do you expect to occur in a child with an anterior edge to edge bite class 3 malocclusion 92 lateral bennet shift is most likely to affected by centric occlusion mesial distal steep incline, facial lingual steep incline 93 which of the following has the greatest coefficient of thermal expansion a. gold b. resin c. amalgam 94 with insufficient light cure, which area is softer (underpolymerized)? Core of the composite 95 what is advantage of a bonded bracket over a wire band no separator need 96 conjunctivitis hemorrhage is the result of which fracture nasal maxillary sinus zygomatic arch maxillozygomatic temporal 97 fracture of a rest of a chromium cobalt denture is due to a. over-finish and polish b. inadequate occlusal preparation 98 in office bleaching changes the shade through all except a. dehydration b. etching tooth c. oxidation of colorant d. surface demineralization 99 the lingual root of maxillary first molar radiographically appears mesial to the mesiobuccal root because the cone was directed from a. mesial b. distal c. superior d. inferior 100 you fit new completed denture and the patient complains of cheek bite, what will you do a. grinding buccal of lower teeth b. grinding buccal of upper teeth c. grinding lingual of lower teeth d. grinding lingual of upper teeth

5-a 6-c 7-c 8-b 9-b 10-c 11-c 12-b 13-b 14-b 15-a 16- 17-a 18-a 19-a 20-b 21-a 22-a 23-a 24-a 25-d 26-a 27-a 28-a 29-a 30-a 31-a 32-c 33-a 34-b 35-a 36-d 37-b 38- 39-a 40-c 41-a 42-d 43-d 44-a 45-b 46-a 47- 48-b or a 49-b 50-c 51- 52- 53-a 54-a 55-a 56-a 57-a 58-b 59-a 60-b 61-a or d 62-a 63-a 64- 65-a 66-b 67-a 68-c 69-a 70-a 71-a 72-a 73-a 74-a 75-a 76-c 77-b 78-a 79-a 80-a 81-a 82-a 83-a 84-a 85-a 86-a 87-d 88-d 89- 90- 91- 92- 93-b 94- 95- 96- 97-b 98-a 99-a 100-a

Desquamative gingivitis 1-LE 2-EM 3-pemphigus 4-pemphigoid 5-all of the above

5-all of the above(Desquamative gingivitis involves lesions of the free and attached gingiva. )

Facial weakness or facial paralysis is a likely symptom of which of the following conditions? (a) Mixed tumor. (b) Adenoid cystic carcinoma. (c) Mucoepidermoid carcinoma. (d) Papillary cystadenoma lymphomatosum. 1. (a), (b) and (c). 2. (a) and (c) only. 3. (a), (c) and (d). 4. (a), and (d) only. 5. (b) and (c) only. 6. (b)and(d).

5. (b) and (c) only.

Disinfection of impression matrial 1% naocl 3% h2o2 70% alcohol 90 alcohol Gluteraldehyde

5.25% NAOCL for 10 minutes and glutetaldehyde for alginate 2% gluteraldehyde for agar

Initial demineralization of enamel occurs at which pH? 6.9 6.5 5.5 4.5 1.5

5.5

Implant supported CD.. how many implants in both arches?

6 implants are placed in maxillary arch and 4 in mandibular arch in order for them to be sufficient to support complete dentures. Out of these, 4 are placed in anterior for maxi and 2 in anterior for mandi. The 2 in mandi anterior are placed in the canines

What age does mandibular symphysis fuse?

6-9 m

The first indication of the presence of juvenile periodontitis is most often: 1. Toothache 2. Traumatic occlusion 3. Marginal gingivitis 4. Painful gingiva 5. Hyperplastic gingiva 6. Sudden drifting of the teeth

6. Sudden drifting of the teeth

what percent cerebral palsy patient have mental retardation

60% or 2/3 rd have mental retardation.

Film developing time and temperature 650 F, 5 min 680 F, 5 min 700 F,4 min 720 F,3 ½ min

680 F, 5 min

cleft lip and palate 1 every 300 700 1,100 1,500

700

Minimum ridge width for 4mm implant is. 6mm, 8mm, 4mm, 10mm

8mm-4 mm diameter implant, I need 4.0+1.9 + 1.9 = 7.8 mm. .. you need at least 7.50-8.00 mm of ridge width.

What is the type of injury wherein extensive section of skin or mucosa is completely torn off the underlying tissue, severing its blood supply called? Impalement Avulsion Laceration Degloving

: Degloving: It is a type of injury wherein an extensive section of skin or mucosa is completely torn off the underlying tissue, severing its blood supply. It is named by analogy to the process of removing a glove.

Commonly used operation for correction of unilateral cleft lip Rose Thompson straight line repair V-Y palatoplasty Tennison-Randall triangular flap Skoog's procedure

: Several types of cleft lip operations have been described for unilateral cleft lip. The most commonly used operations are Millard's rotation advancement flap and Tennison - Randall triangular flap methods. Rose- Thompson straight line repair, the Skoog's procedure are less frequently used. The procedures like rectangular flap me-thod of Hagedorn-Le Mesurier are rarely used. Bilateral cleft lip can be repaired in two stages by the above mentioned procedure or in a single by Veau III procedure, Millard's single stage procedure or Black procedure.

4mm exposure - a-gold foil suture, b-flap, c- leave it d- antibiotics

<2mm leave 2-6mm figure of 8 >7mm flap

After performing root canal therapy on a mandibular anterior tooth, the post-obturation radiograph reveals a radiopaque horizontal line in the mid-root area that extends mesiodistally from one side of the periodontal ligament (PDL) space to the other. This finding is most likely indicative of which of the following? A mid-root fracture Accessory canals External resorption Internal resorption No answers are correct

A

Bur# 245 n 330.what is the shape? A. Pear B.inverted cone

A

Initial symptom of HIV? Reduction of lymphocytes Kaposis Candiasis Oral hairy leukoplakia

A

Which structure is least likely to show intramural radiograph? A) mandíbulas foramen B) mental foramen c) hamulus notch

A

Which thermal property is most important in selecting a restorative material to protect the pulp from excessive temperature changes? A. conductivity. B. diffusivity. C. expansion coefficient. D. modulus.

A

A dentist will remove a mandibular lingual torus from a patient's premolar region. This dentist should use an envelope flap design that A. has no vertical components. B. has an anterior vertical component only. C. has anterior and posterior vertical components. D. is incised at the junction of attached gingival tissue and free mucosa

A usually a horizontal incision along the gingival margin only, allows adequate access to the torus and no need for vertical incision which may injure the vital organs

The acid-etch technique used for composite restorations does not improve which of the following? A. Improved access for the finishing of the restoration B. Maximum conservation of tooth structure C. Improved retention of the restoration D. Improved esthetic outcomes

A Acid etching roughens the exposed enamel rods and dentin thus creating micromechanical abrasions that allow the bonding agent to flow and adhere to the tooth surface. Acid etching removes surface debris, allowing for a clean bonding site for improved esthetics. Acid etching creates higher bond strength between composite restoration and tooth structure and decreases microleakage. Acid etching does not improve access to the restoration.

he _______ of the Gingival marginal trimmer corresponds to the nib of the _______? A. blade, condenser B. shaft, forceps C. blade, periodontal probe D. shank, explorer

A , The blade of the Gingival marginal trimmer corresponds to the nib of the condenser. Gingival marginal trimmers are cutting instruments while condensers are non-cutting instruments. In non-cutting instruments, the blade is replaced by a nib or point. The working ends or nibs of the condensers may be of any shape but they are usually round with flat ends. Triangular, rectangular, or diamond-shaped nibs are also used.

Tooth displacement, worst prognosis: a. Intrusion b. Extrusion c. lateral displacement

A 96 % then lateral luxation 80% and then extrusion 65%

complex vs compound odontoma

A compound odontoma (enamel, dentin and cementum), A complex type is unrecognizable as dental tissues.

What is clear acrylic stent and its use?

A dental surgical stent is an acrylic device made from a model of the patient's mouth that fits over the area where dental implants are to be placed and helps guide the oral surgeon in correct placement of implants. The stent contains holes that are pre-drilled

fact witness?

A fact witness is an individual, sometimes a clinical professional such as a forensic psychologist, who has personal knowledge of events pertaining to the case can testify as to things they have personally observed or witnessed

Which of the following is the most appropriate initial treatment for a patient with HIVassociated necrotizing ulcerative gingivo-periodontitis? A. Debridement and antimicrobial rinses B. Definitive root planing and curettage C. Administration of antibiotics D. Gingivectomy and gingivoplasty

A followed by C

how many xrays we take in full mouth series? PA and bitewings??

A full mouth series is a complete set of intraoral X-rays taken of a patients' teeth and adjacent hard tissue.[1] This is often abbreviated as either FMS or FMX (or CMRS, meaning Complete Mouth Radiographic Series). The full mouth series is composed of 18 films, taken the same day: four bitewings two molar bitewings (left and right) two premolar bitewings (left and right) eight posterior periapicals two maxillary molar periapicals (left and right) two maxillary premolar periapicals (left and right) two mandibular molar periapicals (left and right) two mandibular premolar periapicals (left and right) six anterior periapicals two maxillary canine-lateral incisor periapicals (left and right) two mandibular canine-lateral incisor periapicals (left and right) two central incisor periapicals (maxillary and mandibular)

While performing a root canal you separate a file 1mm short of the apex. The file cannot be removed and by-passed by the operator. Which of the following treatments is indicated for managing such an occurrance after the dentist finishes the root canal treatment? Place the patient on a recall schedule for further evaluation Perform an apicoectomy of the mesiobuccal root Extract the tooth Amputate the mesiobuccal root

A is the right answer ! The dentist must evaluate and reassess the tooth to check for signs of disease progression. Having a root canal-treated tooth with a broken file inside poses some risks of reinfection due to the incomplete sealing of the canal. The process of instrumentation, cleaning, and shaping is compromised because of the broken file, which may result in a failure of the root canal treatment later. The root canal-treated tooth with an unretrieved broken file may stay inside the mouth if the tooth is asymptomatic and if a radiographic lesion shows evidence of healing. The tooth may undergo other complicated procedures such as retrograde filling, hemisection, and in hopeless cases, extraction if symptoms and infection persists. The decrease of this particular tooth's prognosis is less than if it had a necrotic pulpal diangosis because the irreversible pulpitis diagnosis suggests that the infection has not reached the apical end of the root cana

Which of the following is NOT a malignant lesion of the gingiva? A. Neurofibroma. B. Proliferative verrucous leukoplakia. C. Sarcoma. D. Squamous cell carcinoma

A it's malignant form is called neurofibrocarcinoma

Osseoperception ?

A phenomenon of developing some amount of tactile sensation in osteointegrated implants in absence of pdl mecenoreceptors

FDA want to check safety medicament A. RCT B. Case controlv

A randomized controlled trial (or randomized control trial;[2] RCT) is a type of scientific (often medical) experiment which aims to reduce bias when testing a new treatment. The people participating in the trial are randomly allocated to either the group receiving the treatment under investigation or to a group receiving standard treatment (or placebo treatment) as the control. Randomization minimises selection bias and the different comparison groups allow the researchers to determine any effects of the treatment when compared with the no treatment (control) group, while other variables are kept constant. The RCT is often considered the gold standard for a clinical trial. RCTs are often used to test the efficacy or effectiveness of various types of medical intervention and may provide information about adverse effects, such as drug reactions. Random assignment of intervention is done after subjects have been assessed for eligibility and recruited, but before the intervention to be studied begins.

A drug with high LD50 and low ED50 has high therapeutic index, therefore relatively safe. TRUE OR FALSE

A small therapeutic index indicates that a drug is UNSAFE while a large therapeutic index indicates that the drug is relatively SAFE. LD50/ED50 ratio: if lethal dose is 100mg and the effective dose is 10mg there is a 10X difference and indicates that it should be safe to take the medication at the 10mg level. LD50/ED50 ratio: if lethal dose is 20mg and the effective dose is 10mg there is only a 2X difference and indicates that- the highest dose required to produce therapeutic effects, I.e., the drug is not very safe.

Patient with manual dexterity will have a problem with? A) flossing B) brushing

A) flossing

what is the ideal length for a post in post-core in an endodontically treated tooth A- 2/3 rd of the root length B- 1/2 of the tooth length C- 1.5 times that of the crown D- same as the anticipated crown

A- 2/3 rd of the root length

Recommended daily intake of fat Intake as per USDA is A- 30% of total daily caloric intake B- 10% saturated fat intake of caloric intake c- both d- non

A- 30% of total daily caloric intake

Which of the following statements is true regarding Class IV composite restorations? A. A final veneer layer of micro-filled resin will help create a smooth, glossy surface. B. The usage of retentive pins for support on Class IV restorations is recommended. C. Tooth fragments should never be reattached to the remaining tooth structure. D. No answers apply

A- A final veneer layer of micro-filled resin will help create a smooth and glossy surface due to its small-sized particles. Polished surfaces are less receptive to plaque accumulation and extrinsic staining. Retentive pins are never recommended for support and retention in Class IV restorations.

More tooth suseptible to caries A. Max 1st molar B. Mand 1st molar

A- according to shafer's text book page 441 6th edition

What is contraindicated in asthmatics? Cholinergics, Anticholinrgics, Both, None

A- because produce bronchoconstriction(Inhaled anticholinergics are usually used for severe asthma attacks)

10 yrs pt primary mand molar missing which space maintainer - a-no space maintainer required, b-band & loop, c- distal shoe d-removable acrylic

A- no space maintainer required

. Primary incisor 5mm intrude - a-observation, b- splint, c-reposition, d-splint & reposition

A- observation

Little girl had ALL, had radiolucency in furcation of primary 2nd molar. What is the treatment? • Extraction • Pulpotomy • Pulpectomy

A- should be done before radiation therapy.because the contra indications of pulpotomy are : Contraindications for Primary Tooth Pulpectomy • 1. Teeth with nonrestorable crowns, • 2. Periradicular involvement extending to the permanent tooth bud, • 3. Pathologic resorption of at least one-third of the root with a fistulous sinus tract, • 4. Excessive internal resorption, • 5. Extensive pulp floor opening into the bifurcation, • 6. Systemic illness such: as congenital or rheumatic heart disease, hepatitis, leukemia, and children on long-term corticosteroid therapy, or those who are immunocompromised, • 7. Primary teeth with underlying dentigerous or follicular cysts.

drug used for a.petit mal b.grand mal c.temporal epilepsy?

A- valproic acid, ehosuximide B-carbamazepine, phenytoin c-carbamazepine, valproate, and phenobarbital

Which of the following locations would a perforation demonstrate the best prognosis? Apical 1/3 of root Middle 1/3 of root Coronal 1/3 of root Floor of pulp chamber

A-Apical perforations occur through the apical foramen or the body of the root to form a new canal. The more apical the perforation the more favorable the prognosis for the tooth (except for perforations of the crown).

Displacement of a drug from protein binding sites is expected to increase the a. drug effect observed b. duration of drug effect c. dose required for a given effect d. none of the above

A-Displacement of drugs from plasma proteins -can affect the pharmacokinetics of a drug in several ways: -Directly increase the free drug concentration as a result of reduced binding in the blood. -Increase the free drug concentration that reaches the receptor sites directly, causing a more intense pharmacodynamics (or toxic) response. -Increase the free drug concentration, causing a transient increase in Vd and decreasing partly some of the increase in free plasma drug concentrations. -Increase the free drug concentration, resulting in more drug diffusion into tissues of eliminating organs (e.g., liver and kidney), resulting in a transient increase in drug elimination.

What is maturity: A) Environmentally dependent B)environmentally independent

A-In psychology, maturity is the ability to respond to the environment in an appropriate manner. This response is generally learned rather than instinctive.

schedule 2 drug vicodin percocet hydrocodein+ibuprofen

A-The Drug Enforcement Administration is reclassifying so-called "hydrocodone combination products" from Schedule III to Schedule II under the Controlled Substances Act, which will more tightly restrict access. Vicodin, for example, is an HCP because it has hydrocodone and acetaminophen

Glass fiber post compared to custom made post Aesthetic. Tooth conservation. Less chances of fracture.

A-for custom made posts, it's esthetics. Because for custom made posts, you do not need to remove a lot of root structure in order to fit it in (since you take an impression of the canal space and cast it into a metal that fits accurately (and passively) inside the root canal). Having that been said, both fiber/glassfiber and custom made posts reduce risk of fracture because: 1. Custom made = less root removal 2. Fiber/glass fiber posts have a modulus of elasticity close to or similar to that of dentin, and so they flex with the root dentin under stresses in the same movement, and therefore reduce the internal stresses that might be caused by a post that has a different modulus of elasticity than dentin (like metal) So answer is A. Because glass fiber posts are transparent and thus do not show through the composite core/ceramic crown.

Pt has a deficiency in pseudocholinesterase, if you give one of the following drugs which will be prolonged? a. D-tolucurarine b. Succinylcholine

A-in pseudocholinesterase deficiency you have prolonged paralysis when given something like succinylcholine they will have prolonged paralysis.

Radiation injury is caused by which of the following? A- free radical formation from indirect. B- free radical from direct

A-indirect theory of radiation injury suggests that x-ray photons are absorbed within the cell and cause the formation of toxins, which damage the cell.frequently because of the high water content of cells.

which tooth most likely to lose from perio dz? mx molars, mx anteriors, md molars, md anteriors

A-maxillary 2nd molar

Which of the following glucocorticoids is the strongest a. Dexamethasone b. Corticoid c. Hydrocortisone

A-steroid drugs: strongest to weakest glucocorticoid. dexamethasone > fludracortisone (but high mineralcorticoid potency) > 6a methylprednisone

Band & loop has nt feature - a-vertical stop for opposite teeth, b-causes decalcify after bands looses cemented, c-plaque accumulation

A-vertical stop for opposite teeth

When does calcification first molar end: A. 2-3 years B. 4-5 years C. 6-7 years

A. 2-3 years

How does the pontic looks like? A. Convex in MD, Concave in FL B. Concave in MD, Convex in FL

A. Convex in MD, Concave in FL

After TMJ disorder treatment all the following will be fixed except Deviation Pain Dysfunctions

A. Deviation. TMJ disorder treatment is not focused on deviation treatment. Also, once mandible adjusts with deviation, it is very hard to correct it. Deviation does not necessarily cause pain so is unimportant for treatment.

Which graft is the preferable A. Osteoinductive B. Osteoconductive

A. Osteoinductive

how do u slow metabolism of lidocaine ??By addidng - a. propanolol b. epinephrine

A. Propanolol. Epinephrine only stops it from moving out of site for metabolism through enforcing local vasoconstriction. Propanolol - slows metabolism. Epinephrine - delays metabolism.

Mandible follows which growth curve A. Somatic B. Genital C. Neural D. Lymphoid

A. Somatic

First bone formed in respinse to orthodontic loading is A. Woven bone B. Composite bone C. Bundle bone D. Lamellar bone

A. Woven bone

Q4: Patient trouble keeping her lower complete denture in because of a low ridge, how to fix this problem? A. ridge augmentation B. place anterior implants C. alveolar bone sectioning

A. ridge augmentation

Which of the following radiolucency is closest to dentin? A)ZOE B)Calcium phosphate C)Silicophosphate D)Gutta percha

A?

Early signs of Malignant Melanoma

ABCDE Asymmetry Borders ( irregular ) Color (multi colored) Diameter ( more than 6mm) Evolving over time

A 32 year old male complains of weight loss and diarrhea. The clinical examination shows lymphadenopathy, multiple flat erythematous lesions on the palate and a linear gingival erythema. What is the most likely diagnosis? AIDS Crohn's disease Diabetes mellitus Leukemia

AIDS

What is the purpose of beveling while doing as composite resin restoration?

ANS-Esthetic, reduce microleakage and Retention

Acid etching is done for the aesthetic purposes also...T/F What is the acid used for etching

ANS-False on tooth- 37% Phosphoric acid Porcelain- Hydro fluoride acid

ZOE cannot be used as base or liner for composite restoration -----why?

ANS-Interferes polymerization

X-ray shows cupped out or crater shaped radiolucency just below CEJ...what is the diagnosis

ANS-Root surface caries

Initial mixture pH of zinc phosphate cement is _________?

ANS-Very acidic...pH =3.5

Pulse rate in an: 1)adult 2) child 3)newborn

Adult 70, Child 75- 120 , Newborn 120- 160

Main advantage of doing direct composite over composite onlay?

Advantage of Direct composite over comp. inlay/onlay--- 1. Less time consuming due to lack of lab procedure 2. Easy restorative technique (no extensive lab procedures involved) 3. Cost effective 4. No dependency on lab technicians n multiple persons,so technical errors are less 4. Marginal leakage due to polymerization shrinkage is not there as it is polymerized extra-orally & 5. Open contacts r not dere n proximal contours r more defined as its fabricated in lab.... 6. Physical properties r also superior due to extraoral complete polymerization

main advantage of doing direct composite over composite onlay? a. less shrinkage b. better marginal adaptation c. greater hardness and wear resistance

Advantage of Direct composite over comp. inlay/onlay--- 1. Less time consuming due to lack of lab procedure 2. Easy restorative technique (no extensive lab procedures involved) 3. Cost effective 4. No dependency on lab technicians n multiple persons,so technical errors r less Advantages of Composite Inlay/Onlay over direct composite restorations--- 1. Marginal leakage due to polymerization shrinkage is not there as it is polymerized extra-orally & 2. Open contacts r not dere n proximal contours r more defined as its fabricated in lab.... 3. Physical properties r also superior due to extraoral complete polymerization

alcohol content of hand gels used in dental office?

Alcohol-based hand rub. An alcohol-containing preparation designed for application to the hands for reducing the number of viable microorganisms on the hands. In the United States, such preparations usually contain 60%--95% ethanol or isopropanol.

Main function of the occusal guard: • Distribute forces more evenly • To relax the musculature • reduce Bruxism

All

mangement of hiv pt in dental office ?

All dental practices should be able to provide routine dental care for adult or pediatric HIV-positive patients. -dental treatment planning must be done on an individual basis, in conjunction with consultations with the patient and their physician as appropriate. -HIV and antiretroviral therapies may be associated with abnormal bleeding, glucose intolerance, or hyperlipidemia, which may be identified through consultation with the patient and their physician. -Indications for dental extractions and other oral surgical procedures are the same for HIV-positive patients as for any other patient. chlorhexidine -0.12%- 0.2 rinse twice daily metronidazole , pencillin can be prescribed Lesions strongly associated with HIV/ AIDS • Oral candidiasis • Oral hairy leukoplakia • Kaposi's sarcoma • Non-Hodgkin's lymphoma • Periodontal diseases - Linear Gingival Erythema Necrotizing Ulcerative Gingivitis Necrotizing Ulcerative Periodontitis Most oral candidal infections are associated with candida albicans • Candidiasis is the most common oral lesion in HIV diseases and found in 90% AIDS patients It has 4 clinical presentations : 1. pseudomembraneous candidiasis 2. erythematous candidiasis 3. hyperplastic candiasis 4. angular cheilitis topical-CLOTRIMAZOLE 10mg tablets: dissolve in mouth 3-5tablets daily for 7-14days • NYSTATIN a) Ointment 15g b)oral Systemic drugs • Ketoconazole 200mg tablets: 2 tablets immediately, then 1-2 tablets daily for 5- 14days. • Fluconazole 100mg tablets: 2 tablets immediately, then 1 tablet daily for 7-14 days. • Itraconazole 100mg capsules: 200mg once daily with meals for 4 weeks. Implants Contraindicated

Flexing of a fixed partial denture under occlusal loads is A. proportional to the cube of the width of its pontics. B. linearly proportional to the length of the span. C. decreased with the use of a high strength material

All fixed partial dentures, long or short spanned bend and flex under masticatory load Bending or deflection varies directly with the cube of the length and inversely with the cube of occlusogingival thickness of the pontic ( D α L 3 ) where D is defelection, L is length of the pontic span on given occlusogingival height Compared with a fixed partial denture having a single tooth pontic span, a two tooth pontic span will bend 8 times as much Double abutments are sometimes used as a means of overcoming problems created by unfavourable crown-root ratios and long span.

Improper temporary coverage of bridge abutments can cause tooth migration occlusal prematurities increased tooth sensitivity All of the above gingival recession

All of the above

Which of these is an etiology for tongue thrusting? Retained infantile swallow Neurological disturbances Respiratory tract infections All of the above

All of the above

In an otherwise acceptable occlusion, an impacted maxillary canine constitutes a problem requiring consultation with an orthodontist could be surgically exposed to speed its eruption could be retained and the first premolar removed to allow the canine to erupt. could be extracted All of the above.

All of the above.

What is slumping down during firing of low fusing porcelains?

Aluminum oxideis added to low-fusing dental porcelains(during its manu-facture)in order to increase its resistance to "slumping down" during firing.

Should oral surgical procedures be postponed in patients taking aspirin?

American Academy of Oral Medicine reported the results regarding 36 patients taking 325 mg Aspirin 2 days before and 2 days after the tooth extraction. There was no difference in bleeding outcomes between the two groups who received the treatment.

potassium sparing diuretic?

Amiloride Eplerenone (Inspra) Spironolactone (Aldactone) Triamterene (Dyrenium)

Which of the following should a dentist consider primarily when restoring the incisal edge or enameloplasty for a patient with slight chipping of the incisal edge of teeth #8 and #9? Height of the smile line Shape of incisal embrasures Location of proximal contacts Amount of translucent enamel present

Amount of translucent enamel present

Foliate papilla associated with A. Hairy tongue B. Leukemia C. Median rhomboid glossitis D. Scarlet fever

Ans is B leukemic feature on the side of tongue " foliate papilla # median rhomboid glossitis red lesion devoid of fili # geographic tongue desquamation of fili # hairy tongue hypertrophy of fili. # scarlet fever inflamed fungiform

arrange in order of greatest fluoride release to lower fluoride release- conventional GIC RMGI Composite resins Compomers

Ans- 2143

Flap WORKHORSE of periodontal therapy?

Ans-apical displaced flap

Nitroglycerin causes all except A Hypotension and bradycardia B Methemoglobinemia C Hypotension and tachycardia D Vasodilation

Ans. A Hypotension and bradycardia a. Nitrates are most commonly used antianginal drugs. These act by delivering nitric oxide in the blood vessels leading to vasorelaxation. As per Merry's law, hypotension is followed by tachycardia and not bradycardia. b. Constipation and gastroesophageal reflux disease is a common problem in patients taking nitrates. Nitrates act by relaxing the smooth muscles.

1. Examination reveals area of gingival recession, exposed wide area of dental roots. Which is the procedure of choice to obtain coverage of the root surface A. Free gingival autograft B. Sub-epithelial tissue graft C. Apically positioned graft D. Free gingival graft E. Modified wide flap 2. What does contra indicate distal wedge in molars' area A. Distal fluting B. Long attached gingiva C. Sharply ascending ramus that limits space distal to molars D. Supra bony pockets distal to molars 3. Which of the following is not a property of Fluoride ion A. Crosses placental barrier B. Deposits in bone C. Excretes rapidly by kidney D. Bacteria static E. Produces extrinsic tooth stain 4. Two conditions of enamel facilitate post eruptive uptake of fluoride element A. Hyper mineralisation and surface dentine B. Surface demineralisation and hypo mineralisation C. Dental fluorosis and enamel opacities 5. In regards to topically applied fluoride A. Effective in incorporated into dental plaque B. Inhibits acid demineralisation of enamel 6. Flexibility of the retentive clasp arm Does not relate to A. Length B. Cross section C. Material D. Degree of taper E. Under cut area 7. How long it would take to see the dentinal bridge after direct pulp capping by using Calcium hydroxide. A. 6-8 weeks B. 4 weeks C. 6-8 months D. 4 months 8. What is contraindicated to the use of calcium hydroxide for pulp capping A. Accidental exposure of pulp B. Carious exposure of pulp in otherwise asymptomatic tooth C. Carious exposure of pulp in tooth that has been painful for weeks 9. How would you treat hyperaemia "hyperaemic tooth" A. Zinc Oxide and eugenol cement B. Calcium hydroxide C. Corticosteroid paste 10. Bilateral symmetrical swelling of the mandible of a child is likely to be caused by A. Acromegaly B. Paget's disease C. Giant cell lesion D. Primordial cysts E. Dental cyst

Answers Q1=A double check Q2=C Q3=E Q4=C Q5=A & B Q6=E Q7=A, 4 weeks in indirect pulp capping and 6-8 weeks in direct pulp capping Q8=C Q9=C, Useful when there is a hyperaemic pulp and failure of local anaesthesia; most commonly when there is an irreversible pulpitis and/or carious exposure. Use of these pastes may cause relief of symptoms, decreased inflammation with ability to successfully anaesthetise the tooth on the next Q10=C, like cherubism

If both condyles break what do you get?

Anterior open bite without deviation of midline.. if fragments over ride then premature contact of posterior teeth will be seen

Advantage of gutta percha? Antimicrobial Adapts to tooth surface

Antimicrobial

The purpose of barrier is to prevent : 1- Apical movement of PDl cells, 2-Coronal movement of cells

Apical movement of PDl cells

Type of bone formation when using a semi rigid fixation fracture apparatus: 1)Callus formation 2)Primary and secondary bone formation 3)Primary bone formation

As mentioned in DD , rigid fixation healed by primary healing without callus formation Semirigid healed by both primary and secondary Nonridid fixation healed by secondary bone formation

Which of the following describes the dentinal tubules as they become closer to the pulp? Density increases and diameter decreases Diameter and density remain the same Diameter and density increases Diameter and density decreases

As the dentinal tubules approach the pulp, there is an increase in: Density Diameter Permeability Because the tubule diameter is wider, the tubules become more permeable toward the pulp.

A 14 year old boy with Class II Div I malocclusion. Which model analysis method is used decide, whether to do extraction or not? Bolton's Peck and peck Ashley Howe's Pont's

Ashley Howe's( coz class 1 associated with constricted arch so need to increase arch width so Ashely Howe index analysis has to be undertaken)

Which of the following statements is correct about Quantitative Light or Laser Induced Fluorescence used to aid in the early detection of caries? Low specificity when used to detect smooth surface caries. Unable to detect caries around existing restorations. Assesses light reflection as opposed to light transmission. High sensitivity when used to detect interproximal caries.

Assesses light reflection as opposed to light transmission.

Difference btw quality assessment and quality assurance?

Assessment= evaluate & assess Assurance = evaluate & take correcting steps. n former u are only assessing the quality but in assurance you are assuring that you provide good quality by taking steps to improve quality

how many xrays we take in kids PA and bitewing?

At age six, typical to expose 12 radiographs; 10 pa's, 2 bitewings.

After finish class V glass ionomer cement, we do finishing with: a. Pumis slurry b. Aluminum-oxide paste c. Polishing bur d. All the above

B

Which of the following is NOT a reason why fluoride prevents tooth decay? Interference of bacterial metabolic activity Prevention of bacterial aggregation Remineralization of tooth structure Decreased enamel solubility

B Fluoride is essential to strengthen teeth and making them resistant to acid demineralization. Fluoride remineralizes the portions of teeth that have been previously demineralized by acids produced by bacteria or by dietary acidic fluids like soda pop. Fluoride replaces the hydroxyl groups of the hydroxyapatite crystals of enamel. It then forms fluoroapatite crystals that are significantly more resistant to acid.

Paul aged 8 years fell from roof and fracture his mandible into multiple pieces which communicates with oral environment: Type of fracture involved is: ( select all that apply ) Simple Comminuted Compound Green stick

B and C here

Baby-bottle caries most commonly affects which of the following teeth? Maxillary molars Maxillary incisors Mandibular molars Mandibular incisors

B is the right answer Feeding bottles often rest against the palatal surfaces of the maxillary anterior teeth, exposing them to a cariogenic environment for prolonged periods of time. Maxillary anterior teeth are most commonly affected in baby-bottle caries because of bottle positioning. Baby bottles are often left inside the mouths of babies, even while they are sleeping, which provides the baby with a continuous supply of cariogenic environment.

Predominant immunoglobin in gingival fluid is A)IgA B)IgG C)IgM D)IgE

B)IgG

Best radiographic view of condylar and ramus fracture is A)Towne's view B)Reverse Towne's view C)Water's view D)caldwell view

B)Reverse Towne's view

The only antibiotics in periodontal therapy to which all stains of actinomycetans comitans are susceptible to A)azithromycin B)ciprofloxacin C)cvefidipina D)metronidazole

B)ciprofloxacin

which is used for prophylaxis of migraine? carbamazepine topiramate ethosuximide phenobarbitone

B-First-line therapies for migraine prophylaxis in adults include propranolol (Inderal), timolol (Blocadren), amitriptyline, divalproex (Depakote), sodium valproate, and topiramate (Topamax).

The junction between primary and secondary dentine is A. A reversal line B. Sharp curvature C. A resting line D. A reduction in the number of tubules

B-The primary dentin is secreted at a relatively high rate and constitutes the major part of the dentin in the tooth. It is regular in structure and contains dentin tubules that form an S-shaped primary curvature as a result of the directional movement of odontoblasts, which have a cell process that extends from a broad peripheral border towards a narrow, centrally located cell layer. After tooth eruption, the odontoblasts continue to lay down dentin but slightly change their direction, which contributes to bending of the dentinal tubules. It is referred to as secondary dentin and is synthesized at a much lower rate and is less regular in structure.

Worst cantilever you can mak? A) premolar to molar B) lateral to cntral

B. Ideally Cantilevers are only acceptable when premolars or lateral incisors are to be replaced.

For most patients affected with periodontitis, what is the recommended interval for maintenance appointments? A. 1 month B. 3 months C. 6 months D. 1 year

B. 3 months

Removal of primary upper 2nd molar is done by which of following forces A. More buccal,less lingual B. More lingual,less buccal C. Slightly buccal and rotational D. Apical and buccal

B. More lingual,less buccal

Saucering out of small pits to improve self cleansing property is known as A. Enameloplasty B. Odontotomy C. Saucerisation D. Osteotomy

B. Odontotomy

5 diastema, elastic bands around central inc. What will happen? A. More control over tooth mov w rotation B. Teeth will become necrotic C. Diastema closer

B. Teeth will become necrotic ( Wrapping an elastic around your teeth will NOT achieve the same result. In fact, it can harm the gums and connective tissue, and result in irreversible damage and tooth loss)

Quad helix A. Used in cleft lip B. Widens upper arch C. Retracts maxillary canine D. None of above

B. Widens upper arch

A 50 year old edentulous patient is complaining that spicy foods are no longer enjoyable since he received his new complete upper and lower dentures. The most likely reason for this occurrence is that the A. dentures cover his taste buds. B. free nerve endings are now covered by the dentures. C. dentures are obstructing air flow to the olfactory epithelium

B. free nerve endings are now covered by the dentures.

Not enough room when edentulous pt closes. what do you do? A.adjust mandibular denture B. tuberosity reduction c. adjust maxillary denture

B. tuberosity reduction

cause of angular cheilitis

B2-riboflavin vitamins and iron deficiency anemia Allergic contact cheilitis mouth breathing candida decrease vdo edentulism

what is BMI with metabolic syndrome?

BMI is >30 kg/m², central obesity can be assumed

difference of BSSO and Vertical osteotomy

BSSO- PROCEDURE is postero anterior sagittal split , open procedure, along the IANERVE , so possible nerve exposure. rigid internal fixation,no maxillary mandibular fixation. VERTICAL OSTEOTOMY- lateral medial cut , blind procedure, no exposure of IANerve , no bone fixation, maxillary mandibular fixation is necessary.

In which of the following, the Balanced force technique gives the best result? Step-back approach Step-down approach Anti-curvature filing Double flare

Balanced force After many years of experimentation, Roane et al. introduced the "balanced force" concept of canal preparation in 1985. The concept came to fruition, they claimed, with the development and introduction of new K-type file design, the Flex-R file (originally manufactured by Moyco Union Broach, now Miltex, York PA). the technique can be described as "positioning and preloading an instrument through a clockwise rotation and then shaping the canal with a counterclockwise rotation., The authors evaluated the damaged instruments produced by the use of this technique and discovered that a greater risk of instrument damage was associated with clockwise movement. For the best results with the "Balanced Force" technique, preparation is completed in a step-down approach. The coronal and mid-thirds of a canal are flared with GG drills, beginning with small sizes, and then shaping with hand instrument is carried out in the apical areas. Similar to techniques described above, increasing the diameter of the coronal and mid-thirds of a canal removes most of the contamination and provides access for a more passive movement of hand instruments into the apical third. Shaping becomes less difficult: the radius of curvature is increased as the arc is decreased. In other words, the canal becomes straighter and the apex is accessible with less flexing of the shaping instruments.

barbiutrate elevate pain threshold?t/f

Barbiturates, however, differ from some other Sedative-Hypnotic drugs in that they do not elevate the Pain threshold

Best brushing technique

Bass- Best Scrub- Most commonly Sulcular- perio Stillman- Recession Fone- first taught to children.

Microscopic appearance showing actively proliferating osteoblasts with increased vascularity & moderate number of multinucleated giant cells is a feature of: Benign osteoblastoma Central giant cell granuloma Fibrous dysplasia Osteoid osteoma

Benign osteoblastoma

Mechanism of Alprazolam?

Benzodiazepines bind to gamma aminobutyric acid (GABA) receptors in the brain and enhance GABA-mediated synaptic inhibition; such actions may be responsible for the efficacy of alprazolam in anxiety disorder and panic disorder.

What happens to the width in the maxillary arch ? (6 to 18)

Bi canine: 5 mm increase Bi molar: 4 mm increase

What happens to the width in the mandibular arch? (6 to 18)

Bicanine: 3mm increase Bi molar: 2mm increase

Bleeding after tooth extraction??? Secondry Mostly capillary in nature May be from nutrient vessels

Bleeding which occurs several days after tooth extraction is known as secondary bleeding

Elongated melanocytes packed with melamin and found in lower third of dermis usually oriented parallel to dermis is histological feature of ________ type of nevus: Junctional nevus Compound nevus Blue nevus Intradermal nevus

Blue nevus

bodily movement a- labial and lingual inclination b- mesiodistal movement

Bodily Movement: This is the most desirable type of tooth movement in Orthodontic treatment. Both the crown and root move in the same direction either labially or lingually the same amount of distance

Bone morphogeny protein(BMP) - vipppp osteoinductive osteoconductive osteogenic

Bone morphogeny protein(BMP) - osteoinductive, osteoconductive, osteogenic(if this missing than go for osteoinductive as lat comes osteo inductive) Bone grafting materials are generally evaluated based on their osteogenic (ability to induce the formation ofnew bone by cells contained in the graft), osteoinductive (ability of molecules contained in the graft to convert cells into osteoblasts), or osteoconductive (ability of the graft material to serve as a scaffold that favors outside cells to penetrate the graft and form new bone) potential

Acc to silness and loe score that elicit bleeding 0 1 2 3

Bop 2 spontaneous 3

Bisphosphate used to treat Multiple myeloma Osteomyelitis Both None

Both

Which of the following bur gives most conservative tooth preparation design ? No 245 No 330 No 169 L No 160

Burs used for conservative cavity for amalgam restorations · No. 245 inverted cone bur · No. 330 bur (pear shaped) · No. 169L bur The no. 330 bur is a small and pear shaped version of No. 245 bur. This pear-shaped bur is indicated for the most conservative amalgam preparations.

Which of the following laboratory tests would be MOST important in staging the progress of this patient's HIV disease? A. ELISA test B. Western Blot test C. T-cell count D. All of the above

C - A normal CD4 count is from 500 to 1,500 cells per cubic millimeter of blood. It is more important to pay attention to the pattern of results than to any one test result. In general, HIV disease is progressing if the CD4 count is going down.

Theory affinity receptor antagonist bind and intrinsical activity agonist bind no intrinsical activity more receptor bind better response

C because the affinity is the term that refers to the attractiveness of drug to its receptor

Major c factor Less c factor

C factor is ratio of bonded to unbonded surface and it is high in class 1 and 5 And low in class 4

A mature pulp does all EXCEPT one of the following. Which is the EXCEPTION? Provides the nerve blood supply to the tooth Forms dentin Forms enamel Forms cementum

C is the right answer Mature pulps lay down tertiary dentin throughout life for protection from physical or chemical insult. Pulps contain capillaries that approximate the dentin just below the layers of odontoblasts, and nourishes the entire tooth. Pulp is also innervated by autonomic nerve fibers that control smooth muscles of arterioles and the afferent sensory fibers coming from the trigeminal nerve.

ALL of the following are symptoms associated with a Patients undergoing chemotherapy except one which one is the exception? A) Parotitis. B) Mucositis. C) angular vertical bone defect

C) angular vertical bone defect

You have HIV+ pt you can do all of the following except a) treat with metronidozole b) free gingival graft c)prophylaxis to treat candidiasis

C- because given prophylaxis to treat candida in HIV patient increase resistance to the infection curing

What is the best predictor for pulpal anesthesia? A-Concentration of anesthetic B-Volume of anesthetic C-Back pressure D-Type of anesthetic

C-PATHWAY OF THE PULP: strong back-pressure is a major factor in achieving successful intrapulpal anesthesia.depositing the anesthetic solution passively into the chamber is insufficient because the solution will not diffuse throughout the pulp.

Adding a complementary color to a restoration would: A. Increase the hue B. Increase the value C. Decrease the value D. Increase the chroma

C. Decrease the value

Test with 2 continuous variables what would you use? A. Chi squared B. Linear regression C. Pearson correlation

C. Pearson correlation

An upper anterior inclined plane provides A. Intramaxillary anchorage B. Absolute anchorage C. Reinforced anchorage D. Simple anchorage

C. Reinforced anchorage

All are Beta Blocker except: A. Propranolol B. Atenolol C. Tramadol D. Metoprolol

C. Tramadol Because the Tramadol Hydrochloride is a type of strong opioid centrally acting analgesic and it's a synthetic 4-phenyl-piperidine analogue of codeine, and it is a central analgesic with a low affinity for opioid receptors, in fact the Tramadol acts on the mu opioid receptor, blocking the neuron from communicating pain to the brain and also belongs to the class of drugs known as opiate agonists, however some people believe it's a Non Steroid Anti Inflammatory Drugs (NSAIDs) but it is not.

calcium channel Blocker is _______ ? A. Propanol B. Atenolol C. Verampimil D. Metoprolol

C. Verapamil Because the Verapamil is a Non-dihydropyridine calcium channel blocker antagonist, or a cardioselective and phenylalkylamine Calcium Channel Blockers (CCB) which have negative inotropic effects, as a result it can cause flushing, headache, excessive hypotension, edema and reflex tachycardia, and a baroreceptor reflex activation of sympathetic nerves and lack of direct negative cardiac effects, and as a consequence it can make dihydropyridines a less desirable choice for stable angina than diltiazem.

The most important anticoagulant effect of heparin is to interfere with the conversion of : A. PTA to PTC. B. PTC to Factor VIII. C. fibrinogen to fibrin. D. prothrombin to thrombin. E. proaccelerin to accelerin.

C. fibrinogen to fibrin-It acts on factor X

Which one of the following is an established clinical use of morphine? A. Management of generalized anxiety disorder B. Relief of pain associated with biliary colic C. Pulmonary congestion D. Treatment of cough E. Suppression of the ethanol withdrawal syndrome

C..pulmonary congestion...morphine is used in pulmonary congestion ,in part because of its sedative (calming) and analgesic effects and also because of its vasodilating actions,which result in favourable hemodyanamics in terms of cardiac and pulmonary function.

coloboma of iris, nick in ear lobe is associated with which syndrome?

CHARGE syndrome Treacher Collins syndrome, cat eye syndrome patau syndrome

difference between canker and chanker-

Canker sores are caused by damage to the mouth, foods, or an underlying disease, while cold sores are caused by the herpes virus. Eventually, cold sores become crusted over while canker sores simply go away.

Consequences of tooth extraction in a patient with adrenal crisis?

Cant bear stress of extn and leads to hypotension,hyponatrimia, loss of consciousnes

Carbamazepine for epilepsy and neuro disease?

Carbamazepine is an epilepsy medication used to prevent seizures. It may also be used for bipolar disorder and trigeminal neuralgia.

A 28 year old patient who has severe anterior crowding, an Angle Class II malocclusion, and a high DMFS score wants orthodontic treatment. Which of the following is the most appropriate initial step in management of this case? Fluoride rinse prescription. Diagnostic wax-up. Cephalometric analysis. Caries risk assessment.

Caries risk assessment.

Effect of histamine and that it's derived from histidine?

Causes bronchoconstriction, bronchial smooth muscle contraction, vasodilation,

Why don't you use acidic fluoride with glass ionomer ?

Causes removal of glass particularly glacé and soluBility

Digitalis toxicity 1.inc PR interval 2.AV conduction block 3.yellow green vision 4.nausea and vominting 5.extrasystole

Characteristic EKG changes include bradycardia (the most frequent vital sign abnormality in toxicity), a prolonged PR interval. An accelerated junctional rhythm or bidirectional ventricular tachycardia suggests digoxin toxicity until proven otherwise.

10 years old child present with bilateral swelling of submandibular area, what could be disease 1. Fibrous dysplasia. 2. Cherubsim. 3. Plermorphic adenoma

Cherubsim.

most common clasp in rpd ?

Circlet clasp

cleidocranialb dysplasia hypertelorism?

Clavicle deformation too. multipl supernumerary teeth retained primary teeth narrow facial frontanel failed to close

Pt has sever pain in lower 3rd molar and he has Pent...Filed syndrome according to infection control guide how to sterile instruments: - Leave it dry until cleaning then disinfect - Clean it & wash it then put it in a bag 10 mins in sterilization - Chemical disinfect after each pt - Sterile it at the begin of all pt

Clean it & wash it then put it in a bag 10 mins in sterilization

When do the following occur in embryonic life- 1. Cleft lip 2. Cleft palate

Cleft palate 8-10 weeks Cleft lip 6-7 weeks

Normal value of PTT PT Bleeding time

Coagulation Parameters Prothrombin time PT 11-14 seconds Partial thromboplastin time PTT 25-35 seconds International normalized ratio INR 0.8 to 1.2

how to test tooth with non metal crowns for vitality?

Cold testing can be accomplished by individually isolating teeth with a rubber dam. This technique for cold testing is especially useful for patients presenting with porcelain jacket crowns or porcelain-fused-to-metal crowns where there is no natural tooth surface (or much metal) accessible. Frozen carbon dioxide (CO2), also known as dry ice or carbon dioxide snow, has been found to be reliable in eliciting a positive response if vital pulp tissue is present in the tooth.

Advantage of resin over porcelain is

Composite resin veneers are accomplished in one visit. An adequate amount of tooth structure is removed to allow for placement of composite resin in the desired shape without added tooth bulk. Bonding agent is applied. Composite resin is then added, light cured, then finished and polished. Porcelain veneers are more expensive than composite veneers. The placement of veneers requires more time, expertise and resources in order to fabricate and bond and therefore cost more. Porcelain veneers cannot be repaired. If they break porcelain veneers must be replaced.

What components makes a PMF green in the cervical 1/3 ?

Copper in cervical and silver margins and remaining places

Which of the following muscles is most likely to be affected by a depressed fracture of the zygomatic arch? A. Lateral pterygoid. B. Temporalis. C. Masseter. D. Medial pterygoid

Correct Answer C

Which of the following creates the phenomena of dentinal plugs? A. Oxalic acid B. Sclerotic dentin C. Adhesive D. Primer (form micro/macrotags) E. Debris from the smear layer

Correct Answer is E Dentinal or smear plugs are composed of the debris from the smear layer. The smear layer is documented to be about a 0.5-2mm thick layer of debris with a mainly granular substructure that entirely covers the dentin. The surface of the smear layer appears quite irregular. The orifices of dentinal tubules are obstructed by debris tags known as smear plugs or dentinal plugs. These dentinal plugs extend into the tubules to a depth of 1-10 micrometers. The smear layer is reported to reduce dentinal permeability by about 86% due to dentinal plugging. Dentinal plugs are removed by acid etching the tooth surface with phosphoric acid.

Length of dental floss used on retainer during rubber dam placement ?

Correct ans is 12 inches or 30.5 cm

An eye that has chronic exposure to visible light curing units will sustain the most damage to which of the following structures? Retina Lens Iris Cornea Ciliary muscle

Correct answer is A ! Continuous exposure to visible light used for curing the composite can cause serious damage to the retina. Care should be taken not to look directly at the visible light cure source. Use a light-curing machine with a protective barrier covering the light source. Prolonged visible light exposure can lead to photoreceptor cell damage.

Autoimmune conditions with higher incidence of periodontal diseases

Crest syndrome,crohns disease, rheumatoid & lupus erythematous,multiple sclerosis

maxillary complete denture, accurate adaptation of the border of maxillary facial flangeaffects which of the following most: Speech, Support, Stability, Esthetics

D If there is an undercut area at maxillary facial flange we can leave that flange. For retension and stability we have to make the base and other flanges as broad as much have we can.

What is the best treatment plan to be employed to manage an iatrogenic perforation involving the furcation of the tooth? ---Repair the perforation and continue the root canal treatment at a subsequent appointment. ----Perform no further treatment at this time and schedule the patient for an extraction. ----Repair the perforation at a subsequent appointment only if symptoms occur. ----Repair the perforation immediately with mineral trioxide aggregate (MTA/Portland Cement), inform the patient, and reassess the tooth at the recall appointment. ----Continue the root canal treatment as normally planned because tertiary dentin will fill in.

D is the right abswer Root perforations during endodontic treatments should be addressed immediately through the use of MTA to help stimulate healing of periradicular tissues and to promote cementum deposition. Mineral Trioxide Aggregate (MTA) is regarded to be the most ideal perforation repair material because of its ability to create a seal when moisture is present. MTA is biocompatible and is capable of inducing periradicular tissue repair and cementum deposition.

Which of the following is always present in class II division two malocclusion A)cross bite B)open bite C)close bite D)deep bite

D)deep bite

What is the major difference between a class V cavity preparation for amalgam and one for composite resin by acid-etch tech.? a. depth b. convenience form c. position of retention points d. angulation of the enamel cavosurface margins

D- amalgam- 90 degree "butt"joint at cavosurface composite- bevel-obtuse angle. helps in increasing surface area for bonding and improves aesthetics

When u give sedative inhalation for Pt to prevent hypoxia u give: A- 95%oxygen and 5% nitrous oxide . B- 90% and 10%. C-85%and 15%. D-100% oxygen and zero nitrous oxide.

D-100% oxygen and zero nitrous oxide.

Daily cleaning of the root surface by the patient has been shown to A.cause root resorption B.cause rot sensitivity c.stimulate epi attachment D..allow mineralization of root aurface

D-gingival recession becuase the root is not exposed in regular condition so to reduce root sensitivity and to allow for remineralization, the pt has to clean the root surface daily and remove the plaque.

Methode of subgingival cleaning plaque debris ?? A) floss B) tooth brush C) electric brush D) oral irrigator

D. Oral irrigator,,,,,, floss only remove from proximal surface and accessible subgungival area... Irrigator cab remove plaque and debris from sybgingival area including deep pocket as far as 7mm and also furcation area.

: Which of the following is frequently accompanied by melanin pigmentation (cafe-au- lait spots)? A. Osteomalacia B. Hyperparathyroidism C. Osteogenesis imperfecta D. Polyostotic fibrous dysplasia

D. Polyostotic fibrous dysplasia

What happens to the circumference in the mandibular arch? 6-18 years

Decrease of 4 mm

Rapid mixing of Zinc Phosphate cement will increases compressive strength or decreases it?

Decrease( in small increments, over a large area, to dissipate heat and acid)

Decreased caries rate is seen in

Dentinal dysplasia type 1 Downs syndrome Cystic fibrosis

pt with steroid in dental office?

Depending on the consciousness level of the patient, proper management of an emergency situation of acute adrenal insufficiency includes the following steps: Conscious patient Terminate dental treatment as soon as the initial signs of adrenal insufficiency, such as mental confusion, abdominal pain, nausea and/or vomiting, become evident in a person on glucocorticoisteroid therapy. If signs of hypotension are evident, place the patient in a supine position with legs slightly elevated. However, in the absence of such signs, placing the patient in their preferred comfortable position is recommended. Provide basic life support and maintain proper circulation, airway and breathing. Provide definitive care that includes monitoring vital signs, activating emergency medical assistance, oxygen administration, if needed (5-10ml/min), and administration of glucocorticosteroids. In the case of a patient with chronic adrenal insufficiency, 50-100mg of hydrocortisone should be administered intravenously; re-administration should follow every 6-8 hrs. If the patient has no known history of adrenal insufficiency or gluococorticosteroid administration, but exhibits the diagnostic signs and symptoms of adrenal insufficiency, 4 mg dexamethasone phosphate (IV) should be administered immediately rather than waiting for the lab results (ACTH stimulation test). The drug should be re-administrated every 6-8 hours, as needed. Additional management may include the administration of 1L of normal saline, and management of hypoglycemia by administration of 50-100 mL of 50% dextrose solution. Unconscious patient Determine the unconsciousness and place the patient in a supine position with legs elevated. Provide basic life support, maintain circulation, airway and breathing. Usually the pulse is rapid, weak and thready, but in rare circumstances the pulse might be absent, which warrants immediate initiation of external chest compressions. Definitive care includes the administration of oxygen and emergency medicines. Emergency medical assistance should be activated at this stage. If the patient's medical history indicates the possibility of adrenal insufficiency, 100 mg hydrocortisone should be administered via IV or IM route. An additional 100mg of hydrocortisone should be administered by IV infusion (over 2 hrs.) or IM route. Further, management may include administration of 1ml of normal saline and 50% dextrose solution. If the cause of unconsciousness cannot be established, no drug administration is indicated, and basic life support (BLS) steps should be continued until the arrival of emergency medical assistance.

diameter of implant?

Depends on the ridge.. 3.25 to 6 mm. Mini implants around 3.25

Which of the following describes the putative explanation for why dental plaque adheres to tooth surfaces? Low surface tension of plaque Levans are insoluble and sticky Dextrans are insoluble and sticky Bacteria secrete a lioprotein cement

Dextrans are insoluble and sticky

Endo surgery contraindicated when

Diabetes type II Hypertension

why is diazepam contraindicated in asthma ?

Diazepam should be avoided in Acute asthma as it can cause cns and respiratory depression

how benzodiazepine anxiolytic?

Diazepam, one of the most common BZDs used for anxiety,2 is available in intramuscular, intravenous, oral, and rectal gel forms. Diazepam interacts with equal affinity on all BZD-sensitive receptors in the central nervous system. Anxiolytic effects are seen at low doses because of diazepam's interaction with α2-containing receptors in the limbic system.

Differences between MRI and CT?

Differences between normal and abnormal tissue is often clearer on an MRI image than a CT.

Digoxin and Digitalis

Digoxin is the only available preparation of digitalis in the United States. Digitalis is a plant-derived cardiac glycoside commonly used in the treatment of congestive heart failure (CHF), atrial fibrillation, and reentrant supraventricular tachycardia

what is water irrigation used for?

Dilute the bacterial toxins

In which of the following, characteristic Carpet tack extensions are seen on skin? Syphilis Discoid lupus Erythematosus Basal cell carcinoma Lichen planus

Discoid lupus Erythematosus

Which of following is the "Anti platelet drug ______ ? A. Disprin B. Colchicine C. Thalidomide D. Allopurinol

Disprin tablets contain the active ingredient acetylsalicylic acid,otherwise known as aspirin.Aspirin belongs to a group of medicines called NSAID's It works by blocking the actions of an enzyme in the body called cyclo-oxygenase(COX).

What is the appropriate treatment for intruded primary tooth

Do nothing n let the tooth reerupt spontanously bcz has potential or exrat immediatly if it aproximat or trauma to permanant tooth bud

dental managment of pt with liver cirrosis?

Drugs use in patients with liver diseases: table1. Drug Use in patients with liver disorder Comments Acetaminophen Yes with modifications To be given in divided doses < 4.0 gm per day for up to 2 weeks without adverse hepatic effects. Amide local anesthetics (lido & mepivacaine) Yes with caution Lidocaine has a large and rapid volume distribution. Only 6% of injected volume is present in blood and minimal elevation of peak blood concentration occurs after single dose which is clinically insignificant. Aspirin and NSAIDs Avoid A decreased serum protein levels cause increased toxicity of drug as they are protein bound causing more free drug availability Benzodiazepines Yes with modifications Decreased metabolism causes increased sedation also receptors in brain are more sensitive. Dosage to be reduced with less frequent intervals. Use drug without active metabolite ( e.g., alprazolam, lorazepam) rater than with active metabolite( e.g., diazepam) Beta lactam antibiotics (penicillin, amoxicillin) Yes Elimination of drug is mainly by renal filtration and tubular excretion. Penicillin, ampicillin, amoxicillin, cepalexin, and cefazolin are well tolerated. Clindamycin Avoid It causes progressive liver disease causing more damage. Azithromycin Avoid It is principally eliminated by liver, so to be

epilepsy in pregnancy what medicine?

During pregnancy, a daily multivitamin containing 0.4 mg folate, as well as an additional 1- to 4-mg folate supplement, is recommended for all women of childbearing age.

Which of the following compounds is responsible for the photoinitiation step of light-cured sealants? Bisphenol A Benzoyl peroxide Fumed silica Fluoride Camphorquionone

E is the right answer ! Camphorquinone is used as a photoinitiator and visible light or an argon laser is used as the activator in light-cure sealants. One of the most important requirements of a pits and fissure sealant is that it should prevent leakage at its periphery and give an adequate working time. It has been shown that the adherence of pits and fissure sealants and working time is significantly better in light-cured sealants as compared to chemical cure sealants. Benzoyl peroxide is incorporated as a component in auto-curing/self-curing sealants.

A patient has an opening interincisive 30 mm before feeling pain. His maximum aperture is 44mm with pain. What is the diagnosis likely? A. TMJ Internal Derangement with reduction. B. TMJ Internal Derangement without reduction. C. Trismus muscles of the chewing. D. subluxation ATM. E. myofascial pain.

E- Axis I: Diagnosis temporo mandibular disorders (somatic axis) Group I: Muscle Disorders -Ia: myofascial pain with normal mouth opening of 45 to 50 mm between the central incisors antagonists. -Ib: myofascial pain with limited mouth opening less than 40 mm. Sometimes very limited to about two to three millimeters. -Ic: myofascial pain with exaggerated mouth opening more than 60 mm. This form is the atypical swallowing and salivary Pulsive (dental open bite), accompanied by a significant laxity of ligaments ATM. Group II: Moving the articular disc -IIa: Displacement of the disc with reduction =IIb: Displacement of the disc without reduction with limited opening -IIc: Moving the drive without reduction without limited opening Group III: arthralgia, arthritis, osteoarthritis. These are exceptional. Axis II: pain-related disability and psychosocial status (psychosocial axis) Chronic Pain Grade Rating Axis II scales

What should be added in Composite to increase it color stability? A-BIS-GMA B-UDMA C-PMMA D-HEMA E-TEGDMA

E-TEGDMA

Which of the following medical conditions would require antibiotic prophylaxis for receiving nonsurgical root canal treatment? A. Cardiac pacemaker. B. Diabetes mellitis. C. Cerebrospinal shunts. D. History of cerebrovascular accident. E. None of the above.

E. None of the above.

Most common bacteria found in root canals --- gram positive?t/f

E. faecalis are gram positive cocci and facultative anaerobes.

best test for necrotic pulp?? cold test or EPT

EPT

Test performed to differentiate endo vs. perio lesions?

EPT+ vital tooth... Perio EPT- non vital... Endo

What is the difference btw gingivitis and establish gingivitis

Early gingivitis- pmns Intermidiate- lymphocytes and macrophages Established- plasma cell. There are 3 stages of gingivitis initial -2 to 4 days and has pmn,early stage-4 to 7 days has lymphocyte and established 14 to 21 days has plasma cell

treatment of ectopic eruption?

Ectopic eruption of maxillary first molar : Most likely needs ortho 50% self resolves 66% self correct Its depend on location of ectopic tooth 1st is slight trimming of tooth 2 is disking and last os extn

edrophonium used for what

Edrophonium ( Tensilon test) is used to differentiate myasthenia gravis from cholinergic crisis and Lambert-Eaton myasthenic syndrome.and also used to prevent muscle contraction during surgical procedure.

pH that enamel starts to demineralize

Enamel demineralization occurs when pH levels fall to 5.7.

Eosinophilic bone lesion look similar to Langerhans Myeloma

Eosinophilic Granuloma and langerhans cell histiocytosis

Which of the following is not used for sterilization of hand piece a. Dry heat b. Autoclave c. Ethylene oxide d. Chemical vapors

Ethylene oxide

A lesion of non endodontic origin remains at the apex of suspected tooth regardless of xray cone angulation.t/f

F - normal structrures - changes place...endo origin - remains

organism implicated on causing severe spreading abscesses include a. Fusobacterium b Campylobacter c. Enterococci d. Bacteroides

F. Nucleatum P. Intermedia P. Gingivalis P. Micros and T. Forsythia predominate abscess so A

Mini implant

FDA approved term for implants 1.8 to 3.0mm in diameter are termed as "narrow diameter implants" and that 3.0 to 3.5 as "Mini implants" and 3.5 to 6.0 mm as regular diameter implants.

Caries detection criteria

FLOSS - mnemonic Flaking of enamel Loss of translucency Opacity Sticking of explorer Softening of tooth structure

Factor present both in serum and plasma: Prothrombin Factor V Factor VII Factor VIII

Factor VII

Q. Which will aid in the cognitive appraisal of a threat? 1.Adaptability, preventability, inevitability, and constancy 2.Controllability, familiarity, predictability, and imminence 3.Interference, adaptability, longevity, and reactance 4.Validity, reliability, adaptability, and predictability 5.Accountability, reliability, validity, and familiarity

Factors affecting coping with a threat/stress: Controllability: to control the stress Familiarity: to be familiar with a threat Predictability: to predict the expected threat Imminence: threat about to happen

T/F *Ideal Pontic is ovate *most esthetic is modified ridge lap

False Ovate more esthetic used in some cases not always Modified ridge lab good esthetic and hygienic ( ideal)

Fear of choking because?

Fear of choking is associated with anxiety, depression, panic attacks, hypochondriasis, and weight loss

according to itched enamel will reach complete remineralization: - Few hours - Few weeks - Few hours - Few months

Few months

atm card?

Fidelity health savings account (HSA) to be eligible for a card Fidelity HSA customers can apply for a debit card to pay for qualified medical expenses

Finasteride -PROSCAR Used for BPH?

Finasteride belongs to a group of medications known as 5-alpha-reductase inhibitors. Finasteride is used to treat benign prostatic hyperplasia (BPH), also known as enlarged prostate. Finasteride can be used alone or in combination with another medication called doxazosin

The main objective of scaling and root planing is to restore gingival health by removing etiologic factors For scaling and root planing the best evidence of success of imstrumentation is radiographically seeing the calcifications removed .

First one is true second one is false

Best way to detect furcation involved in trifurcation tooth is from

First probing important factor then radiographs . For trifurcation start from mesiolingual towards buccal .

Patient with class II division II, the lateral incisor is missing. You want to make a fixed bridge which of the following is suitable? A. Rocket bridge using central incisor as abutment. B. Cantilever using central incisor. C. Fixed bridge using the central incisor and bicuspid.

Fixed bridge using the central incisor and bicuspid.

Correction of an inadequate zone of attached gingiva on several adjacent teeth is best accomplished with a/an? a. apically repositioned flap. b. laterally positioned sliding flap. c. double-papilla pedicle graft. d. coronally positioned flap. e. free gingival graft.

Flap techniques can only correct a "small" muco-gingival defect, when the area is large (SEVERAL), the free graft is the most appropriate indication.

Flumanezil is antagonist of benzodiazepines? T/F

Flumazenil is a recently discovered pharmacologic antagonist of the CNS effects of benzodiazepines. It acts by binding CNS benzodiazepine receptors and competitively blocking benzodiazepine activation of inhibitory GABAergic synapses.

What is bilateral on buccal mucosa 1) linea alba 2) frictional keratosis

Focal (frictional) keratosis

NUG when has systemic manifestation which antibiotic : 1) Penicillin 2) Metronidazole 3) Tertracycline 4) None

For any signs of systemic involvement, the recommended antibiotics are: Amoxicillin, 250 mg 3 x daily for 7 days and/or Metronidazole, 250 mg 3 x daily for 7 days

Silicate restoration is

For extensive caries involvig root

Radiation injury is Free radical from indirect Free radical from direct

Free radical from indirect

Freeway space is normally about____ and ____mm.

Freeway space is 2 to 6 mm

Which of the following is NOT a sign of occlusal trauma? Tooth migration. Widened periodontal ligament. Fremitus. Gingival recession.

Gingival recession.

Beta blockers and Calcium channel blockers ,,, Glucagon is Antidote for both of them ... T/F ?

Glucagon has become an accepted antidote to beta-blocker poisoning because it stimulates cAMP synthesis independent of the beta-adrenergic receptor.Glucagon has shown positive inotropic and chronotropic effects despite beta-receptor blockade in numerous animal models and in humans.Several animal studies and case reports have also demonstrated a benefit in CCB toxicity, though many treatment failures have been noted as well.Side effects include dose-dependent nausea and vomiting, hyperglycemia, hypokalemia, and allergic reactions.

Why we use green stick in taking imp for complete denture

Good properties in recording peripheral seal area (easy manipulation, accurate details while muscle trimming)

The beat grasp to use when holding an instrument to be sharpened is

Grasp the instrument -Palm gasp -Stabilize hand on countertop -Stabilize instrument handle with your thumb

gunther's disease

Gunther disease, also known as congenital erythropoietic porphyria (CEP), uroporphyrinogen III synthase deficiency and UROS deficiency,

Histamine Features

H - Histamine production I - Inflammation S - Strong vasodilator T - Therapeautic value ( none ) A - Allergy M - Mast Cells I - Inflammation N - Neurotransmiter / Narrow airwarys E - IgE

medication to treat peptic ulcer

H2 blockers include cimetidine (Tagamet), ranitidine (Zantac),

Which one can human eye see, hue vs value, vs chroma?

Half- closed eyes can increase the sensitivity to better choose " VALUE"

What is untrue about EPT? A/It is more reliable than cold testing for necrotic teeth B/It gives relative health status of pulp C/ Tells if there are vital nerve fibers

Health status of pulp depends on vascularity and not vitality , ept acknowledges about vitality , so b is untrue for ept .

Postherpetic neuralgia: Herpes Zoster HSV 1 HSV 2

Herpes Zoster

Boggy red gingiva?

Herpes simplex virus stomatitis is common in childhood but is increasingly seen in adults. There is a diffuse, purple, "boggy" gingivitis, especially anteriorly, with multiple vesicles scattered across the oral mucosa and gingiva and leukemia

diseases which show Tzanck cells

Herpes simplex. Varicella and herpes zoster. Pemphigus vulgaris. Cytomegalovirus.

How is heart rate affected by hyperventilation?

Hyperventilation is by definition a low level of carbon dioxide, CO2, in the blood due to excessively rapid or deep breathing. When carbon dioxide levels fall, this results in anxiety which in turn activates the sympathetic nervous system which increases breathing, heart rate and blood pressure

First cause for problems during iv sedation?

Hypoxia Burning sensation Embolism

Before filling a class V abrasion cavity with GIC you should A. Clean with pumice, rubber cup, water and weak acid B. Dry the cavity thoroughly before doing anything C. Acid etch cavity then dry thoroughly

IN CLASS V we restore it with gic as its a moisture bearing material . we clean with pumice ,rubber cup and don't dry it much ,as gic is moisture loving . for 10% polyacrylic acid conditioning is done for 20 sec 20% polyacrylic acid conditioning is done for 10 sec as we don't etch gic because there is a chemical bond rather than micromechanical bond

hypertrophy of mandibular condyle may lead to the following scenarios except? anterior open bite anterior cross bite unilateral class 3 malocclusion ipisilateral posterior open bite

If the hypertrophy is in a vertical direction it will cause anterior open bite

Why we cannot give bacteriostatic together with bacteriocidal antibiotics? or Penicillin is ineffective when you give tetracycline why?

If you give Tetracycline, it will eventually stop bacterial replication. Without replication, there will be no synthesis of new peptidoglycans. Without new peptidoglycans, there will be no cross linking. Without cross linking, Penicillin couldn't elicit its response. So, we can see there is an antagonism happening between Tetracycline and Penicillin. most of them believed that Tetracycline reduces Penicillin action into half. Therefore, Tetracycline and Penicillin are said to be antagonizing with each other.

alzehimer drug and managment in dental office

Improved oral hygiene would benefit all patients with dementia .Short appointments for patients with AD can beless stressful. Treatment plans shouldbe designed with minimal changes to the oral cavity andshould not involve complete rehabilitation. During the treat-ment procedure, it may be necessary to administer oralsedatives or anxiolytics, such as short-acting benzodi-azepines, before dental treatment to manage behavior.

How do you treat actinic cheilitis?

Imquimide 5-fluorouracil Cryotherapy. Electrocautery. Vermilionectomy (surgical removal of the external lip)

in a healthy mouth alveolar process most thinnest around a.facial of mandibular centrals b.facial of maxiallry centrals

In a healthy mouth, the alveolar process is thinnest around the facial of the mandibular central incisor.

when patient have to pay from his pocket?? deducible copay third pary payment

In an insurance policy, the deductible is the amount of expenses that must be paid out of pocket before an insurer will pay any expenses. Co-pay is a type of insurance policy where the insured pays a specified amount of out-of-pocket expenses for health-care services such as doctor visits and prescriptions drugs at the time the service is rendered, with the insurer paying the remaining costs. So A and B . Third-party health insurance is defined as insurance coverage in which a third party, namely the insurance company, pays the actual provider of healthcare services for services rendered to the employee.

The study of oral health on pregnant woman in many health centers? a) cohort study b) cross sectional c) case control d) observational study

In cohort study, participants are disease free at baseline. They followed through time to see if disease develops

Law of hazard communication law, what is it and what organization created?

In order to ensure chemical safety in the workplace, information about the identities and hazards of the chemicals must be available and understandable to workers. OSHA's Hazard Communication Standard (HCS)

In young child with avulsed and replanted permanent teeth that have open apices the blood supply is usually regained 7days 10days 20days 30days

In young children, in cases of avulsed and replanted teeth with open apices, the blood supply is regained within the first 20 days after replantation

What are the reasons for beveling in composite restoration?

Increase in surface area because stronger enamel to resin bond Ends of enamel rods are etched. esthetic blending due to cavosurface bevel.

What happens to the circumference in the max. arch ?

Increase of 1 mm

Restoratives options if remaining dentin thickness is 1mm?

Indirect pulp capping " CaOH"

Interpersonal distanc--- Hall described the interpersonal distances of man (the relative distances between people) in four zones: intimate space, personal space, social space, and public space.

Intimate distance for embracing, touching or whispering Close phase - less than 6 inches (15 cm) Far phase - 6 to 18 inches (15 to 46 cm) Personal distance for interactions among good friends or family Close phase - 1.5 to 2.5 feet (46 to 76 cm) Far phase - 2.5 to 4 feet (76 to 122 cm) Social distance for interactions among acquaintances Close phase - 4 to 7 feet (1.2 to 2.1 m) Far phase - 7 to 12 feet (2.1 to 3.7 m) Public distance used for public speaking Close phase - 12 to 25 feet (3.7 to 7.6 m) Far phase - 25 feet (7.6 m) or more.

Metabolite of ingestend isopropyl alcohol is

Isopropyl causes the same level of "drunkenness" at a serum level approximately half that of ethanol. It is rapidly absorbed, and approximately 80% is metabolized to acetone by alcohol dehydrogenase (ADH) following first order kinetics. The remaining 20% is excreted unchanged in the urine.

What happens to a drug after conjugation- more ionic, more hydrophilic, more active

It becomes more inactive, more polar, larger, and more easily excreted in the bile or urine. so b

Metronidazole MAO

It inhibits nucleic acid synthesis by disrupting the DNA of microbial cells

chelation therapy for mercury poisoning

It involves intravenous injections of a chelating agent, EDTA (ethylene diamine tetra-acetic acid), a synthetic amino acid.

Stochastic effects are associated with long term , low level exposure of radiation yes or no

It is True. Radiation exposure at high doses act like deterministic, that means after a certain threshold is reached, it will cause cancer or other disastrous effect. While you can continue taking low level exposure for life and you may or you may not get effects. This random probability (can occur or may not occur too whole life) is called stochastic effect.

dental phobia is seen in what percentage of population a 5% b 25% c 50% d 75%

It is estimated that as many as 75% of US adults experience some degree of dental fear, from mild to severe.[2][3][4] Approximately 5 to 10 percent of U.S. adults are considered to experience dental phobia; that is, they are so fearful of receiving dental treatment that they avoid dental care at all costs

Which of the following conditions puts patients, especially the elderly, at an increased risk for caries? Gingival recession Erosion Bruxism Attrition

Its A Gingival recession results in exposure of the root surfaces. Root surfaces are less resistant to caries attack as compared to the enamel. The passive movement of gingival tissue occurs apically with the aging process.

Light-cured composite resin is most affected by which of the following variables? A. Thickness of increment B. Shade of material C. Opacity of material D. Length of exposure

Its A Polymerization of light-cured composites is dependent upon the penetration of light through the composite resin. As the material thickness increases, the penetration of the light decreases, which may leave some composite unpolymerized, resulting in the failure of restoration. Placing composite resin in small increments is advised because it allows for complete penetration of curing light and complete polymerization. Incremental placement of composite resin material also aids in minimizing the polymerization shrinkage which: •Increases bond strength •Decrease cuspal fractures •Decreases marginal leakage

For a Class V restoration, where should the rubber dam be punched? A. 1 mm facial to the designated tooth B. 3 mm facial to the designated tooth C. 3 mm lingual to the designated tooth D. 1 mm lingual to the designated tooth

Its A The rubber dam should be punched 1 mm facial to the designated tooth for Class V restorations, enabling the rubber dam to completely cover the area to be isolated and making sub-gingival placement easier if needed. Class V restorations require planned measures for maintenance of isolation as it is extremely difficult to place rubber dam in these areas. The location of Class V restorations makes it very difficult to prevent moisture contamination.

Each of the following drugs can produce xerostomia as a side effect EXCEPT one. Which one is the EXCEPTION? Pilocarpine Benzodiazepines Antihistamines Antidepressants Anticholinergics

Its A Xerostomia (dry mouth) is characterized by a decreased production of saliva or salivary flow in the oral cavity. Pilocarpine is used to treat xerostomia in patients. The subjective feeling of dryness in xerostomia can cause: Dental caries Oral candidiasis Oral dysesthesia Dry mucosa Dysphagia Fissured tongue associated with atrophy of filiform papillae Difficulty in wearing dentures

On which of the following portions of primary teeth is staining most evident? Cervical Incisal/Occlusal Facial/Buccal Lingual Distal Mesial

Its A , and here is why - The cervical surface of primary teeth is where staining is usually evident because: •Cervical bulges can trap debris underneath. •Cervical portions are more constricted in primary teeth thus making plaque, caries, and stain formation easier. •Children are incapable of brushing the cervical areas properly due to limited dexterity.

Which part of the tooth is most commonly misdiagnosed as caries in a radiograph? Cemento-enamel junction (CEJ) Pulp horn Marginal ridge Cingulum CuspA

Its A - An apparent radiolucency is often found just below the cemento-enamel junction on the root surface called "cervical burn-out" and is often misdiagnosed as root caries in many cases. Cervical burn-out is evident because of: A gap between the enamel and cementum or bone covering the root anteriorly A concave root formation posteriorly

Which of the following is the best treatment plan to control rampant caries (many proximal caries that undermine enamel)? Restore all teeth with crowns. Remove caries, place glass ionomer restortations, then complete the treatment plan after the vitality of the teeth has been established. Restore all teeth with amalgam as quickly as possible. Prescribe high fluoride toothpaste, give oral hygiene instruction, and wait until effectiveness has been appraised.

Its B Rampant caries control protocols suggest that caries should be removed and glass ionomer cement restorations should be placed until the true pulp status can be assessed. Removing the caries allows the clinician to see if caries have extended to the pulp cavity. The placing of glass ionomer restorations allows the affected dentin to be remineralized.

Which is the optimal range of drinking water fluoridation as defined by the United States Centers for Disease Control (CDC) guidelines? 0.1-0.6ppm 0.7-1.2ppm 0.3-1.8ppm 1.9-2.4ppm

Its B ..... United States Centers for Disease Control (CDC) guidelines indicate that the optimal range of fluoride in drinking water should be 0.7 to 1.2ppm. Water fluoridation is the most economical and the most effective technique in delivering fluoride to the people of the community. The amount of fluoride allowed in water varies widely from place to place depending on the weather conditions and its relation to the amount of water intake of the people in the community.

Each of the following is an indication for the placement of a base EXCEPT one. Which is the EXCEPTION? Replacement of missing tooth structure All answers apply Prevention or decrease of marginal leakage Thermal protection for the pulp Prevention of other dental materials from irritating the pulp

Its C Bases replace missing tooth structure and support the final restoration. Bases insulate the pulp from drastic temperature changes experienced by the restoration. Bases provide structure to condense against when placing amalgam. Bases protect the pulp from agents which may irritate it.

Fluoride ions incorporated at which stage of tooth development produces a tooth structure that is more resistant to caries? Proliferation Histodifferentiation Calcification Apposition

Its C Fluoride is incorporated into enamel during the calcification/mineralization stage of tooth development. Fluoride ions can replace the hydroxyl ions during the calcification stage of tooth development, which results in a structure that is more compact and resistant to caries.

The earliest manifestation of a carious lesion is which of the following? Rough surface texture Cavitation of enamel Change in enamel opacity Radiolucency Sensitivity

Its C - Change in the opacity of enamel becomes more evident along the areas of active caries progression. The caries process demonstrates areas of demineralization that are suggested by a chalky white appearance in the enamel. Change in enamel opacity will eventually lead to further tooth surface destruction and formation of tooth decay/cavitation if not addressed immediately.

Which of the following factors of caries susceptibility is modified most by fluoride therapy and occlusal sealants? Time Substrate Host Microflora Submit

Its C - Hydoxyapatite crystals are the building blocks of tooth enamel. Hydrogen ions (H+) from acids attack the hydroxyl ions of the hydroxyapatite crystals, resulting in the loss of hydroxyl ions (OH-). Fluoride replaces the hydroxyl ions with fluoride ions (F-), resulting in the formation of fluorapatite instead of hydroxyapatite. Fluorapatite is more compact and resistant to acid attack, making the host's enamel more resistant to caries. Occlusal sealants also aid the host in resisting caries by filling deep pits and fissures and making these areas more cleansable.

If a permanent maxillary central incisor was avulsed and rigidly splinted, which of the following is the most likely outcome? Root fracture Internal resorption Normal healing response Replacement resorption Calcific metamorphosis

Its D The avulsed tooth should be splinted with a flexible splint to avoid extensive healing complications such as replacement resorption. Avulsed teeth in the arch should still have some degree of freedom for movement to prevent dentoalveolar ankylosis from splinting. Freedom for slight movement lessens the pressure on bone and allows better healing along the periapical aspect of bone surrounding the traumatized tooth. Splinting should also last for about two weeks to prevent replacement resorption. If splinting lasts too long the portion of the tooth that was resorbed is replaced by the healing bone, causing ankylosis of the tooth.

Which describes the correct order for the progression of carious lesions? i. Subsurface enamel lesion ii. Cavitation iii. Infection of demineralized dentin iv. Dentin demineralization v. Dentin-matrix dissolution. vi. Pulpal necrosis A ----------------i, ii, iv, iii, v, vi B ----------------i, ii, v, iv, iii, vi C ----------------i, iii, vi, iv, v, ii D ----------------i, iv, ii, iii, v, vi

Its D The correct order of progression (sequelae) of a carious lesion is: Subsurface enamel lesion Dentin demineralization Cavitation Infection of de-mineralized dentin Dentin-matrix dissolution Pulpal necrosis

Which of the folowing is the best way to treat superficial decalcification/staining seen in deep fissures of molars? Daily irrigation with pressurized water Oral prophylaxis at three-month intervals Full removal of the affected area with good retention and resistance form and restoration Enameloplasty limited to the superficial depth of the decalcified enamel

Its D ! Localized and decalcified superficial stains within the limits of the enamel layer can be addressed by enameloplasty procedures. By performing enameloplasty, unnecessary tooth preparation required for a full restoration is prevented and the decalcified buccal groove is made more cleansable. Enameloplasty of the affected tooth structure removes superficial stains and is a conservative treatment to manage staining.

Which of the following patient situations is associated with the highest caries risk? All answers are correct Arthritis Lactation Pregnancy Xerostomia

Its Xerostomia . Patients with low salivary flow rate are at a higher risk for dental caries. Reduced salivary flow prevents the flushing of food debris, bacteria, and acid away from the tooth structure. Salivary flow allows for the distribution of minerals to help remineralize tooth structure. Saliva can act as a buffer or can neutralize the acidic environment created by bacteria.

Patients who chose not to treat tooth decay may experience which of the following? No answers are correct All answers are correct Gingivitis Dietary deficiency Pain Occlusal interferences

Its all - ie option B Untreated caries will continue to enlarge and will eventually affect the pulp, cause irreversible pulp damage, and ultimately, tooth loss. As tooth decay progresses, the tooth continues to be weakened, resulting in probable fracture and may cause problems like: Loss of esthetics Changes in eating habits Occlusion issues

PATIENT tests POSITIVE HEP B ANTIBODY? All of his organs will be affect except. 1.Pancreas 2.Kidney 3.GI 4.thyroid

Its not thyroid. If the question is saying patient has hep B it means that liver is diseased therefore cholesterol synthesis is impaired. Thyroid hormones are made of cholesterol. -in hepatitis the pancreas(rbc destruction), kidneys(dark urine) and GI (vomitting) is affected to an extent but not thyroid

probing furcation best from? facial mesial. distal. lingual

Knowledge of the anatomy of multi-rooted teeth is essential in order to correctly identify the presence of furcation involvements. Maxillary molar teeth: the mesiobuccal roots are comparatively wide in the buccopalatal direction, and frequently have marked concavities the distobuccal roots are smaller in dimension, more rounded in outline and less frequently have distinct concavities the palatal roots are wider in the mesiodistal than in the buccopalatal direction Maxillary prernolar teeth: most maxillary first premolars are bifurcated at the middle third of the root trunk the canine fossa creates a deep concavity on the mesial root that merges with the opening of the furcation Mandibular molar teeth: generally have two roots of similar size and length * the mesial root is usually barbell-shaped due to mesial and distal concavities the distal root has a mesial concavity and usually has a smaller buccolingual dimension

dental managment of hemolytic pt and thalissemia

LA is the safest method for pain control. It is preferred to avoid prilocain which (in over dose) may precipitate methemoglobinemia. Drugs that can potentially cause hemolysis such as Aspirin and nonsteroidal anti- inflammatory drugs (NSAIDs) should be stopped. Prophylactic antibiotics (Penicillin V or Clindamycin) should be givenfor surgical procedures, and infections must be treated vigorously, sincethe patient may be immunocompromised if the spleen is non-functional or removed.

Perforation of teeth

Lateral >>> mesial Central >>>buccal Max 1pm >> mesial mandibular 1st premolar Buccal.

When a weaker abutment is not splinted to the adjacent stronger tooth in a fixed partial denture, the fixed partial denture will: a) Supraerupt b) Laterally drift c) Become mobile d) Get dislodged

Laterally drift

Which of the following is the best way to manage a tooth with a ledge that is detected while performing a root canal? Remove the ledge with a Gates Glidden bur Obturate the canal as far as was reamed Use a larger file to remove the ledge Use a smaller file to bypass the ledge

Ledges can be bypassed by pre-curving a smaller file to probe past the ledge. The canal coronal to the ledge must also be straightened to allow a file away from the curve. If the ledge cannot be bypassed, the canal should be cleaned and filled up to the ledge.

All of the following are characteristics of leukemic enlargement EXCEPT : Gingival enlargement Blush red & cyanotic gingival growth Seen in all patients : dentulous & edentulous in equal frequency Flat macules & papules due to accumulation of leukemic cells in dermis & cutaneous connective tissue

Leukemic gingival enlargement is not found in edentulous patients suggesting that it is the accumulation of immature leukemic blast cells in the gingiva adjacent to tooth surfaces with bacterial plaque. Leukemic gingival enlargement consists of a basic infiltration of the gingival corium by leukemic cells that increases the gingival thickness and creates gingival pockets in which bacterial plaque accumulates, initiating a secondary inflammatory lesion that contributes to the enlargement of the gingiva. It may be localized to the interdental papilla area or expand to include the marginal gingiva and partially cover the crowns of the teeth. Clinically, the gingiva appears bluish red and cyanotic, with a rounding and tenseness of the gingival margin. The abnormal accumulation of leukemic cells in the dermal and subcutaneous connective tissue is called leukemia cutis and forms elevated and flat macules and papules

Loma Linda design is made with a incisal access. It is used when esthetics is of a major concern : Both statements are true Both statements are false First statement is true. Second is false Second is true. First is false

Loma Linda Design The cavity preparation under this design is made with a lingual access. It is therefore best indicated in those areas where esthetics is the major concern and where the carious extent is such that the lingual marginal ridge is involved. Restoration of these cavities is primarily done with powdered gold and gold foil veneer.

vitd deficieny causes secondary hyperparathryoidism abd osteoporosis. this causes loss of periodontal attachmenta nd loss of teeth 1) both statements are correct 2)1st statement is correct 3)2nd statement is correct 4)both ststements are incorrect

Low levels of serum vitamin D and calcium result in stimulation of the release of parathyroid hormone (PTH). As deficiency progresses, the parathyroid is overstimulated and causes secondary hyperparathyroidism This release of PTH also results in phosphatemia, low levels of serum phosphorus that cause diminished mineralization of the collagen matrix, resulting in osteomalacia, and eventually osteoporosis. This inflammatory disease causes loss of periodontal attachment, including ligaments and alveolar bone.[23] Poor bone quality is thought to be a risk factor of periodontal disease, and it has been shown that vitamin D supplementation can reduce tooth loss.

Most common trauma Avulsion Instrusion Lateral luxation Fracture

Luxation for primary teeth fracture for permanent teeth

Child primary mo do caries t/t - 1-mod amalgam,2- MO DO amalgam,3- mo do composite

MO DO amalgam

Syndrome with macrodontia? Chondroectermal dysplasia Down syndrome Facial hemihypertrophy

Macrodontia in children may include: Associated with various syndromes: KBG syndrome Oculofaciocardio-dental syndrome Klinefelter syndrome 47,XYY syndrome Hemihyperplasia Polydactyly, postaxial, with dental and vertebral anomalies

Cell in granulomas

Macrophages (specifically histiocytes) are the cells that define a granuloma. They often, but not invariably, fuse to form multinucleated giant cells (Langhans giant cell). The macrophages in granulomas are often referred to as "epithelioid".

main reason failure of post comp tecnique shrinkage type of resin case selection

Major problems associated with composites failures are polymerization shrinkage & contraction stress related to polymerization shrinkage, water sorption, solubility, discoloration of restoration (staining), elution of material from restoration, marginal failures, secondary caries, and fracture of the restoration, post-operative sensitivity, and micro-leakage.

What would be hardest to anesthetise in irreversible pulpitis? Mand. Molar Max. Molar

Mand. Molar

Most common Cracked tooth

Mandibular second molars, followed by mandibular first molars and maxillary premolars are the most commonly affected teeth

FPD (maryland bridge)

Maryland bonded bridges (also called a resin-bonded bridge or a Maryland bridge) are made of porcelain, porcelain fused to metal, or plastic teeth and tissue supported by a metal or porcelain framework. Metal or porcelain wings often on just one side of the bridge are bonded to adjucent

Which of the following muscles is most likely to be affected by a depressed fracture of the zygomatic arch? Lateral pterygoid. Temporalis. Medial pterygoid. Masseter.

Masseter.

Teeth Most to least in ectopic eruption?

Max 1st molar max canine mandibular canine mandibular 2nd premolar and last max lateral

What will have the most traumatic effect on the abutment? Molar abutment with a premolar Pontic Max lateral abutment with a max central Pontic Max central abut with max lateral Pontic Max canine abut with max lateral Pontic

Max lateral abutment with a max central Pontic (It goes according to the Ante's Law "The sum of pericemental area of abutment teeth should be equal to or surpass that of teeth being replaced.")

Max ridge in denture pt is .............. over time? A)Narrow. B) wide. C) small. 4) large

Maxillary ridge -small . Resorption pattern-upward and inward Mandibular ridge -wide. Resorption pattern -downward and outward.

Inorganic component of calculus is crystalline ,which is more common in mandibular anterior region 1) hydroxyapatite 2) brushite

Md anterior - brushite posterior- magnesium whitlock

long term maintenance of opioid withdraw? methadone pethidine nalorphine fentanyl

Methadone relieves withdrawal symptoms and helps with detox. It is also used as a long-term maintenance medicine for opioid dependence.

Methotrexate having drug interaction with A. Nsaid B. Beta lactamase C. Beta blocker D. Alpha blocker

Methotrexate can be taken with other medications - NSAIDS (Celebrex®, Vioxx®, ibuprofen, naproxen), prednisone, Enbrel, plaquenil. You should not take methotrexate while taking any medicine containing trimethoprim-sulfa (Bactrim®, Sulfatrim®

Fatal alcohol syndrome Encephalopathy Mid face deficiency

Mid face deficiency

By product of polysulfide impression material is : Water Ethyl alcohol No by-product None of the above

Moisture and temperature have a significant effect on the course of the reaction. In particular, hot and humid conditions will accelerate the setting of polysulfide impression material. The reaction yields water as a by-product. Loss of this small molecule from the set material has a significant effect on the dimensional stability of the impression.

When treatment planning an RPD for patient what is the first thing you do?

Mounted cast and then surveying

most common salivary malignancy in children?

Mucoepidermoid carcinoma is the most common salivary gland malignancy in children.

Unusual mobility of shoulders and ability to bring them forward in midline is seen in : Mutational dysostosis Mandibulofacial dysostosis Craniosynostosis None of the above

Mutational dysostosis

Most powerfull electron shell Least powerfull electron shell

N-highest energy K-highest binding

What's meaning when you get the following reading on EPT : High reading -----> ??? Low reading ------> ??? No reading -------> ???

Necrosis Vital Calcified

SS files vs NiTi files?

NiTi rotary files remain better centered, produce less transportation, and instrument faster than stainless steel files due to their superior flexibility and resistance to torsional fracture. They have 10x the stress resistances of stainless steel (stronger).

The main biologic effects of dental material is : Toxicity Allergic reactions Trauma Both (A) and (B)

No dental device (including restorative materials) is absolutely safe. Safety is relative, and the selection and use of dental devices or materials are based on the assumption that the benefits of such use far outweigh the known biological risks. However, there is always uncertainty over the probability that a patient will experience adverse effects from dental treatment. The two main biological effects are allergic and toxic reactions.

Leteral translation to semiadjestable articulater which mov on articulater? Leteral Protrusive Retrusive Opening and closing

Non adjustable -> horizontal axis only. Semi -> lateral & horizontal axis.protrusive Full -> 3D movement.

antiarrythmics, b blockers and ca blockers all contraindicated in chf. Why?

Non-dihydropyridine calcium channel blockers -verapamil and diltiazem-Negative inotropic effect may further depress cardiac function. Risk is greatest with verapamil, then diltiazem and least risk with dihydropyridines, but use with caution. Some antiarrhythmics, such-flecainide and dronedarone • Flecainide may increase the risk of ventricular arrhythmias in impaired left ventricular function and may worsen heart failure.

turner's hypoplasia causes?

Nutritional factors. chicken pox, congenital syphilis Hypocalcemia. dental fluorosis Birth injury. Preterm birth. Infection. Trauma from a deciduous tooth.

most common odontogenic tumor of mesenchymal origin?

Odontogenic mesenchymal tumors are: Odontogenic myxoma Odontogenic fibroma Cementoblastoma i think myxoma is most common

method for subgingival cleaning plaque/debris electric brush tooth brush floss oral irrigator

Oral irrigator,,,,,, floss only remove from proximal surface and accessible subgungival area... Irrigator cab remove plaque and debris from sybgingival area including deep pocket as far as 7mm and also furcation area

Pt with TMJ pain in morning for 30 mins which wakes & progress during the day, limited joint space, Radiographically formation of osteo pain divert head, clinicall pain ausicular area; - Osteo-Arthritis - Myofacial syndrome (no x-ray) - Rhumatoid arthritis

Osteo-Arthritis

bisphosphonate oral drug iv drug

Pamidronate, Zolandronate, Ibandronate and Etidronate (all IV) Residronate, Alandronate (Only oral) Ibandronate and etidronate (IV +oral)

Narcotics is contraindicated in which of the following: A) 2nd trimester pregnancy. B) hypothyroidism

Patients with hypothyroidism may have increased risk of respiratory depression and prolonged CNS depression associated with the use of narcotic (opioid) analgesic agents NSAIDs should generally be avoided in pregnancy (despite Category B before third trimester) Second trimester use is likely safe Miscarriage risk in first trimester Premature Ductus Arteriosus closure in third trimester Tramadol should be avoided in pregnancy Second trimester use may be safe Fetal toxicity in animals (highest risk in first trimester) Respiratory problems and withdrawal symptoms in newborn (avoid in third trimester) Opioids should be avoided in pregnancy unless there is no viable alternative First trimester use is associated with heart defects and Neural Tube Defects (e.g. Spina bifida) Late pregnancy use is associated with respiratory distress and Opioid Withdrawal

Which of the following is histologically identical to a pyogenic granuloma? 1. Hemangioma 2. Peripheral giant cell granuloma 3. Central giant cell granuloma

Peripheral giant cell granuloma

la reversal?

Phentolamine mesylate (OraVerse), a nonselective a-adrenergic blocking drug, is the first therapeutic agent marketed for the reversal of soft-tissue anaesthesia and the associated functional deficits resulting from an intraoral submucosal injection of a local anaesthetic containing a vasoconstrictor.

What is Gingival hyperplasia caused due to phenytoin now termed? Dilatin hyperplasia Phenytoin hyperplasia Phenytoin - induced gingival hyperplasia Phenytoin - induced gingival overgrowth

Phenytoin - induced gingival overgrowth Phenytoin (Dilantin, or diphenylhydantoin), a major anti-convulsing agent used in the treatment of epilepsy, was first introduced in 1938 by Merrit and Puntnam. Varying degrees of gingival hyperplasia, one of the most common side ef-fects of phenytoin therapy, were first described by Kimball in 1939. He reported that 57% of 119 patients taking phe-nytoin for the control of seizure activity experienced some degree of gingival overgrowth. • Early research showed an increase in the number of fibroblasts in patients receiving dilantin, thus the condition was termed dilantin hyperplasia. Now the term phenytoin induced gingival overgrowth is preferred because Hassel & col-leagues found true hyperplasia not to exist.

Which drug has methemoglobinemia as side effect.?

Primaquine and local anesthetics, such as topical or spray benzocaine (used prior to upper endoscopic procedures) and prilocaine, can cause methemoglobinemia. Treatment includes cessation of the inducing agent, oxygen, and methylene blue.

Drugs used for conscious sedation?

Propofol and fentanyl Sodium pentathol Valium

Propoxyphene Schedule

Propoxyphene is an opioid medication that was first approved by FDA in 1957. It has been marketed in the United States since 1976 as a Schedule IV Controlled Substances used as either a single ingredient (e.g., Darvon) or in combination with acetaminophen (e.g., Darvocet) to treat mild to moderate pain.

G.V. Black Class IV lesions typically occur on which of the following surfaces? Proximal surfaces of anteriors including incisal edges Proximal surfaces of anteriors NOT including incisal edges Smooth facial and lingual surfaces in the gingival 1/3 Incisal edges of cusp tips

Proximal surfaces of anteriors including incisal edges

Emancipated minor?

Pt married Pregnant Individual dependence

FDA want to check safety medicament?

RCT-Randomized clinical trial

A young adult has generalized areas of chalky, opaque, cavitated lesions on the vestibular surface of multiple teeth and interproximal gingiva that is red and swollen. Which of the following is the most appropriate diagnosis? Arrested caries. Secondary caries. Interproximal caries. Smooth surface caries. Rampant caries.

Rampant caries.

Retention form & internal line angle in class ii inlay ?

Rectangular box and reverse bevel z retention form in class 2 inlay

Accidentally etched enamel complete remenralized A.few days B.few hours C.few months D.never remeneralized

Remineralization of etched enamel occurs from the saliva, and after 24 h it is indistinguishable from untreated enamel. this wat given in oxford hand book of clinical dentistry pg 355

Histologically 'jigsaw or mosaic' pattern is seen in : Fibrous dysplasia Familial fibrous dysplasia Osteopetrosis Pagets disease

Repeated episodes of bone removal and formation results in the appearance of many small irregularly shaped bone fragments that appear to be joined in a jigsaw or mosaic pattern with deeply staining hematoxyphilic reveral lines. This pattern is the histologic hallmark of Paget's disease

Not an Advantage of distraction osteogenesis over other osteotomies? Requires more maintainence Relapse Paresthesia

Requires more maintainence?

class v restoration? GI compomer

Resin-Modified Glass Ionomer Cements (RMGICs) and compomers (or modified composites). ... restoration of anterior teeth and were recommended for restoring Class III and Class V cavity preparations.

A 78 year old patient presents with several carious lesions on the root surfaces of the maxillary posterior teeth. The most appropriate restorative material for these lesions is microfilled composite resin. hybrid composite resin. amalgam. Resin-modified glass ionomer.

Resin-modified glass ionomer.

est material for cementation of porcelain crown

Resin-reinforced glass ionomer (RRGI) cements appear to be better than zinc phosphate and glass ionomer cements when placing porcelain-to-metal crowns.

Retention grooves are placed in the ____________,and____________ line angles and extended to the height of the ____________ in an amalgam class 2 preparation.

Retention grooves: Placed on axiofacial and axiolingual line angles. Extend from the gingival floor to the occlusal surface

Define reverse architecture ?

Reversed architecture defects are produced by loss of interdental bone, including the facial plates, lingual plates,or both, without concomitant loss of radicular bone, thereby reversing the normal architecture Such defects are more common in the maxilla when the crest of the interdental gingiva or bone is located apical to it's midfacial and mid -lingual margins.

Which of the following is the suggested protocol if the pulpal floor is perforated during access preparation? - A. Repair the perforation and continue the root canal treatment at a subsequent appointment. . Perform no further treatment at this time and schedule the patient for an extraction. . Repair the perforation at a subsequent appointment only if symptoms occur. . Repair the perforation immediately with mineral trioxide aggregate (MTA/Portland Cement), inform the patient, and reassess the tooth at the recall appointment. . Continue the root canal treatment as normally planned because tertiary dentin will fill in.

Root perforations during endodontic treatments should be addressed immediately through the use of MTA to help stimulate healing of periradicular tissues and to promote cementum deposition. Mineral Trioxide Aggregate (MTA) is regarded to be the most ideal perforation repair material because of its ability to create a seal when moisture is present. MTA is biocompatible and is capable of inducing periradicular tissue repair and cementum deposition.

Russell body?

Russell body is characteristic of the distended endoplasmic reticulum. They are found in the peripheral areas of tumors seen in multiple myeloma.

Which skin disease has endocarditis and gloom?

SLE and splinter hemmorhages

Which of the following pontic designs will make it most difficult to maintain optimal oral hygiene? Modified ridge lap. Hygienic. Conical. Ovoid. Saddle.

Saddle

Bennet angle is the angle formed during lateral movement, by the path of the advancing condyle and the: a. Horizontal plane b. Anterior plane c Lateral plane d. Sagittal plane

Sagittal plane

Pt. With parkinson what less technique benefit? Chair pillow Protect eyes Saliva suction Monitor case for any change

Saliva suction- Dry mouth is common among patients of Parkinson's disease, usually due to medicine, carbidopa. Also, photophobia or decreased accommodation to light is seen. Chair pillow is needed due to continuous jerking of body/ head.

Most related to seizure A. Hyperkalemia B. Hyponatrrmia C. Hypoglycemia D. Hyperglycemia

Seizures are more frequently observed in patients with sodium disorders (especially hyponatremia), hypocalcemia, and hypomagnesemia.

Countersinking?

Site preperation in bone with drill same as in implant flutes so implant can passively fit into bone with stability-Coronal end enlargement of osteotomy

Increase in caries is seen in

Sjogrens syndrome Cerebral palsy

What happens to the length in the max arch?

Slight decrease due to incisor uprighting

Veneer fracture, what do you do ?

Small chipping may be corrected through recontouring and polishing f the patient has fractured pieces Remove the cement from the tooth and/or the internal surface of the veneer with a rotary instrument. In some cases, it may be impossible to remove cement from the internal surface of the veneer without causing significant damage to the veneer. These cases should be treated as if the patient does not have the fractured veneer pieces with them. If resin cement removal is difficult on a veneer fragment, the cement could remain but needs to be acid etched or microetched (sandblasted). The cleaned piece should be tried in to ensure a good fit. To achieve proper fitting prior to re-cementation, the piece should correspond with the fracture line. Produce a 45° bevel in the porcelain around the area to be repaired. Clean off the internal surface of the fractured porcelain and microetch it with silica-coated aluminum oxide particles (50mm). Etch the microfractured ceramic fragment for a minute with a 9-10% buffered hydrofluoric acid gel. Suction the acid gel and then rinse with a copious amount of water and air dry. The surface should appear dull and frosty. Apply silane coupling agent mixture and allow to dwell for 30 seconds. Dry with an air syringe. Etch the exposed tooth preparation for 15-30 seconds with 35-38% phosphoric acid. Apply a silane coupling agent to the etched ceramic surface of the intact veneer and lightly air dry. Apply bonding resin over ceramic surface and tooth structures as per the manufacturer's directions. Apply adhesive on veneer surface and lightly thin it out with air. Place light-cured resin cement onto the internal surface of the fragment and seat over the tooth structure. Eliminate the excess cement with a microbrush and polymerize under light pressure to seat the fragment for a total of 60 seconds from all aspects: facial, lingual and incisal. If the porcelain fracture extended to an interproximal area, you can use a matrix strip when bonding and interproximal finishing strips to remove excess resin cement and polish. Remove marginal flash with fine needle diamond or multi-fluted carbide finishing burs used in high-speed handpiece with water spray, then use polishing points/discs/paste to add lustre to the surface. If the patient does not have the fractured pieces There are 2 options: Remove the remaining portion of the veneer, refine preparation and take an impression to fabricate a new replacement veneer. If the fractured portion was small then repair it by adding resin composite freehand. This will require roughening the enamel surfaces with a medium-grit, needle-shaped diamond bur to remove cement remnants and expose fresh enamel for optimum bonding. Phosphoric acid-etching and bonding follows before the resin composite repair is completed. This can be done freehand.

Restoring a carious tooth relieved the toothache in a patient which further motivated him to perform better oral health care. This is a type of 1. Positive reinforcement 2. Negative reinforcement 3. Positive punishment 4. Negative punishment

Smriti Dasari Positive - adding / giving something good (reward, praise) to encourage certain behaviour , negative - taking away the bad to encourage certain behaviour.... In this case you're taking away the pain/bad (negative ) and motivating the patient to maintain oral hygiene ( reinforcing)

MA 10 exposure 1 second then if u do exposure 0.5 then whats MA?

So if 10*1= 10mas So if time is 0.5 sec then ma u have to inc 20 to make 10 mas So 20*0.5 = 10 mas

thin long finger like projections of epithelium with a thin central connective tissue core and occasionally showing keratinocytes is histological feature of : Keratoacanthoma Squamous papilloma Verruciform xanthoma Herpes simplex

Squamous papilloma

What is the difference between the stationary and cortical anchorage?

Stationary - Type of anchorage where the teeth move in a translation manner, not tipping. ortical bone is known to resorb slower than the medullary bone. Therefore, cortical anchorage is used in orthodontics to allow for slower movement of teeth, especially molars in the posterior dentition.

Reason for straight line access

Straight line access will reduce the likelihood of iatrogenic problems such as zips, elbows and ledges

The pain associated with pulpitis often disappears when a patient visits a dental office. This occurs due to which of the following events? Local mediators of pulpal pain are not released when a stress-induced rise in blood pressure occurs Stress-induced sympathetic activity inhibits pulpal sensory fibres that cause pain Stress-induced sympathetic activity causes vasodilatation of pulpal blood vessels Pulpal pain receptors undergo fatigue

Stress-induced sympathetic activity inhibits pulpal sensory fibres that cause pain

mercury poisoning?

Sub acute hair loss muscle weakness Acute tremor Chronic blindness

which of the following is conditioned stimulus a) dentist b)dental chair

Such conditioning, known as classical conditioning, was first demonstrated by Pavlov and is defined as being the repeated pairing of a neutral stimulus (e.g., a stimulus which by itself has no real meaning or is neutral, for example the dentist's chair, dentist's office or dentist's practice location) with an unconditioned stimulus (e.g., a stimulus which has meaning naturally or innately in its capacity to produce a response, for example pain).

Which of the following carbohydrates found in the human diet is most responsible for caries? Galactose Glucose Sucrose Lactose Cellulose Glycogen

Sucrose

71/2 years old pedo pt 46 erupt and each antagonist how to describe - Sulcus of junctional epi at enamel and 2/3 of crown - Sulcus of junctional epi at CEJ and 2/3 of crown - Sulcus of enamel, junctional epi at CEJ and ¾ crown

Sulcus of junctional epi at enamel and 2/3 of crown

in moderate stage in insulin reaction patient present with : a) sweating b) belligerence

Symptoms of Insulin Reaction Mild Stage Severe Stage • Hunger ... the patient becomes incoherent, uncooperative, and sometimes belligerent or ... rapid and thready pulse, hypotension, and hypothermia may be present

Simillar to synchondrosis ? Suture Synostosis Frontanelle

Synchondroses - have a temporary plate of hyaline cartilage - example: epiphyseal plate. Once ossified it is called a synostosis.

Epi+proponalol =BP INC Epi +prazosin = BP DEC BUT WHAT ARE THE EFFECTS ON HEART RATE?

Taking beta-blockers with epinephrine may cause blood pressure to be increased. heart rate may slow down. prazosin. selective alpha blocker -> block alpha 1 only. Dec BP. get reflex tachycardia

Maximum radiation source in USA? A) Cosmic B) occupational C) medical D) terrestrial.

Terrestrial max radiation Next cosmic medical , occupational.occupational is very minimal especially dental. There is no chance that everyone need to be exposed to medical but there is every chance that one may be exposed to terrestrial / natural radon . So most opt answer terrestrial

systemic antibiotics should be given in localized aggressive periodontitis ?

Tetracycline (250 mg 3 times daily for 2 weeks). Metronidazole 400 mg and Amoxicillin 250 mg 4 times daily for 1 week in severe cases.

thalidomide is used in?

Thalidomide is used for a number of conditions including erythema nodosum leprosum, multiple myeloma (in combination with dexamethasone), and various other cancers, for some symptoms of HIV/AIDS, Crohn's disease, sarcoidosis, graft-versus-host disease, rheumatoid arthritis, Behçet's disease and a number of skin conditions that have not responded to usual treatment.

Best radiograph of midface is... Water Latetal sk Pa skull Towne'sview

The SMV view provides a good view for zygomatic arch and midface fractures.

Typical recommended W/P ratio range for Type II plaster : 0.45 to 0.50 0.28 to 0.30 0.04 to 0.05 0.22 to 0.24

The W/P ratio is an important factor in determining the physical and chemical properties of the final gypsum. Typical recommended W/P ratio range for Type II plaster is 0.45 to 0.50; Type III plaster is 0.28 to 0.30; Type IV stone is 0.22 to 0.24.This means that lesser powder can be incorporated into water for plaster as compared to stone. This reflects on the strength.

Which of the following describes what would happen to the caries risk of a mouse that was fed by a gut tube so that no carbohydrates were allowed in the oral cavity? The caries risk of the mouse would decrease. The caries risk of the mouse would increase. The caries rate would stay the same as if the mouse was fed directly.

The caries risk of the mouse would decrease.

Which of the following is NOT a reason that adjacent class II or class III cavities should be prepared and restored using composite resin during a single appointment? Time is conserved for the clinician and patient The contact area will be better Matching the color is easier Access to the cavities is simplified

The contact area will be better

Why is a surgical stent often necessary for an immediate denture A) prevent hematoma B) to determine occlusion C) to give an idea of the anatomy of the region D) none of the above E) all of the above

The correct answer is (C). An immediate denture is a complete denture or partial denture inserted on the same day, immediately following the removal of natural teeth. A surgical stent (a guide for recontouring tissues after extraction) is often necessary and more follow-up visits are needed for adjustments and re-fitting. Understanding the cross-sectional 3-D anatomy using CT images will allow for an new appreciation of the true volume of bone surrounding our natural teeth. The "Triangle of Bone" is a concept which aids clinicians in determing this available bone - and thus the correct placement of an implant, with the desire tooth position always remaining constant. There are several advantages of an immediate denture. The most important factor is that you will never need to appear in public without teeth. It is also easier to duplicate the shape, color and arrangement of your natural teeth while some are still present in your mouth. When an immediate denture is inserted at the time of extraction, it will act as a band-aid to protect the tissues and reduce bleeding. An immediate denture will allow you to establish your speech patterns early. You will not have to learn to speak without a denture in place and then later relearn to speak with a new denture. An immediate denture will also allow you to chew better than without any teeth and minimize facial distortion that may occur when teeth are removed

What is the MOST toxic form of mercury in dentistry? A) elemental mercury B) ethyl mercury C) methyl mercury D) organic mercury E) all of the above

The correct answer is (D). Mercury and its compounds have been used in medicine, although they are much less common today than they once were, now that the toxic effects of mercury and its compounds are more widely understood. The element mercury is an ingredient in dental amalgams.The most toxic forms of mercury are its organic compounds, such as dimethylmercury and methylmercury. Mercury can cause both chronic and acute poisoning. if organic is not in option choose methyl mercury

What is the MOST common measure of dispersion? A) coefficient of variation B) mean C) median D) mode E) standard deviation

The correct answer is (E). The data values in a sample are not all the same. This variation between values is called dispersion. When the dispersion is large, the values are widely scattered; when it is small they are tightly clustered. The width of diagrams such as dot plots, box plots, stem and leaf plots is greater for samples with more dispersion and vice versa. There are several measures of dispersion, the most common being the standard deviation. These measures indicate to what degree the individual observations of a data set are dispersed or 'spread out' around their mean. Standard deviation is a measure of the spread or dispersion of a set of data. It is calculated by taking the square root of the variance.

Eccrine sweat gland innervation = Sympathetic cholinergic . T/F ?

The duct of eccrine gland is formed by two layers of cuboidal epithelial cells. Eccrine glands are innervated by the sympathetic nervous system, primarily by cholinergic fibers whose discharge is altered primarily by changes in deep body temperature (core temperature), but by adrenergic fibers as well.

Fastest route of absorption is A. inhalation B. intravenous (IV) C. oral

The fastest route of absorption is inhalation, and not as mistakenly considered the intravenous administration

Frontanelles closing time Synchondrosis

The fontanelle in the back of the head (posterior fontanelle) most often closes by the time an infant is 1 to 2 months old. The fontanelle at the top of the head (anterior fontanelle) most often closes between 7 to 19 months. intersphenoid-at birth(first to close) spheno-occipital-17-20yrs spheno-ethmoidal-5-25yrs

How much is the probing force? 0.50 N 0.85 N 0.75 N 0.80 N

The forces of 0.75 N have been found to be well tolerated and accurate

Which of the following bacterial species is considered an early colonizer of supragingival plaque? Streptococcus sanguinis Streptococcus salivarious Porphyromonas gingivalis Lactobacillus casei Aggregatibacter actinomycetemcomitans

The formation of dental plaque consists of 3 distinct stages: Formation of pellicle on the tooth surface Initial colonization by bacteria Secondary colonization with plaque maturation Within a few seconds after cleaning teeth, a thin layer called pellicle is formed on the tooth surface by the saliva, which is then colonized by bacteria within a few hours. The initial colonizing bacteria is collectively known as supragingival plaque and predominantly consists of gram-positive facultative microorganisms. Actinomyces viscosus Streptococcus sanguinis

Patient has an all veneer on incisal edge, small piece of porcelain came off and wants you to fix the chip only, what is the sequence of events?

The fractured porcelain veneer is air-abrade using aluminum oxide to increase micromechanical retention with the abrasion unit------- etchant-----silane-----Bonding resin is placed on the restoration ------Composite material----The completed restoration and polish.

which space involved in trismus?

The hallmark of a masticatory space infection is trismus or infection in anterior compartment of lateral pharyngeal space results in trismus.

Which of the following locations would a perforation present the highest failure risk? Furcation Middle 1/3 of the root Apical 1/3 of the root Crown Apex

The highest risk for endodontic failure is the perforation through the furcation because it is most difficult to seal. Perforations of the crown may be sealed with the final restoration. Perforations near the root apex can be sealed during obturation. Teeth with endodontic perforations may still be successful. Perforations are NOT a guarantee of endodontic failure.

Presence of lipoxin in periodontal tissue indicate : Worsening of periodontal inflammation Resolution of periodontal inflammation Activation of gram positive bacteria Activation of gram negative bacteria

The lipoxins include lipoxin A4 (LXA4) and lipoxin B4 (LXB4). The appearance of these molecules signals the resolution of inflammation. Lipoxins are lipoxygenase derived eicosanoids and are generated from arachidonic acid. They are highly potent, possessing biologic activity at very low concentrations, and inhibit neutrophil recruitment, chemotaxis, and adhesion. (Ref. Carranza's, 11th Edition, Pg. 206)

Which of the following is the minimum number of flutes needed on a carbide bur for it to be considered a "finishing bur"? A. Finishing carbide burs do not have flutes B. 24 C. 6 D. 18 E. 12

The minimum number of flutes needed on a carbide bur to be considered as a finishing bur is 12. Carbide burs are used most commonly for excavating and preparing cavities, finishing cavity walls, finishing restoration surfaces, drilling old fillings, finishing crown preparations, contouring bone, removing impacted teeth, and separating crowns and bridges. Carbide burs are made of tungsten carbide, a metal that is extremely hard and can withstand high temperatures. Because of their hardness, carbide burs can maintain a sharp cutting edge and be used many times without becoming dull. Carbide burs are brittle and have a tendency to fracture under pressure. Because of their brittleness, they are best operated at high speeds with light pressure. Trimming and finishing burs have more blades than operative burs, and the blades are closer together and shallower, making them ideal for the fine finishing and polishing

Most common fracture in the face?

The most common isolated fracture site nasal bone (37.7%), mandible (30%), orbital bones (7.6%) , zygoma (5.7%), maxilla (1.3%) and the frontal bone (0.3%)

Most esthetic pontic Ovate modified ridge lap

The ovate pontic is the most esthetic of all pontic designs because it most closely resembles the emergence profile of natural teeth. Ovate pontic. This is a pontic form with a rounded base; it is indicated when esthetics are of paramount importance.Only disadvantage of ovate is surgically placed

The recommended vertical angulation when making bite-wing radiographs of the posterior teeth may vary from - -10 degrees to -5 degrees . -5 degrees to 10 degrees . 0 degrees to +5 degrees . +5 degrees to +10 degrees

The recommended vertical angulation for bite-wing radiographs usually ranges from +5 to +10 degrees because the upper and lower portion of the film contacts the palate and the lingual of lower posterior teeth at a different angle. When taking bitewing radiographs, the upper part of the film is usually positioned at an angle of +20 degrees while it contacts the palate and the lower part of the film is upright. Because of this angulation, the x-ray tube must be angulated to approximately +5 to +10 degrees vertically to compensate for the film position and to achieve accurate bitewing records.

Role of plaque vague in what? Aggressive Desquamative ANUG

The role of plaque is vague in desquamative gingivitis.

One of the most common solid tumor of earlt childhood

The types of cancers that develop most often in children are different from those in adults. Leukemia. Brain and other central nervous system tumors. Neuroblastoma. Wilms tumor. Lymphoma (including both Hodgkin and non-Hodgkin) Rhabdomyosarcoma. Retinoblastoma. Bone cancer (including osteosarcoma and Ewing sarcoma)

Extrusion of canine what flap technique is used 1)Envelope flap 2) Semilunar flap 3) Apical repositioning flap

There are 3 tech for uncovering a labially impacted maxillary canine: excisional uncovering apically positioned flap closed eruption techniques

the drug of choice to treat overdosage with tricyclic antidepressants? (ASDA II M) a. atropine. b. phenytoin. c. physostigmine. d. pentobarbital. e. amphetamine.

There is no role for physostigmine in the treatment of tricyclic toxicity as it may increase cardiac toxicity and cause seizures.there is considerable evidence suggesting that the life-threatening manifestations of tricyclic antidepressant overdose--the conduction defects, bradyarrhythmias, heart block, etc--are much more like quinidine and are more appropriately treated with phenytoin,(acc to google)

Transilumination diagnosis Class 2 Class3

To detect anterior caries the transilluminator is placed first on the labio-cervical region of the tooth and moved from the mesial to the distal while observing from the lingual with a dental mirror. The transilluminator is then placed on the lingual-cervical region of the tooth both mesially and distally while viewing from the labial surface. Anterior caries will show up as a well defined dark shadow in the class III region of the tooth

Purpose of GTR??

To direct the growth of new bone and gingival tissue at sites with insufficient volumes or dimensions of bone or gingiva for proper function, esthetics or prosthetic restoration. indication -ridge augmentation , class 2 and 3 furcation defects, gingival recession

How do you transfer a patient in a wheelchair?

To get the patient into a seated position, roll the patient onto the same side as the wheelchair. Put one of your arms under the patient's shoulders and one behind the knees. Bend your knees.

early stage of peripheral cemental dysplasia is best differentiated from chronic apical periodontitis by percussion subjective symptoms periodontal examination results of pulp testing rediological appearance

Tooth vitality! PCD will test + for vitality While Chronic periapical odontitis will test - for vitality.

1-Probing reference point in tooth ----- 2-probing reference point In implant -

Tooth. .cej Implant..implant shoulder

Difference between self etch and total etch?

Total etch requires separate phosphoric acids top to etch enamel n dentin a subsequent rinse and application of primer n bond.. Self etch system Hav an acidic resin which etched n primes with out the needful etchings ringing ten subsequent application of bond Self etch - smear layer not removed Total etch- smear layer removed

Streptomycin cause respiratory difficult.. t f

True- by neuromuscular blockade cause difficult breathing Neuromuscular blockade is a rare but potentially life-threatening toxicity as it can affect the muscles of respiration. The aminoglycosides are the agents most frequently associated with this side effect. In the presence of very high concentrations of aminoglycoside at the neuromuscular junction, there is interference with neuromuscular transmission, resulting in the potential for paralysis and respiratory arrest. Excessive serum concentrations can occur during rapid intravenous administration.

True or false Pt on B-Blocker - took epinephrine Will lead to increase blood pressure and bradycardia

True. Causes hypertensive crisis

Correlation analysis shows that as the income of population increases, the number of decayed teeth decreases. Therefore, an expected value for this correlation coefficient (r) would be -1 yes or no

True.. Correlation coefficient (r)is the relationship of 2 variables(x and y) so if both increace in same direction so the value is positive r.. But if x and y in opposite direction like in q assume income is( x) and decayed teeth (y )so r will be negative

Type of wax used for inlay, onlay ad full crowns ?

Type c soft inlay wax used

Which of the following type of GIC is used for luting purposes ? Type I Type II Type III Type IV

Types of GIC · Type I : luting agents · Type II: restorative · Type III : liners & bases

Cavity liners minimal thickness 1mm 1.5mm 15mm 125mm

Types of liners: 1.thin film liners: A . Solution liners: varnishes, 2 - 5 µm thick. B . Suspension liners: 20-25 µm. 2.thicker liners: also known as Cement liners: 200-1000 µm (0.2-1mm) used for thermal protection, pulpal medication.

periodontal pack purpose?

USES OF PERIODONTAL DRESSING 1. Provide mechanical protection for the surgical wound and therefore facilitate healing . 2. Enchancement of patient comfort . 3. Prevents post operative bleeding by maintaining the initial clot in place. 4. Maintainance of debris free area. 5. Control of bleeding 6. Supports mobile teeth during healing PERIODONTAL DRESSING 7. Helps in shaping or molding the newly formed tissue 8. Provide patient comfort by isolating area from external irritations or injuries.

Amalgam showing in the mesio facial of 13 with dark and rough surface Corrosion Defect in condensation Under trituration Defect in matrix application

Under trituration

management of angina pectoris in dental office? Angina Pectoris Pain can last up to 20 minutes; however, prolonged discomfort should be evaluated in an appropriate emergency medical facility TREATMENT 1. Avoid situations that can produce increased myocardial oxygen demand 2. Stop procedure and allow the patient to rest 3. Monitor vital signs repeatedly 4. Place the patient in a semi reclined position 5. Provide supplemental oxygen Administer sublingual nitroglycerin (0.4 mg) every 5 minutes for three doses; if symptoms are not relieved, assume that the patient is having a myocardial infarction and transport him or her to an appropriate emergency medical facility.

Use short appointments (less than one hour), preferably in the morning. The intraoperative use of nitrous oxide and oxygen is also a reasonable strategy for patients with cardiovascular disease, particularly those with ischemic heart disease. Effective local anesthesia is important in order to avoid undue stress during the appointment as long as the guidelines for the administration of epinephrine are followed. he use of epinephrine impregnated gingival displacement cord should be strictly avoided in patients with cardiovascular disease.

what validity and Reliability differ in ?

Validity is the correctness of a result/ results. Reliability is the same result probability.

Protease is a virulence factor of P.gingivalis. Its function is : Inhibition of IL-8 secretion Resistance to complement Degradation of signaling molecules Induction of apoptosis in host cells

Virulence factors of P.gingivalis that Interact with the Immune System Virulence Factor Effect on Immune System Proteases (gingipains) Degradation of signalling molecules (CD14) and cytokines (IL-1b, IL-6) Cell invasion capability Inhibition of IL-8 secretion LPS Antagonism of the stimulatory effects of LPS from other species; no upregulation of E-selectin Fimbriae Inhibition of IL-12 secretion in macrophages Cell surface polysaccharide Resistance to complement Short-chain fatty acids Induction of apoptosis in host cells (Ref. Carranza's, 11th Edition, Pg. 208)

herpes simplex is the most common cause of bell's palsy?

Viruses that have been linked to Bell's palsy include the virus that causes: Cold sores and genital herpes (herpes simplex) Chickenpox and shingles (herpes zoster)

How to distinguish periapical cyst a) vitality b) precussion

Vitality Bc sometimes the periapical cyst is chronic and you can't know it from percussion

Which is the most cost-effective means of decreasing caries in the general population? Topical fluoride varnish application programs Mouthwash programs Plaque control programs Water fluoridation School/community sealant programs

Water fluoridation

Which crown is the weakest? 1,Empress 2,Euphlite 3,zirconia Examples of leucite-reinforced ceramics include: Empress Examples of lithium-disilicate ceramics include: IPS e.max Press, IPS e.max CAD Examples of glass-infiltrated ceramics include: In-Ceram Spinell, In-Ceram Alumina

Weakest is Euphlite Weaker is leucite reinforced ceramic ( Empress ) Strongest is Zirconia The second stronger after Zirconia is Aluminum

fetal alcohol syndrome?

When mothers ingest alcohol when they r pregnant,symptoms-abnormal appearance, short height, low body weight, small head size, poor coordination, low intelligence, behavior problems, and problems with hearing or seeing.

Each of the following could be ruled out for the lesion distal to the mandibular third molar EXCEPT one. Which one is this EXCEPTION?

When you rule out, A and D are immediately eliminated. B & C remains.. Osteomas are relatively rare in jaws, Fibromas are more common. Have read somewhere that central odontogenic fibromas occur mostly in between condyle and distal root of 3rd molar. Stafne defect are on medial side and below the inferior alveolar canal between the molar and angle.. questn says dital to molar.. so rule out D

Interferences in working non working and protrusion and their corrections

Working interference - LUBL Correction- BULL Non-working Interference - LUBL Correction- LUBL Protrusive Interference - DUML Correction - MUDL Retrusive Interference - MUDL Correction - DUML

Unilateral post crossbite If due to thumb sucking habit quad helix , or w arch appliance Functional cross bite then franckle?

Yes.. quad helix or W arch.. 👍 Frankel app mainly used when there is problem with lip and tongue. It guides the tongue to stay in proper position as well as the buccal shield, moves away the muscle from post teeth leading to correction of crossbite by expansion of arch. It is best used in young adults where there is problem with musculature.. In that way u can say that frankel can be used in Functional cross bite.. the effect of frankel is best seen in Class III

zone of dentinal caries?

Zones of carious dentin Normal dentin Subtransparent Transparent Turbid Infected Zones Incipient Enamel Translucent Dark Body Surface

1. how do you treat traumatic bone cyst a. leave it alone b. excise c. give meds

a

When primary mandibular right first and second molars are lost in an 8-year-old patient, space is best maintained by constructing and placing multiple space maintainers. bands on the remaining teeth. a functional removable appliance. a bilateral nonfunctional space maintainer

a bilateral nonfunctional space maintainer

When performing a periodontal screening and recording (PSR) for a patient, the code asterisk (*) is used for a sextant when a tooth in the sextant needs to be extracted. all the teeth in the sextant are missing. a tooth in the sextant has supraerupted. a mucogingival problem is present.

a mucogingival problem is present.

When pt say s,ch ? - tongue b/w incisal edge of ant teeth -incisal edges far apart - incisal edges touching - incisal edges almost touching

a or b ( "S" position is offered by Bilabial sounds - formed by lips. setting incisal edges of four lower anterior teeth slightlylingual to labial edges of upper incisors teeth with space of 1 - 1.5 mm when "S" or "Z" is pronunced

Child had carious. Lower molar with swelling and complain from mild pain and on radiograph show mild extension on this area with scattered radioopaque lines Paget' disease Fibrous dysplasia Garre'sosteomylities Osteosarcoma

a or d

The effectives of toothbrushing is best measured by: a) The amount and location of plaque. b) The caries experience. c) The toothbrushing frequency. d) The condition of the toothbrush

a) The amount and location of plaque.

Which of the following does not associate with Regional Odontodysplasia a) germination b) derived from epithelial c) derived from meschamyal d) ghost teeth

a) germination explan-Regional odontodysplasia is a rare development anomaly affecting the teeth with an unknown etiology. This dental abnormality involves the hard tissues of the teeth that are derived from both epithelial (enamel) and mesenchymal (dentine & cementum) components of the tooth forming apparatus. regional odontodysplasia, demonstrating the "ghost-like" appearance of the teeth

82 yr old patient, son is the GUARDIAN what sld the dentist consider while treating a)consent of guardian b)consent from patient c)guardians consent req only in irreversible procedures

a)consent of guardian

what occlusion does flush terminal plane lead to normally?

a)flush terminal plane lead to -class1(late mesial shift), class 2 , end to end b)mesial step lead to- class 1 normally, class 3 c)distal step lead to- class 2

best indicator of periodontal disease at a specific site is the evidence of a)sub gingival calculus b)past disease

a)sub gingival calculus

As caries progresses from enamel to dentin and toward the pulp interproximally the radiographic appearance resembles which of the following a)v shape in enamel with apex toward Dej and v shape in dentin with apex toward pulp b)v shape in enamel with apex toward the proximal surface and v shape in dentin with the apex toward pulp resulting in diamond appearance c)v shape in enamel wit apex toward the dej and v shape in dentin apex toward dej resulting in bowtie appearance d)v shape in enamel wit apex in proximal surface and v shape in dentin with apex tiward the dej

a)v shape in enamel with apex toward Dej and v shape in dentin with apex toward pulp

where is the gold directed on an MO onlay spruce a, faces pulpal axial line angle b. occlusal floor c. pulpal floor d. gingival floor

a, faces pulpal axial line angle

Which of the following is the most likely diagnosis for a tooth with a cavity floor that is hard and leathery with a very dark appearance? Chronic caries Aggressive caries Severe chronic periodontitis Acute caries Localized caries

a- Lesions with hard, leathery cavity floors and very dark in appearance are most likely diagnosed as chronic carious lesions. These lesions occur as a result of longstanding caries involvement and affect a fewer number of teeth and are smaller in size. Tertiary dentin formation occurs in these cases of chonic caries. Pulp prognosis is very good in cases of chronic caries. Acute and aggressive caries is a rapidly occurring disease process that typically involves a greater number of teeth with light brown-to-grey color and caseous (cheese-like) consistency.

The structures of enamel that are more resistant to the actions of acids are: a. Enamel cuticles b. Enamel lamellae c. Enamel rods d. Interprismatic substance of enamel e. Ameloblast

a- enamel cuticle

amelogenesis imperfecta is a a- hereditary ectodermal defect b- hereditary mesodermal defect

a- hereditary ectodermal defect

Failure of the tuberculum impar to retract prior to fusion of the lateral halves of the tongue results In: a. Median rhomboid glossitis b. Cleft (bifid) tongue c. Geographic tongue d. Scrotal tongue e. Sarcoma of tongue

a- median rhomboid glossitis

oral radiograph show a diffuse widening of the periodontal ligament and the enlarged space is uniform in width due to increase in collagen fibers seen in a- scleroderma b- osteosarcoma

a- scleroderma (osteosarcoma abnormal , symmetric widening of the pdl)

pneumococcal vaccination indicated in a- sickle cell anemia b- polycythemia c-agranulocytosis d-acute lymphoblastic leukemia

a- sickle cell anemia

It is necessary to avoid the ingestion of tetracyclinesbetween the age groups from a) 4months to 7 years b) 1 year to 3 year c) 4month to 10 month d) 7 month to 5 year

a-4 months to 7 years

the congenital epulis of the new born usually appears on the a-ant maxillary gingiva b- ant mandibular gingiva c- lips d-tongue

a-ant maxillary gingiva

The classic triad of Hand-Schuler-Christian disease Includes lesions of bone, exopthalmos and: a. Diabetes insipidus b. Hepatosplenomegaly c. Diabetes mellitus d. Albuminuria e. Hypothyroidism

a-diabetes insipidus

A pear-shaped radlolucency causing displacement of the roots of vital maxillary lateral and cuspid teeth Is characteristics of: a. Globulomaxillary cyst. b. Apical periodontal cyst. c. Primordial cyst. d. Nasoalveolar cyst e. Lateral Periodontal Cyst

a-globulomaxillary cyst

Which of the following cyst arises from glands of Serres a) odontogenic keratocyst b) dentigerous cysts c) radicular cysts d) ) paradental cyst

a-odontogenic keratocyst(dental lamina rests /glands osf serres)

Features of familial multiple neurofibroma with café-au-laft spots of the skin are typical of: a. Von Recklinghausen's disease of skin b. Peget's disease of skin c. Hereditary ectodermal dysplasia d. Familial fibrous dysplasia e. Papillion Lefevre Syndrome

a-von recklinghausen's disease of skin

Extraradicular plaque in an infected tooth. What should the Dentist do? a. Mechanochemical irrigation and debridement of the canal b. Do surgical Endo(apicoectomy).

a. Mechanochemical irrigation and debridement of the canal

8yrs child has trauma to 21 teeth with pulpal involvement. What is the immediate action a. Pulpotomy b. Apexification and complete RCT later c. Extraction d. Do nothing

a. Pulpotomy

imbibition and syneresis affect which one the most a. reversible hydrocolloid b. impression compound c. polysulfide d. silicone

a. reversible hydrocolloid

Periostat

acc-fda Periostat 20 mg twice daily as an adjunct following scaling and root planing may be administered for up to 9 months. Periostat should be taken twice daily at 12 hour intervals, usually in the morning and evening. It is recommended that if Periostat is taken close to meal times, allow at least one hour prior to or two hours after meals.

patient has pain without swelling or fever, which pain medication: acetamenophin (tylenol) tylenol 3 (acetamenophin + oxycodone 30mg) codeine

acetamenophin (tylenol)

which of the following is the predominant organism causing root caries? strep.mutans strep.sanguis actino.viscosus lactobacillus

actino.viscosus

which drug administrated when the pt is still on alchoholic beverage? activated charcol, glibenclamide, chloral hydrate, cefexime

activated charcol

The activator for autopolymerizing resin is likely to be heat. hydroquinone. tertiary amine. benzoyl peroxide. glycol dimethacrylate

activator is tertiary amine and initiator is benzoyl peroxide.

patient with recent history of nephritis complains in pain , fever, headache. What treatment should you prescribe for pain: a)diclofenac b)meloxicam c)celecoxib d)tolmentin e)aspirin

acute interstitial nephritis following treatment with celecoxib

null cells are feature of

acute lymphoblastic leukemia

auer rods seen in

acute myeloblastic leukemia

Guidelines for prescribing xray is are determined by Ada Fda Osha Two of them All

ada and fda

most durable type of amalgam?

admixed?

when ugly duckling stage close?

after canine eruption 11-12 yrs

:cracked tooth, definitive diasgnosis by (select all that apply) 1-tooth slot 2-visual 3-local pressure on tooth 4-transillumination

all

Disease with retardation ? hurler syndrome edward syndrome chat syndrome

all

Techniques used to increase width of attached gingiva? 1. Free gingival autograft 2. Free CT autograft 3. Apically positioned flap 4. All

all

which of the following agents causes plasma cell gingivitis a.mint candy b.cinnamon gum c.spicy pepper d.herbal tooth paste select all that apply

all

Advantage of distraction osteogenesis over ostectomy . -better patient compliance, wider -range movements, less relapse and can be done in yonger -patients

all true Advantages of distraction osteogenesis over conventional osteotomies which are often mentioned are application at a younger age, lower risk of nerve damage, the displacement of the jaw or parts of the jaw over a greater distance, better stability of the treatment result, no need for bridging bonegrafts, lower surgical risk and shorter surgery time.

-3 walls vertical defect is called intrabony defect(all vertical defect) -2 walls intrabony defects are the osseous craters(what is it called?) -1 wall vertical defect is called hemiseptum. t/f

all true One wall defect - usually only one interdental wall remains and is called hemi septum if remaining wall is proximal. Two wall defect - most prevalent bone defect found interdentally with facial and lingual walls remaining, involves both the interproximal walls which are mainly called crater defects or interdental crater defects Three wall defect - occurs most frequently in the interdental region, usually the remaining bony walls are facial, lingual and proximal can be circumferential defects

Periosteal New bone formation or neperiostitis is associated with: A) Infantile Cortical Hyperostosis B) Ewings Sarcoma C) Syphilis D) Hipervitaminosis A

all-Other diseases which present with similar features to Garre's osteomyelitis are infantile corticalhyperostosis (Caffey's disease), hypervitaminosis A, syphilis, leukaemia, Ewing's sarcoma and metastatic neuroblastoma

scrap amalgam stored in ? sealed container metal container ethyl chloride

amalgam scrap should be stored under water that contains sodium thiosulphate(photograhic fixer) sulfur /glycerine

how to differentiate between odontogenic myxoma and amelooblastoma in X Ray,

ameloblastoma---which are classically expansile "soap-bubble" lesions, with well-demarcated borders and no matrix calcification. Occasionally erosion of the adjacent tooth roots can be seen. When larger it may also erode through cortex into adjacent soft tissues. odontogenic myxoma--- multilocular radiolucencies with ill-defined borders, though unilocular cyst-like tumors can occur, especially when associated with impacted teeth.

when is amine responsible for discoloration and when is bis-gma responsible for discoloration in resins?

amines-redox reaction responsible for discoloration Bisgma is resin and its mainly long time after and due to microlekage

A preparation for a Class II amalgam restoration in primary molars should include undercut enamel and dentin. an isthmus that occupies two thirds of the intercuspal distance. occlusally divergent buccal and lingual walls. an axial wall that parallels the dentino-enamel junction.

an axial wall that parallels the dentino-enamel junction.

Beckwith-Wiedemann syndrome

an overgrowth disorder usually present at birth, Common features used to define BWS are: macroglossia (large tongue), macrosomia (above average birth weight and length), midline abdominal wall defects (omphalocele/exomphalos, umbilical hernia, diastasis recti), ear creases or ear pits, neonatal hypoglycemia (low blood sugar after birth). Hepatoblastoma

trauma from oclussion Angular bone loss Hztal bone loss Vertical boneloss

angular bone loss or buttressing bone according to carannza pg 386

C-factor : Is related to cavity preparation geometry Is represented by the ratio of bonded to non-bonded surface areas Minimizing C-factor reduces polym- erization stress All of the above

ans -D(One technique attempts to reduce the so-called C-factor (configuration factor). The C-factor is related to the cavity preparation geometry and is represented by the ratio of bonded to nonbonded surface areas. Residual polymerization stress increases directly with this ratio. During curing, shrinkage leaves the bonded cavity surfaces in a state of stress and the nonbonded, free surfaces (i.e, those that reproduce the original external tooth anatomy) relax some of the stress by contracting inward toward the bulk of the material. A layering technique in which the restoration is built up in increments, curing one layer at a time, effectively reduces polymerization stress by minimizing the C-factor. That is, thinner layers reduce bonded surface area and maximize nonbonded surface area, thus minimizing the associated C-factor.)

All of the following are possible effects of aspirin EXCEPT a. Reduction of fever b. Shortening of bleeding time c. Suppression of inflammatory response d. Bleeding from the gastronintestinal tract e. Increase in the renal excretion of uric acid at high doses This question is answered wrong in TUFTS

ans B(Aspirin in high doses increases renal excretion of uric acid.)

how many international units are in 125 mg of pencillin G? 100000 200000 300000 400000 500000

ans is 200000 international Standard for Penicillin as containing 1,670 International Units (IU) per mg,

Dry socket which of the following not associated with it Infection 3 days post- treatment pain Refer pain to ear Bad Odour

ans is infection check in mosby

In which of the following, the center of mandible and facial midline coincide in rest position? A. True cross bite B. Laterocclusion

ans- B during mandibular closure , the midline of mandible is observed .in case of unilateral crossbite , this analysis is relevant to differentiate between laterognathy and laterocclusion. laterognathy or true cross bite - the center of the mandible and facial midline do not coincide in rest and in occlusion. laterocclusion- the center of the mandible and facial midline coincide in rest poistion but in occlusion the midline deviates due to tooth interface leading to noncoinciding midlines.

exogenous intrinsic stains examples?

ans- amalgam, topical fluoride, tobacco stains,plaque ref perio dd 99

Chronic nasal stuffiness assoc with? Class 3 Class 2

ans- class 3 Class Severity and duration† Symptoms I Mild, intermittent Nasal rhinorrhea Sneezing Itchy, watery eyes II Moderate, intermittent Moderate-severe, intermittent Severe, intermittent ,Mild, persistent More severe than those for class I III Moderate, persistent Constant nasal stuffiness Severe, limiting quality of life IV Moderate-severe, persistent Severe, persistent Severe ongoing nasal stuffiness Congestion Sinus drainageans-

which of the following can usually be identified by a radiographic means alone a- cementoma apical cyst c- mental foramen d-chronic apical periodontitis e-all the above

ans- mental foramen cause its a antomical portion that can be confirmed by slob technique.

the symptom complex which includes pain in lower jaw, tongue and side of the head , and middle ear deafness is called. horton's syndrome eagles syndrome trotters syndrome forthergills disease

ans- trotters syndrome this symptom complex caused by nasopharyngeal tumor, has been called trotter's syndrome.

a RG was taken with 90KVp,15mA,8 inch distance (PID), with exposure time 0.5sec.. what is the exposure time if PID is 16 inch and rest of the parameters are same a)0.25sec b)1sec c)2sec d)4secs

ans-2sec PID = X Exposure time =X Now Suppose the PID = 2X than Exposure time = 4X

What main reason implants fail? A) Surgical error B) Lack of early loading , C) Inadequate occlusal design

ans-A

Pt has a cranial dyostosis, midface def and hypertelorism A. Crouzon B. Treacher collins

ans-A Treacher Collins syndrome is a genetic disorder that most often affects the cheek bones, jaw, chin, and ears.

Same recepter Phsiologic antagonism Competitive antagonist

ans-A competitive antagonist is a receptor antagonist that binds to a receptor but does not activate the receptor. The antagonist will compete with available agonist for receptor binding sites on the same receptor (Physiological antagonism describes the behavior of a substance that produces effects counteracting those of another substance (a result similar to that produced by an antagonist blocking the action of an agonist at the same receptor) using a mechanism that does not involve binding to the same receptor.)

Suppuration is mainly the result of the combined action of four factors; which of the following is not one of these factors A. Necrosis B. Presence of lymphocytes C. Collection of neutrophils D. Accumulation of tissue fluid E. Autolysis by proteolytic enzymes

ans-C explanation---Neutrophils constitute the "first line of cellular defense" against invading organisms and particulate material. They are the first to gather in acute inflammation. Neutrophils are seen in response to pyogenic organisms and are the principal constituent of pus. Remember, the number of neutrophils in the circulation increases greatly in the early stages of inflammation. (1) phagocytosis, (2) production of proteolytic and lipolytic enzymes to digest bacteria, dead cells, etc., (3) they may produce substances which neutralize the toxic products of bacteria and (4) they may act as an energy source for other cells (there is evidence that neutrophils shed their cytoplasm which is then transferred to "mononuclear cells" as an energy source). Lymphocytes appear late in inflammation. The arbitrary time of 48 to 72 hours is usually given as the time of appearance. They are seen as a prominent part of long-standing inflammatory reactions. chronic inflammation: Mononuclear cells (monocytes, macrophages, lymphocytes, plasma cells), fibroblasts suppuration is chronic infection 1. The formation or discharge of pus. 2. Pus.

In regards to HIV infection, which of the following is the earliest finding A. Kaposi sarcoma on the palate B. Reduced haemoglobin C. Infection with pneumocystic carinii D. Reduction in white cells count E. B cell lymphoma

ans-D CD4 cells are a type of white blood cell that fights infection. Another name for them is T-helper cells. CD4 cells are made in the spleen, lymph nodes, and thymus gland, which are part of the lymph or infection-fighting system. CD4 cells move throughout your body, helping to identify and destroy germs such as bacteria and viruses. The CD4 count measures the number of CD4 cells in a sample of your blood drawn by a needle from a vein in your arm. Along with other tests, the CD4 count helps tell how strong your immune system is, indicates the stage of your HIV disease, guides treatment, and predicts how your disease may progress. Keeping your CD4 count high can reduce complications of HIV disease and extend your life.

Factors responsible for minimum permanent deformation of an elastomeric impression is : A material has adequately gelled Minimum undercuts Rapid removal of impression along path of tray insertion All of the above

ans-D(The amount of permanent deformation exhibited by an elastomeric impression material should be clinically negligible, provided that : (1) the material has adequately gelled, (2) negligible pressure is applied to the tray during polymerization, (3) the impression has been removed rapidly along the path of tray insertion, and (4) the undercuts present in the cavity preparation are minimal.)

What does the Weight and height stand for in recordings? ordinal nominal ratio interval

ans-Interval scale uses a scale graded in equal increments.In the scale of length,we know that one inch is equal tp any other inch. Interval scale allow us to say only that two things are different,but by how much.If a measurement has a mean and a standard deviation,treat it as an interval scale.It is also called a "numerical scale." Eg- Height Weight Blood pressure nominal scale is exclusive and exaustive .like a social security number ; ordinal scale has rank order . from small to big like cell ,tissue ,organ body interval scale like body weight , etc , they r additive Ratio scale has a 0 starting point .eg Kelvin temperature scale .Interval scale uses a scale graded in equal increments.In the scale of length,we know that one inch is equal tp any other inch. Interval scale allow us to say only that two things are different,but by how much.If a measurement has a mean and a standard deviation,treat it as an interval scale.It is also called a "numerical scale." Eg- Height Weight Blood pressure nominal scale is exclusive and exaustive .like a social security number ; ordinal scale has rank order . from small to big like cell ,tissue ,organ body interval scale like body weight , etc , they r additive Ratio scale has a 0 starting point .eg Kelvin temperature scale .

Which has best prognosis Endo from perio Perio from endo

ans-Perio from endo because-In primary endodontic lesions with secondary periodontal involvement, the root canals contamination occurs because of the caries process, traumatic lesions and coronal microleakage Pulp inflammation or necrosis leads to an inflammatory response in the periodontal ligament, apical foramen, and the underlying alveolar bone. Clinically, it is presented as deep and localized periodontal pockets extending mostly up to the tooth apex, resulting in a localized diffuse swelling.

Not a risk factor for ANUG? a) plaque b) diabetes c) smoking

ans-b Risk factors for ANUG incudes-- Immunocompromised state (HIV,etc) Malnutrition Mental stress Poor oral hygiene (Plaque) Smoking Untreated gingivitis or periodontitis

Injury that results in pulp necrosis Avulsion Intrusion

ans-b(Highest to lowest in severity) Intrusion - a VERY damaging injury, massive crushing of the PDL - poorest prognosis. Intrusions are almost always result in pulpal necrosis Avulsion - we only have very small areas of damage. Where the PDL is torn there IS the potential for complete healing. But we have a very small window of opportunity. If the tooth is not treated properly, the PDL cells then become strong inflammatory stimuli and this causes root damage with resultant "unfavorable healing". Avulsions always result in pulpal necrosis. Lateral Luxation Extrusive Luxation Subluxation Concussion

Study how to do your find causation?? 1-analytic 2-cross section 3-cohort 4-case control

ans-c Case control studies have a number of drawbacks. They cannot show causation, as mentioned; nor can they provide information on incidence (e.g. what percentage of people have a condition). As well, the information collected can be faulty or incomplete because it depends on people accurately and truthfully recalling their past.

Which of the following diagnoses represents an injury that causes the tooth to loosen but is not displaced? A. Concussion B. Luxation C. Subluxation D. Extrusion E. Intrusion

ans-c Concussion- The tooth is not mobile and not displaced. 2. Subluxation- The tooth is loosened but not displaced. 3. Lateral Luxation- The tooth is displaced in a labial, lingual, or lateral direction. 4. Intrusion- The tooth is driven into its socket. 5. Extrusion- Central dislocation of the tooth from its socket. 6. Avulsion- Tooth is completely displaced from the alveolus.

Your patient presents with the partiality edentulous arch. Tooth #28 has the survey line and undercut , there is no contraindication to any type of class or to any rest location . what would be your first choice for the class assembly on tooth #28 1) distilled rest and cast circumferential clasp 2) distal rest and I bar clasp 3) distal rest and modified T bar clasp 4) distal rest and wrought wire circumferential clasp

ans-c Undercut is Adjacent to edentulous area and modified T-bar is an excellent choice. t bar acts as a tripping action and this clasp is most commonly used as canine and premolar abutments, it engages the undercut.

Which of the following is NOT true about furcation involvement? 1. Classify based on horizontal measurements 2. Most furcation problems can be dealt with using srp 3. None

ans-c vertical is from floor of the furcation to the roof of the furaction.it is given by fletcher in class a,b,c horizontal is attachment loss in the furcation.by glickman in grade

cavernous sinus thrombosis. This complication involves which of the following cranial nerves? A. IV, V, VII, IX B. II, III, IV, X C. IV, VI, VII, VIII D. III, IV, V, VI

ans-d

All of the following bacteria are considered intermeditate-to-late supragingival plaque colonizers EXCEPT one. Which one is the EXCEPTION? Aggregatibacter actinomycetemcomitans Streptococcus sanguinis Streptococcus salivarius Porphyromonas gingivalis Lactobacillus casei

ans-d The formation of dental plaque consists of three distinct stages: Formation of pellicle on the tooth surface Initial colonization by bacteria Secondary colonization with plaque maturation Within a few seconds after cleaning teeth, a thin layer called pellicle is formed on the tooth surface by the saliva, which is then colonized by bacteria within a few hours. The initial colonizing bacteria is collectively known as supragingival plaque and predominantly consists of gram-positive facultative microorganisms: Actinomyces viscosus Streptococcus sanguinis

if increase distance of the tube from receptor by 2- intensity is---? and the exposure time is ----?

ans-intensity will be decreased by 1/4 times so exposure time has to be increased by 4 times (mA) × length of exposure time (S) = mAs • Increasing mAs = increase in primary signal intensity • Decreasing mAs = decrease in the primary signal intensity Source image distance, the "Inverse Square Law" has an inverse relationship with the intensity of an x-ray beam: Old mAs Setting × New SID² = New mAs Setting Old SID² • Increasing the distance = decreasing in the beam intensity at the receptor • Decreasing the distance = increasing the beam intensity at the receptor

codman's triangle is a feature of osteoblastoma osteosarcoma ameloblastoma ewings sarcoma

ans-osteosarcoma in the long bonesaffected with osteosarcoma, the periosteum is elevated over the expanding tumor mass in a tent like fashion. at the point on the bone where the periosteum begins to merge , and acute angle between the bone surface and the periosteum is created , this is called codman's triangle and is highly suspicious for osteosarcoma.

to differentiate acute and chronic apical periodontitis? radiology features histology features percussion thermal test

ans-radiology features( chronic is percussion negative) Acute is sensitive on percussion-In acute apical periodontitis there is minor inflammation of the periodontal fibers and ligaments producing symptoms like tenderness on percussion , biting and chewing pain as disease progress to chronic apical periodontitis , The initial symptoms disappear as now there will be no pain on percussion nothing , and we can see PDL widnening along with radiolucency at the apical region due to resorption of cortical bone and further infection ...So radiographically we can differentiate and very well appreciate

Bifurcation involvemnnrt 4.5 mm pocket.tx Srp Curretagr Rct Rct and periodontl tx

ans-rct and periodontl tx Pulpo-periodontal disease The high percentage of molar teeth with patent accessory canals opening into the furcation suggests that pulpal disease could be an initiating cofactor in the development of furcation involvement. If there is no established periodontal furcaion involvement, these pulpal lesions are initially pure endodontic sinus tracts draining through the periodontal ligament and gingival sulcus. If detected and treated early by endodontic therapy, these furcation defects resolve with regeneration of new interfurcal bone and attachment. If not detected and treated before plaque and calculus are allowed to form on the root surface adjacent to the endodontic therapy, these furcation defects resolves with regeneration of new interfurcal bone and attachment.

width of upper and lower permanent incisors in a year old child is measured at 23mm and 22mm respectively, the estimated width of unerupted canine and premolars 22.5 and 24.5 respectively. T/F how do you calculate it?

answer- moyer's analysis predicted width of maxillary canine and premolar=sum of mandibular incisor+11/2 predicted width of mandibular canine and premolar=sum of the mandibular incisor+10.5/2

OSHA stander for Mercury 0.05 mg cubic meter for 4 hrs 0.5 mg cubic meter for 4 hrs 0.1 mg cubic meter for 4 hrs 1 mg cubic meter for 8 hrs

answer-A Or 0,1 for 8 hrs

Acquired Immune Deficiency syndrome (AIDS) may be characterized by 1. candidiasis 2. rapid weight loss and night sweats 3. extreme malaise, fever and chills 4. a smooth and red tongue a. 1, 2 & 3 b. 1 & 3 c. 2 & 4 d. 4 only e. all of the above: in the early stage, a periapical abscess can be differentiated from a lateral periodontal abscess by a. pain b. type of exudate c. tenderness to percussion d. response of pulp to percussion e. radiographic examination

answers are e & c

A daily dose of 81 mg of acetylsalicylic acid is used for its analgesic properties. antipyretic effect. anti-inflammatory function. antiplatelet action.

antiplatelet action.

Which Antibiotic for AnuG and LAP and in chronic do we give Antibiotics?

anug pencellin and mitanidazol lap pencilline chroninc no need

st johns wort used for?

anxiety, mild to moderate depression, stomach upset, insomnia, fluid retention, and hemorrhoids.

________unpleasent negative state without identifiable cause

anxiety-unpleasant negative emotional state without identifiable cause, feeling of apprehension, uneasiness, agitation, or uncertainty resulting from anticipation of a threat of danger whose source is unknown

Which is most successful in each of the following (CaOH , MTA) Apexification? Apexogenisis?

apexification procedure with MTA. apexogenesis is calcium hydroxide USP (CaOH)

Patient comes in and they say "oh I hate the dentist, I hate being here" What would be your response

ask the pt what makes him think so and ask him about his past dental experiences

aspirin- irreversible or reversible

aspirin has irreversibly inhibited the enzyme

At what age should all speech disorders be fixed?- A) 8 B) 6 C) 11 D) 7

at 8 years

What is Endotec II used for? Warm vertical compaction Warm lateral compaction Thermoplastic GP injector None

ateral compaction of gutta-percha provides for length control, which is an advantage over thermoplastic techniques. The Endotec II device (Medidenta) provides the clinician with the ability to employ length control while incorporating a warm gutta-percha technique. Investigators demonstrated that the Endotec II produced a fusion of the gutta-percha into a solid homogeneous mass. One study evaluated three thermoplasticized filling techniques and lateral compaction, using a bacterial metabolite model, and found the Endotec to be superior to lateral compaction alone, lateral thermocompaction, and the Ultrafil 3D.The use of warm lateral compaction with the Endotec demonstrated an increased weight ofheat generated by the technique. Evaluation of the effects of warm lateral and warm vertical compaction on periodontal tissues demonstrated that neither technique produced heatrelated damage. The warm lateral compaction technique involves adapting a master cone in the same manner as with traditional lateral compaction. An appropriate-size Endotec II tip is selected. Endotec II tips are available in various taper and tip diameters. The sizes consist of #.02/20 and #.02/40. The device is activated and the tip is inserted beside the master cone to within 2 to 4 mm of the apex, using light pressure. The tip is rotated for 5 to 8 seconds and removed. An unheated spreader can be placed in the channel created to ensure adaptation and then an accessory cone is placed. The process is continued until the canal is filled.

Frey syndrome ,Which nerve does it affect ? How is it diagnosed ?

auriculotemporal nerve, diagnosed with iodine.

azithromycin dose erythromycin dose

azithromycin- adults 500mg/day for 3 days children- 10mg/kg/day for days erythromycin- adults-500-1000mg/6 hours children-50mg/kg/day in 3doses

Gingival retraction cord is placed ______ crown prep is completed and is removed _____ final impression taken. A. After, after B. After, before

b

Making promises to patients A) are not necessary when there is a sound evidence base for what the dentist does B) create relationships C) is only valid when others know you are honest D) should be avoided when possible

b

UFDBA is osteoconductive DFDBA is osteoinductive which one mostly use and y Bone graft causing ossteogenesis A.FDBA B.DFDBA

b

Which of the following describes when caries can be diagnosed accurately without radiographs? When only primary teeth are present No answers are correct All answers are correct When all posterior teeth have proximal restorations When the patient is pregnant

b

There is a mandibular angle fracture. The distobuccal segment displaces by muscles a. Inferiorly and laterally b. Medially c. Anteriorly d. Inferiorly and mediall

b medially

in a well balanced face the ratio between upper and lower facial height should be? a) 3:4 b) 5:6

b) 5:6

Which of the following is least affected by occlusal trauma? a) Alveolar bone. b) Gingival attachment. c) Periodontal ligament. d) Cementum.

b) Gingival attachment.

Distractive osteogenesis advantage over BSSO: a) Less relapse b) Large movements c) Esthetics

b) Large movements(bilateral sagital splunt osteotomy)

Conditioning material of resected root Citric acid EdTA

b- 50% CITRIC-ACID-Ph,(not commonly used) 15-24% EDTA-Ph 7.3(best) Tetracycline Hcl-Ph 1%

Anesthetic testing is most effective in localizing pain to which of the following? A/specific tooth B/mandible or maxilla C/across the midline D/posterior teeth

b- Because an anesthesia can give several teeth so if 5 has a pain for example and you anesthesized the 4 the pain on 5 will disappear not because it's not painful but because it's anesthesized. Because refered pain do not cross the midline. But can be refered between upper and lower max.

When treating avulsed tooth (1hour), you would stabilize and perform pulpectomy and place CaOH, what is the reasoning behind it? 1) CaOH is used to prevent cervical resorption 2) splint for 1-2 weeks and pulpectomy with CaOH

b- If replanted in less then 2 hour, you do not perform pulpectomy right away. Has to be done 10 day after and CaOH is placed to promote secondary dentin formation.

Burning mouth syndrome is associated with: A. Keratinocytes with enlarged nuclei on cytologic smears B. Clinically normal-appearing mucosa C. Clinically erythematous, atrophicappearing mucosa. D. Candidal pseudohyphae on cytologic smears

b- clinically normal apearing mucosa in post menopausal woman mostly.

Cyst resembling the bunch of grapes is a) Gingival cyst b) developmental lateral periodontal cyst c) dentigerous cyst d)okc

b- developmental lateral periodontal cyst also known as botryoid odontogenic cyst

Low occlusal plane leads to what? A.decrease biting force B.tongue biting C.excessive biting force

b- low occlusal plane in a CD fabrication occurs when d mandibular occlusal rim height is way less or maxillary rim height is too larger than normal,this way occlusal plane shifts inferiorly to d normal position...in these cases tongue remains above d occlusal plane n on biting comes b/w d two arches causing a tongue bite...normally/in a correctly fabricated denture the dorsal surface of the tongue shud remain in level with d occlusal surfaces of the posterior teeth...

which oral medication you use to supplement the sedation in children hydroxysine diazepam

b-Actually Midazolam is used in children

Chemical composition of COPALITE: organic resin chloroform solvent acetone solvent alcohol solvent

b-Copalite Varnish is applied as an insulating layer under gold and amalgam restorations.

which is used for prophylaxis of migraine? carbamazepine topiramate ethosuximide phenobarbitone

b-First-line therapies for migraine prophylaxis in adults include propranolol (Inderal), timolol (Blocadren), amitriptyline, divalproex (Depakote), sodium valproate, and topiramate (Topamax).

The office bleaching changes the shade through all except... a. dehydration b. etching tooth c. oxidation of colorant d. surface deminearalization

b-In-office bleaching changes the shade via (dehydration, oxidation of colorant, and surface demineralization), but NOT by etching the tooth.

The office bleaching changes the shade through all except... a. dehydration b. etching tooth c. oxidation of colorant d. surface deminearalization

b-In-office bleaching changes the shade via (dehydration, oxidation of colorant, and surface demineralization), but NOT by etching the tooth.

Resin retained fpd everything necessary except Wrap around proximal Draw Occlusal rest Chamfer Bevel

b-Rbfp needs 180 degree of wrap around,it needs a finish line since it's metal wing so chamfer needed,occlusal rest is needed for molars to get support from abutment and bevel is needed in functional cusp if occlusal surface of abutments alone taken for support. So draw is the answer.

Which of the following is the most common iatragenic side effect of sodium hypochlorite? Reaction with chelating agents Toxicity to vital tissue Corrosiveness to endodontic files Tendency to bleach tooth structure Smell

b-Sodium hypochlorite in undiluted or excessive amounts is toxic to vital tissues and can cause periapical tissue damage and pain. Sodium hypochlorite is a good disinfectant that is used in dentistry as an endodontic canal irrigant at diluted concentrations. Sodium hypochlorite is often used to disinfect gutta percha points prior to placement and obturation of the canal.

When class 3 elastics are used, the maxillary molars will A) move distally and intrude B) move mesially and extrude C) move mesially and intrude D) move only mesially , there will be no movement in the vertical direction

b-Sure, class III elastics extends from max 1st Molar to mnd canine. The force of the elastic causes mesial movement of mx 1M and distal movement of mnd canine. When the molar is moving mesially, it causes flaring of the upper incisor. On the other hand, there is extrusion of upper 1st molar as the elastic is pulling the tooth down .

A carious lesion extending from the distal pit to the central fossa along the central groove of #19 is detected clinically and is confirmed radiographically. The planned amalgam preparation and restoration should A. span the entire occlusal surface, along the central groove, including both the mesial and distal marginal ridges. B. extend from the distal pit to the central fossa along the central groove. It should also include any enamel undermined by the removal of carious tooth structure. C. extend from the distal pit to the mesial pit, along the central groove. Buccal and lingual extensions should be prepared as well, from the central fossa along the buccal and lingual grooves. D. extend from the distal pit to the mesial pit, along the central groove.

b-The amalgam preparation and restoration for a carious lesion extending from the distal pit to the central fossa along the central groove should extend from the distal pit to the central fossa along the central groove. It should also include any enamel undermined by the removal of carious tooth structure. The depth of the preparation should be at least 1.5-2mm in order to provide strength to the restoration. The pulpal floor, depending upon the enamel thickness, is almost always in dentin.

Which of the following describes the most common reason ledges are formed during endodontic procedures? Too large of file too fast Unachieved straight line access Improper irrigation Fallen pulp stone in canal

b-Unnecessary deflection of the endodontic file may cause a loss of instrument control and subsequent ledge formation. Attempts to clean and shape the canal without proper straight line access often induce ledging, transportation, and zipping perforations. The goal of straight line access is to remove coronal tooth structure to allow a straight canal for maximum freedom for the instrument in the more confined, difficult apical sections. Most endodontic cases are ledged because the clinician is attempting to guide the instrument through too many curves.

Features of multiple bone radiolucencies, hypercalcemia, hypophosphatemla, and loss of lamina dura and indicative of: a. Acromegaty b. Hyperparathyroidism c. Hypothyroidism d. Multiple myeloma e. Osteitis deformans

b-hyperparathyroidism

You have a Distal extension RPD and it rocks along the fulcrum, on both sides of the axis? 1. The denture base is not well adapted 2. Indirect retainers are not that retentive 3. Something about the rests

b-if it rocks around fulcrum A-if one side is rocking

Which of the following statement is incorrect about dental caries a)Caries prevalence increases when populations become exposed to sucrose-rich diet b)intrinsic sugars are more damaging than extrinsic sugars c)Sucrose is the most cariogenic sugar d)Frequency of sugar intake is of more importance thantotal amount consumed

b-intrinsic sugars more damage than extrinsic sugards

Pt smoking 15 -20 cigarettes a day for 8 years he came with complain of pain in gingival Diagnosed as chronic gingivitis , so: a-gum shows inflammation as former smoker b-less intensity than unsmoker**** c-more intensity than non-smoker d-same as non-smoker

b-less intensity than unsmoker (known as "disease masking". What happens is that the diseased tissue of smokers tends to have a firmer appearance and less bleeding. The vasoconstrictive property of tobacco smoke tends to "mask" the inflammatory and destructive changes occuring within the periodontium; therefore, there would be "less intensity" in apperance as compared to non-smokers.)

When class III elastics are used, the maxillary first molars . A. Move distally and intrude B. Move mesially and extrude C. Move mesially and intrude D. Move only mesially; there is no movement in the vertical direction

b-move mesially and extrude

When pins are included in an amalgam cavity preparation, the strength of the amalgam is: A. Increased B. Decreased C. Unchanged

b-retention is enhanced but the presence of the pins means interruption of the amalgum bulk which cause its decreasing in strength

The Melkersson-Rosenthal syndrome is characterized by facial paralysis, Chellitis granulomatosa, and: a. Black Hairy tongue b. Scrotal tongue c. Geographic tongue d. Bifid tongue e. White Hairy tongue

b-scrotal tongue

Most common cause of fracture at the isthmus of a class II restoration is A.delayed amalgam expansion B.a sharp axio-pulpal line angle C.isthmus width too narrow D.moister contamination of the amalgam during placement E.lack of interproximal contact

b-sharp axiopulpal line angle is the FIRST most common, Second is inadequate depth of the isthmus not width

Plaque accumulation has a direct effect on which of the following? A. The level of materia alba B. The severity of gingivitis C. The accumulation of calculus D. The severity of periodontitis

b-the quantity of the plaque is related to the severity of gingivitis. BUT the quality is related to severity of periodontitis

There is a mandibular angle fracture. The distobuccal segment displaces by muscles a. Inferiorly and laterally b. Medially c. Anteriorly d. Inferiorly and mediall

b. Medially(the # is at the angle but the muscles at the area all your closing muscles So a #at the proximal segment which is the DB segment should displace it in the closing direction i.e sup and medial )

Which of the following is the most likely cause of pulpal necrosis following trauma to a tooth a. Calcific metamorphosis b. Pulpal hyperemia

b. Pulpal hyperemia

Erythromycin should be avoided in the patient taking a. Aspirin b. Seldane c. Benadryl d. Ibuprofen e. Propranolol

b. Seldane

1. Which of the following may result from an overdose of insulin? a. glycosuria b. convulsions c. hyperglycemia d. diabetes insipidus

b. convulsions Insulin overdose results in hypoglycemia. Insulin/diabetic shock. Unconsciousness, seizures etc

Which of the following teeth have the most consistent canal morphology a. mand. incissors b. max canines c. mand. premolars d. max premolars e. mand. molars

b. max canines

One of the mechanisms of bacterial adherence to the dental pellicle is through A. positively charged bacteria with the pellicle. B. interaction of cations such as calcium. C. hydrophilic interactions. D. interaction of salivary anions.

b.Interactions of salivary cations like calcium, potassium, phosphorus. The majorly predominant one is calcium. When it attaches to the negatively charged enamel (due to overwhelming negative charge of hydroxyapatite in enamel), it first neutralizes the tooth surface's negativity and subsequently makes it positively charged. Positive charged tooth surface attracts proteins and other macromolecules that makes bacteria adhere to them via lock and key mechanism.

Why is erythromycin contraindicated with aspirin?

because erythromycin alters cytochrome p450 enzyme so other medications metabolized in liver will be effected

Composite compare to gic has better - good translucency, better wear

better wear

Thumb Sucking Unilateral cross bite Bilateral cross bite

bilateral posterior crossbite and mild anterior open bite,v shaped palate

atridox?

biodegradable controlled release gel containing the doxycycline deliver via syringe system to the pocket

Which porcelain stain colour is added to give the appearance of translucency? A. Grey. B. Blue. C. Yellow. D. White. E. Orange

blue

Nevus with Tyndall effect ? Most common acquired melanocytic Nevus in oral cavity ?

blue nevi

Who is or are recommand the weekly spore testing Ada Cdc

both

diff bet 245 and 330 bur?

both are pear shaped, 245 is 3mm and 330 is 1.5mm, that's the only difference

In RPD extended oclusal rest are always preferred in : a) Kennedy Class I b)Kennedy Class III c)kennedy class ii

both class 2 and 3 in situations when the most posterior abutment is mesially tipped molar this rest should extend more than one half the mesiodistal width of the tooth , be approx 1/3 the buccolingual width of the tooth. if abutment is severely tilted the extended occlusal rest may be in the form of only.

T or F Biohazard waste : sharps Blood born waste: infectious waste

both true

Aspirin toxicity causes metabolic acidosis or respiratory alkalosis

both-Acute aspirin or salicylates overdose or poisoning can cause initial respiratory alkalosis though metabolic acidosis ensues thereafter.

after perio surgery, the re-attachment can happen a. as soon as in a week b. to the dentin or cementum

both-The junctional epithelium can start forming as early as one week and the attachment can happen to both dentin and cementum

When using forceps to extract a maxillary first molar, the forceps movement should be principally in the buccal direction because the furcation is more accessible from the buccal. risk for sinus perforation is minimized. buccal roots are shorter than palatal root. buccal bone is thinner than the palatal bone.

buccal bone is thinner than the palatal bone.

When you smile what is the black space buccal of teeth and next to cheeks?

buccal corridor

Direction of luxation for extracting mand molars?

buccal for permanent, palatal for primary

intra pulpal anasthesia Produce anasthesia in less than 30 sec. produce anasthesia by pressure

by pressure-The theory behind the Intra-Pulpal injection is to rapidly raise the pulpal pressure beyond the trigger level.

Each of the following is indicated by a high C-factor EXCEPT one. Which is the EXCEPTION? A. More bonded-to-unbonded restoration surface areas B. A higher potential for bond disruption C. A higher Calset temperature D. More stress on the walls of the preparation

c

High velocity gunshot wound causing comminuted fracture, treatment a. Bag of bone ------------------- b. Debridement and immediate load bearing c. Debridement and 2nd stage grafting and fixation

c

Normal range of gingival depth in healthy mouth A. 1-2 B. 2.3 C. 0-3 D. 0-5

c

Propoxyphene contraindicated in pregnant women? A. 1st trimester B. 2nd C. 3rd D. All of the above

c

The cariostatic effect of fluoride is manifested during which of the following stages of tooth development? Proliferation Histodifferentiation Calcification Apposition

c

Tissue conditioner is used for treating a. inflammatory papillary hyperplasia b. Epulis fissuratum c. traumatized mucosa c. rocking denture stabilization

c

When all other removable partial denture consideration remains unchanged; clasps constructed of which material can be engage the deepest under cut A. Chrome cobalt casts B. Nickel chrome casts C. Wrought stainless steel D. Wrought gold

c

Which is least likely to determine future caries a. Previous history of caries and restoration b. Large quantity of fermentable sugars eaten c. How often fermentable sugars are eaten d. Cariogenic bacteria

c

which of the following jaw relation records should be used for setting both the medial and superior condylar guides on a non arcon articulator? a. intercuspation b. centric relation c. Lateral interocclusal record d. Protrusive interocclusal record

c

The dentist accidently did not see Occlusal caries and he placed a sealant over it; Will the caries o Stop o Rapid Progression o Slow Progression

c -by hall tecnique

a patient has a high caries index, short crowns and minimum horizontal overlap. What restoration will you plce a. 3/4 frown b. jacket crown c. PFM d. resin bonded retainer

c is correct(A option eliminated as short crowns,B option short crwns and hrizontal overlap mun are conteaindications and for D nt a choice for High cares index)

"SAUSAGE LINK APPEARANCE" in !!! A. Submandibular gland. B. Sublingual gland. C. Parotid gland

c parotid

a practitioner pickles gold alloy restorations by heating them to redness and plunging them into an acid bath.this precedure can result in which of the following? c warpage of the restoration d surface foughness of the restoration.

c warpage of the restoration

Gingival Fiber conective tissue in implants a) Parallel b) perpendicullar c) parallel with cuff d) perpendicular with cuff

c) parallel with cuff

The distance between junctional epithelium and alveolar bone a)Increases with age b)Decreases with age c)Remains constant d)None of the above

c)Remains constant

A dentist is doing research on 5 unrelated patient with different background. He record data ...... etc.dentist is doing what kind of research? a. clinical trial b. cohort c. sectional

c- Clinical trial evaluates the effects of treatment Sectional sample of people assessed at one time Cohort related to see a specific exposure effects on a population

a Rx was taken with 90KVp,15mA,8 inch distance (PID), with exposure time 0.5sec.. what is the exposure time if PID is 16 inch and rest of the parameters are same a)0.25sec b)1sec c)2sec d)4secs

c- PID = X Exposure time =X Now Suppose the PID = 2X than Exposure time = 4X 0.5 x4=2sec

During canal preparation, excessive instrumentation leads to the loss of the apical seat. The dentist should procede by... Prescribing analgesics and antibiotics Using a chelating agent and packing with calcium hydroxide Increasing the file size and decreasing the file length Filling with a gutta-percha cone that is larger Filling with a warm gutta-percha technique

c- When the apical seat is accidentally removed, the ability to create a hermetic seal at the apex is compromised. In order to avoid this procedural error, a shorter length of file should be used to establish a correct working length. The correction of working length will eventually help re-establish apical seat, then adequate flaring of the apical portion of the canal can be done with the use of larger-sized hand files.

. Inflammation of the lips Is referred to as: a. Stcmataibs b. Glossitis migrican c. Cheilitis d. Vincent Angina e. Sialadenitis

c- chelitis

A constant feature associated with radicular cyst is a) impacted tooth b) missing tooth c) non vital tooth d) anomalous tooth

c- non vital tooth

HERBAL supplement that potentiates anti-coagulation a. St. John's Wort b. Saw Palmetto c. Chamomile

c- potentiate A- inhibit

Which of the following is the most likely cause of pulpal necrosis after trauma to a tooth? A. Ankylosis B. Calcific metamorphosis C. Pulpal hyperemia D. Dilaceration

c- pulpal hyperemia

A cyst occurring under the tongue, caused by obstruction of a salivary gland duct, is called a: a. Follicular cyst b. Dentigerous cyst c. Ranula d. Dermoid cyst e. Epidermoid cyst

c- ranula

PAR seen on asymptomatic tooth, when opened the canal is calcified what do you do: do nothing, refer to endodontist, place EDTA

c- to gain access to calcified canals, which reduce the perforation of canals

Common mole is - A- blue Nevus B- compound Nevus C- intradermal Nevus D- junctional Nevus

c-An intradermal nevus is a classic type of mole or birthmark, with the same degree of pigmentation as the surrounding skin.

Maxillary lateral incisor want to inc width of attached gingiva which flap? A. Pedicle flap B. Lateral flap C. Apical repositioning flap

c-Classification of CrownLengthening Procedures I Gingival reduction only—bone removal not required A Gingivectomy B Gingival flap surgery II Mucoperiosteal flap with ostectomy—bone removal required A One-stage procedures, which require one of the following: (1) Flaps, ostectomy, apical positioning (2) Flaps, ostectomy, gingivectomy, positioning (3) Gingivectomy, flaps, ostectomy, positioning B Two-stage procedure, which requires: Flaps, ostectomy, and repositioning 4 to 6 weeks later—gingivectomy

Each of the following is a criteria when determining the effectiveness of a visible light-curing unit EXCEPT one. Which is the EXCEPTION? A. Wavelength B. Shade of composite C. Fluoride-releasing ability of the composite being cured D. Light exposure time

c-Composite resins do not typically release fluoride and therefore, it is NOT a factor in determining the effectiveness of a visible light-curing unit. The wavelength of the curing light is an important consideration. The normal range of composite curing-light units is approximately 470 nm. The shade of the composite resins also affects the penetration of light required for curing. Exposure time should neither be overextended nor kept short.

porcelain constituents that fuses at firing temperature and forms a matrix is kaolin quartz feldspar aluminium oxide

c-During firing feldspar fuses and acts as a matrix and binds silica and kaolin.

Which of the following accounts for why endodontically treated posterior teeth are more susceptible to fracture than untreated ones? Loss of pulpal vitality Plastic deformation of the dentin Damaged coronal structure Loss of moisture

c-Endodontically treated teeth have a decreased amount of coronal dentin due to the access preparation necessary for root canal treatment. Loss of the coronal dentin alters the internal architecture of the tooth, changing the dispersement of forces in the tooth. The change to the internal architecture puts the tooth at higher risk for fracture.

Which of the following statements is correct? A. Retentive pins should be used in Class IV restorations. B. Rebonding decreases both marginal integrity and color stability. C. Finishing diamond burs impart less surface damage than carbide burs. D. Metamerism aids the dentist when selecting a shade.

c-Finishing diamond burs impart less surface damage as compared to the carbide burs. Retentive pins in Class IV restorations are contraindicated because they can show through the thin enamel and give an unaesthetic appearance to the restoration. Rebonding is completed at the end of restorative procedures to seal the surface and does not decrease marginal integrity and color stability. Metamerism is a complication in color perception as various light sources produce different perceptions of color. It will create problems in shade selection.

Which of the following describes the best management strategy for an instrument that is separated in the apical 1/3 after cleaning and shaping have occurred? Attempt to bypass the instrument Place calcium hydroxide Do not attempt to remove and proceed then to obturation

c-If an instrument separates at the filling/obturation stage, the instrument does NOT require removal because the etiology of the problem has already been resolved through cleaning and shaping of the canals. The prognosis for the tooth will be considered more favorably if the infected dentin has already been debrided from the canal. The prognosis for the tooth will also be considered more favorably the more apical the instrument separation occurred.

Which of the following is the instrument grasp most commonly used in operative dentistry? Palm grasp Palm-thumb grasp Pen grasp Two-handed grasp

c-Pen grasp is the most frequently used instrument grasp in operative dentistry procedures. The modified pen grasp is similar to the pen grasp except the operator uses the pad of the middle finger on the handle of the instrument. It provides more strength and stability in some procedures. The palm grasp is used for bulky instruments. It is commonly used for surgical forceps, rubber dam clamp forceps, straight chisels, and the air/water syringe. The assistant uses the palm‑thumb grasp to hold the oral evacuator. The operator may use this with instruments that require a more vertical movement.

The lifespan of a tungsten carbide bur can be extended by which of the following? be sterilized with heat sterilization. contact tooth structure before rotating. be moving quickly before contacting tooth structure. be moving slowly before contacting tooth structure.

c-So, By bringing the bur up to speed before contacting tooth structure, the lifespan of a tungsten carbide bur can be extended because it decreases the friction between the bur and the tooth structure, thereby decreasing damage to the bur and the pulp. Sufficient rotating speed makes the cutting edge of the bur more efficient even while using light pressure. Heat sterilization corrodes the carbon steel and anneals the cutting edge of the bur.

Each of the following is TRUE regarding self-curing sealants EXCEPT One. Which is the EXCEPTION? A. Benzoyl peroxide or aromatic tertiary amines are components. B. Late manipulation may disrupt polymerization. C. They cure by an endothermic reaction. D. They have two components.

c-The setting reaction of self cure, chemical cure or auto-curing sealants is exothermic in nature. A significant amount of heat energy is liberated. It is advised to use light-curing sealants for safety, good manipulation, and ease of handling. Auto-curing sealants are composed of two components including benzoyl peroxide or tertiary amines. Late manipulation can disrupt polymerization.

The temperature at which there is an abrupt increase in the thermal expansion co-efficient, indicating increased molecular mobility is called as: Co-efficient of Thermal Expansion Temperature of Diffusion Glass Transition Temperature Melting temperature

c-The temperature at which there is an abrupt increase in the thermal expansion coefficient, indicating increased molecular mobility, is called the glass transition temperature (Tg) and it is characteristic of the particular glassy structure. Occasionally, the term is shortened to glass temperature. Below Tg, the glassy structure loses its fluid characteristics and has significant resistance to shear deformation. Synthetic dental resins are examples of materials that often have glassy structures.

after implant placement, an edentulous patient should ? a. avoid wearing anything for 2 weeks b. immediately have healing abutments placed over the implants c. should wear an immediate denture to protect the implant sites

c-You first place the implants the suture back the flap heal for 3 months once osteo integration takes place second surgery open flap place healing collar wait 10-14days then replace with healing abutment and wait for gums to heal in order to take an impression after 6 months or so place the abutment bar and do over denture try in and everything fits well fabricate permanent denture in the mean time use a denture to protect implant sites and to allow healing by directing occlusal forces to other areas other than the implant ?

Vesicles or bullae of the mucous membrane or skin are seen in all the following except a. Herpes simplex b. Herpes zoster c. Agranulocytosis d. Pemphigus e. SLE

c-agranulocytosis

Lesion that characteristically occurs on the alveolar ridges of Infants is: a. Congenital lymphangioma b. Fordyce granules c. Bohn's nodules d. Mite sponge news e. Retrocuspid papilla

c-bohn's nodule

Causes of hypercementosis can be all except: a) Paget's disease b) periapical inflammation c) cleidocranial dysplasia d) mechanical stimulation

c-cleidocrfanial dyplasia(there is lack of cellular cementum following the deposition of acellular cementum)

The most common route by which infection may reach the pulp Is through: a. The blood stream b. Traumatic injuries c. Dental caries d. Erosion e. Periosteum

c-dental caries

Mottled enamel is produced by: a. Syphilis b. Febrile diseases c. Fluorsis d. Acids e. Tuberculosis

c-flurosis

which of the following is the most likely postoperative complication of bleaching a tooth that has not been adequately obturated? - fracture - discoloration - acute apical periodontitis - chronic ....... - external cervical root resorption

c-is correct because for nonvital tooth bleaching, we use hydrogen peroxide and sodium perborate.......which can go to periapical area.....if the obturation is not complete.....can cause inflammation right up there.....an d therefore can cause acute apical periodontitis....which is acute inflammation of peridontium .....periodontal ligament, bone and cementum. In later stages it can cause external root resorption This is probably caused by the oxidizing agent, particulary 30 to 35% hydrogen peroxide. The mechanism of bleaching-induced damage to the periodontium or cementum has not been fully elucidated. Presumably, the irritating chemical diffuses via unprotected dentinal tubules and cementum defects46,47 and causes necrosis of the cementum, inflammation of the periodontal ligament, and, finally, root resorption. The process may be enhanced if heat is applied48 or in the presence of bacteria.38,49 Previous traumatic injury and age may act as predisposing factors.

A static positional record can be used to adjust posterior articulator controls. In order to set the left condylar inclination and the left sideshift, which of the following are minimally required? a. Protrusive check bite in Incisal edge to edge position b. Protrusive check bite in extreme protrusive position c. Right lateral excursive record d. Left lateral excursive record

c-left molar shift which is the bennet movement occur in the non working side moving medially , so to get it , the patient will move the right side so the left ( non working side) will move medially... that's the only explanation i think....

The so-called "split papule," an erosive lesion Involving the commissure of the lips, Is actually: a. An aphthous ulcer b. A traumatic ulcer c. A mucous patch d. A fever blister e. Koplik's spot

c-mucous patch

All the following developmental cysts of the Jaws present as radlolucent lesions except a. Median palatal cyst b. Nasopalatine duct cyst c. Nasoalveolar d. Globulomaxillary cyst e. Epidermoid cyst

c-nasoalveolar

Double teeth is an anamoly most commonly found in- a) permanent incisors b) permanent canines c) primary incisors and canines d) primary molar

c-primary incisors and canines

Papillary cystadenoma Limphomatosum, which occurs almost exclusive In the parotid gland, Is commonly called: a. Cylindroma b. Pleomorphic adenoma c. Wartin's tumor d. Mikulicz disease e. jaffy's syndrome

c-wartin's tumor

If you have a grade III furcation, you can do all of the following except a. Section it and crown both as PFMs, b. Tunneling procedure c. GTR

c. GTR

A 6 year old patient has acute lymphatic leukemia. Her deciduous molar has large various lesion and furcation Lucency . How will you treat ?? A) pulpotomy B) pulpectomy C) extraction D) do nothing

c. extraction The dentist should treat existing dental problems in a manner which will decrease the risk of oral complications later. The dentist will extract teeth which: have acute or chronic infection, are mobile, or grossly carious. Pulpotomies and pulpectomies should not be performed on primary teeth, due to possible treatment failure and subsequent infection. Ideally, all extractions should be completed 10 days prior to the initiation of cancer treatment.

patient who uses nitroglycerine has a. rheumatic heart disease b. asthama c. high bp d. cardiac arrhythmia

c. high bp

histologically, aspirin burn is a. hyperkeratosis b. dysplasia c. mucosal necrosis

c. mucosal necrosis

hemangiomas of the jaw never occur in bone are malignant can appear cystic radigraphoically are metastatic lesions

can appear cystic radigraphoically

why composite is contraindicated on distal surface of canine?

can't be isolated properly-bcz of mesial concavity of premolar

which drug act as epileptic and neural pain inhibitor

carbamazepine and gabapentin

interaction of Erythromycin + antihistamine (Tefenadine) leads to ?

cardiac arrhythmia

recent research shows the relationship between periodontal disease and: hepatitis C alcoholism cardiovascular disease

cardiovascular disease

If the lining cement is left on the gingival cavosurface margin of a Class II amalgam restoration, the preparation will lack appropriate outline form the preparation will lack retention form. cement dissolution will lead to leakage the preparation will lack resistance form to bulk fracture

cement dissolution will lead to leakage

What Is the difference between surface resorption and Cervical resorption?

cervical resorption- is a relatively uncommon form of external root resorption Surface resorption: • It is associated with trauma to teeth in which the injury damages cementum and cementoblasts. • The traumatic event maybe avulsion, luxation, orthodontic forces or periodontal infections.

purpose of addition tin and indium to metal ceramic alloy 1-chemical bond 2-covalent bond

chemical bond

Caries are seen on which aspect of primary first molar?

children the distal surface of the first primary molar is the most caries prone, followed by the mesial surface of the second primary molar

Why we give hydralazine with chloral hydrate?

chloral hydrate may cause GI irritability (cramps) so hydralazine is periphral vasodilator acts by relaxation of smooth muscles in GIT and hence relief of cramps And to calm the patient

Which drug causes aplastic anemia? Cefuroxime Chloral hydrate Chloramphenicol Carbamazapine

chloramphenicol, carbamazepine, felbamate, phenytoin, quinine, and phenylbutazone.

allergic response with urticaria and wheeze Rx?

chlorpheniramine (Chlor-Trimeton), clemastine (Tavist) or diphenhydramine (Benadryl

What type of caries detection is the Dyfoti used for? Class I Class II, Class III •detection of incipient, frank and recurrent caries, demineralization

class 3 DIFOTI enables detection of fissure/occlusal caries, interproximal caries, and smooth-surface caries with superior sensitivity to radiography.

Nasal stuffiness associated with with class? 2 3 And its with bite?

class 3 with anterior cross bite

Transillumination is useful in the diagnosis of . Class 1 class 2 class5 class3

class3

bacteria producing collagenase?

clastrodium,bacillus,Klebsiella .porphyromonas

Treatment of midface deficiency?

cleft lip and palate as well as malar and midfacial retrusion with a Le Fort type III advancement.

For immature teeth, primary teeth and metal crowns which test- cold, ept, heat, percussion

cold test

what are the type of bond between teeth and: 1)Composite 2)Sealant 3)Amalgam

composite-micro-mechanically bond sealant-chemical amalggam-mechanical

what is the difference bet congenital epulis of new born and granular cell tumor though both lesions are identical histologically

congenital epulis does not exhibit peudoepithrliomatous hyperplasia and is S -100 negative the granular cell tumor frequently exhibit PEH and is s-100 positive

Pt says himself he can cope with dental treatment - Cognitive dissonance, conginitive restructuring, operant, sensitization

conginitive restructuring

Caries in children depend mainly on Time consistency Amount

consistency

If 2cm laceration on lip how do u stich- continous, in middle and work both ways, reconnect orbicularis oris first, reconnect vermillion border first

continous -reconnect vermilion border first

Coronal pulp or radicular pulp which pulp has less permeability

coronal pulp

A dentist seats a full gold crown in a patient's mand right second molar. As the patient closes and as the teeth come into initial contact, the patient's jaw deflects to the right. To regain stability, the dentist will adjust the crown. In which incline of which cuspid should the adjustment be made? Incline - cusp a- inner(lingual) - facial b- outer(facial) - facial c- inner(facial) - lingual d- outer(lingual) - lingual

correct answer and is "a". The explanation is: "Since the jaw deflects to the right, the maxillary supporting cusp must be hitting the inner aspect of the facial cusp". (first 2 words ex: inner (lingual) is the INCLINE while facial/lingual is the CUSP I think the lingual cusp of max molar contract with high buccal cusp in working side It's the BULL rule Buccal Upper, Lingual Lower)

which of the following is not penicillinase suseptible? a)Cloxacillin b)ampicillin c)Piperacillin d)Penicillin G

correct answer is a

Disease with most decreased incidence of caries: - down's syndrome - cystic fibrosis - Bell's palsy

cystic fibrosis Cystic fibrosis (CF) is the most common of the severe genetic disorders seen in Caucasians.

High recurrence rate of keratocysts is due to all factors except a) satellite cysts b) difficulty of surgical removal c) focal areas of active growth of the cyst wall d) cystic contents have a low soluble protein level

d- cyst contents have low soluble protein(production of bone resorbing factors)

Which of the following impressions can be electroplated easily : Impression paste Impression compound Hydrocolloids Elastomers

d-Impression pastes are messy to handle. Impression compound can exhibit change in dimension , due to release of internal stresses over period of time required to do plating(10hrs).Hydrocolloids are difficult to electroplate, besides they exhibit imbibitions and syneresis.

Which one will NOT be considered as a differential diagnosis of a periapical granuloma Periapical cyst Dentigerous cyst Condensing osteitis Dental papilla

d-Periapical Granuloma (Localized Osteitis).

For an accelerator to function properly it must : Decrease the solubility of both hemihydrate & dihydrate Decrease the solubility of hemihydrate & increase that of dihydrate Increase the solubility of hemihydrates & simultaneously that of dihydrate Increase solubility of hemihydrate without increasing solubility of dehydrate

d-The most widely accepted mechanism is the dissolution-precipitation theory based on dissolution of plaster and instant recrystallization of gypsum. Accelerator must increase solubility of hemihydrate without increasing solubility of dihydrate. Increasing the solubility of hemihydrate ensures acceleration, but this should not be accompanied by increased solubility of dihydrate.

The most likely diagnosis in a 23-year-old, mentally alert, male dwarf with disproportionate arm and log to body growth, prominent forehead, and retruded maxilla is: a. Cretinism b. Pituitary dwarfism c. Acrornegaty d. Achondroplasia e. Eagles syndrome

d-achondroplasia

. Most cases of pulpitis are caused by a) injudicious cavity preparation b) chemical irritation from sterilizing agent c) bacterial invasion from bloodstream d) bacterial invasion from carious lesion

d-bacterial invasion from carious lesion

Enamel hypoplasia is presented with all except a) Reduction in volume of matrix b) pits and grooves on the enamel surface c) enamel with fewer prisms d) incomplete calcification

d-incomplete calcification

Dry socket Is a form of a. Osteomyelitis b. Osteitis c Osleoma d. Periostitis e. Granuloma

d-periostitis

Atrophy of the pharyngeal and gastric mucosa, kollonychias (spoon nails). and predisposition to oral carcinoma In postmenopausal women are features of: a. Iron deficiency anemia b. Pernicious anemia c. Sturge-Weber-Dimitri syndrome d. Plummer-Vinson syndrome e. Trisomy 21

d-plummer-vinson syndrome

A researcher follows a group of individuals in a population over 10 years to determine who develops cancer and then evaluates the factors that affected the group. What type of study is this? A. Cross-sectional B. Case-control C. Randomized D. Prospective cohort E. Retrospective cohort

d-prospective cohort

Epstein pearls are a) same as enamel pearls b) imperfection in the tooth bud c) seen in vitamin D deficiency d)small keratin cyst of newborn infant

d-small keratin cyst of new born infant(bohn's nodule)

Implant treatment is a better option for smoker than perio surgery, because perio surgery in smoker doesn't work as well as non-smoker. A Both statements are true but unrelated B Both statements true and related C First statement true but reason is not D Neither the statement or the reason is true

d-smoking is a risk factor for implant failure, it is not considered an absolute contraindication Smokers respond less favourably than non-smokers to surgical periodontal therapy

. The end plate potential of the neuromuscular junction and the excitatory postsynaptic potential of the autonomic ganglion is generated by. a. Hydrolysis of acetylcholine b. Greater mobility of potassium ions in the postsynaptic membrane c. Reduction of the resting potential at the postsynaptic area by acetylcholine d. A nonspecific increase in the conductances of sodium, chloride andpotassium at the postsynaptic membrane

d. A nonspecific increase in the conductances of sodium, chloride andpotassium at the postsynaptic membrane

In pursuit of what the dentist believes is best for the patient, the dentist attempts to control patient behavior. This is known as a. Autonomy . b. Competence c. Maleficence d. Paternalism

d. Paternalism

Which of the following type of radiation poses the greatest danger to clinicians during radiography procedures? A. Alpha particles B. Direct primary -beam C. Gamma D. Secondary radiation and scatter

d. secondary radition and scatter

The letters "prn" appearing on a prescription indicate that a medication is to be administered a. at bedtime b. after meals c. by a registered nurse d. when needed

d. when needed

Chlorhexidine is an effective antiplaque agent due to its ability to non-selectively reduce the oral microbial flora. neutralize the bacterial end-products such as lactic acid. bind to the positively charged pellicle. damage the microbial cell membrane

damage the microbial cell membrane

Increased radiographic density is caused by decreased mA. decreased kVp. increased object-film distance. decreased target-object distance.

decreased target-object distance.

when PFM restoration the strongest?

dental porcelain is strongest under compression and weakest under tension

What appointment do you check for sibilant sounds? Wax-rim Denture Try in

denture try in (What are sibilant sounds? S, Z - anterior and posterior teeth should not touch, there should be no hissing or air loss )

What is calcific metamorphosis

deposition of hard tissue within the root canal space and yellow discoloration of the clinical crown.after truma to tooth.

The most important diagnostic element in assessing the periodontal status of a patient is the results of vitality testing. radiographic appearance. mobility of the teeth. depth of periodontal pockets.

depth of periodontal pockets.

During the extraction of an impacted tooth 3.8 the lingual nerve is damaged. All of the following can occur EXCEPT a numbness of the floor of the mouth on the left side. decreased salivary output from the left submandibular gland. decreased salivary output from the left sublingual gland loss of taste from the anterior 2/3 on the left side of the tongue. deviation of the tongue to the left on protrusion.

deviation of the tongue to the left on protrusion.

metabolic deficiency diseases which effect periodontal diseases?

diabetes,Hypophosphatasia

La toxicity ?

diazepam

nitrous oxide alone is not used as a general anesthetic agent because of the

difficulty in maintaing an adequate oxygen concentration.

van der Woude Syndrome

disorder characterized by the combination of lower lip pits, cleft lip with or without cleft palate,

Caries most on primary teeth - distal of first molar, distal of second molar

distal of second molar

HOw to measure attached gingiva?

distance from the gingival margin to the mucogingival junction. Total width of gingiva minus sulcus depth.

how many weeks must elapse between doses 1 and 3 in hep B vaccine?

dose 1-2 is 4 weeks dose 2-3 is 8 weeks dose 1-3 at least 16 weeks after first dose.

To reduce gag reflex Add cold water Bisecting instead of paralel Start from back

drink cold water for impression start from back

Most common complication during rct1) ledge formation2) perforation 3) vertical root fracture 4) instrument separation

during is ledge formation common complication is vertical root fracture.

What mean disclosing solution?

dye in solution or tablet form used to visualize and identify soft debris, pellicle, and bacterial plaque on the surfaces of the teeth ( such as Phloxine B)

Which of the following is the best reason a dentist may choose to use a resin-based composite to restore posterior teeth? Patient demonstration of trismus Unability to isolate tooth with rubber dam Frequent clinching or grinding Bad oral hygiene Esthetics

e

The stigmata of congenital syphilis does not include: a. Saber shins b. Interstitial keratitis c. Eighth nerve deafness d. Rhagades e. Cleft lip

e cleft lip

Endodontic sealant contraindicated and prohibited by ADA dental advisor panel: a)AH26 b)diaket c)roeckoseal d)endorez e)rc-213

e)rc-213

an increase in radiographic density is most likely caused by a decrease in

e-As SID increases, exposure rate decreases and radiographic density decreases.here viseversa

Which of the following is an opioid that has both agonistic and antagonistic activities? a. Codeine b. Methadone c. Naloxone d. Meperidine e. Pentazocine

e. Pentazocine

What happens with intercanine distance after mixed dentition? a. increased b. decreased c. stable, no change

early mixed dentition- increase, late mixed dentition- stable and after mixed dentition - decrease.

irrigation needle gauge?

endodontic irrigating needles (25, 27 and 30 gauge)

Cupping Appearance is a feature of? -Abrasion -Abfraction -Erosion -Attrision

erosion

20 yrs college student with pain bilateral in back bottom teeth with bad oral hygiene, pano show erupting 3rd molars bilateral and hyperkeratosis of operculum over one of them, what is cause of pain, periodontitis, erupting 3rd molars, hypersensitivity

erupting third molars - leads to pericoronitis

Of the following cold testing agents, which is the least effective in producing a response? ​a. bathing a tooth in ice water ​b. dicholorodifluoromethane (DDM) ​c. CO2 snow (dry ice) ​d. ethyl chloride

ethyl chloride

Histologically blue cell tumours such as lymphoma& Ewing sarcoma is differentiated by?

ewing sarcoma contains glycogen(shafers page 165)

Bleaching - external resorption, internal, alveolar bone formation, hypercementosis

external resorption

Compared to composites the Glass Ionomers have good compressive strength, tensile strength and hardness...T/F

false

Correct squance Endo , post - core ,and crown lengthening T/f

false- (endodontic treatment, crown lengthening (when indicated), post and core and prosthetic crown

Bilateral swelling of parotid gland not caused by anorexia T/f

false-Enlargement of the salivary glands can occur in patients with anorexia nervosa (AN). This enlargement appears to be related to both the nutritional deficiencies and the bizarre eating habits

Cuspal coverage - retention form. T/f?

false-increase Fracture Resistance of Weakened Teeth by Cuspal Coverage

. Isolated Expose root surface covered by soft tissuse - a-fenestration, b-dehiscence

fenetration

common benign epithelial tumor?

fibroma and adenoma

Collimation does everything except?? reduce pt exposure reduce operator exposure film fog reduce average energy of x-rays

film fog

Alveoplasty following closed extraction is usually performed with A. A bur B.fingers C.a rongeur D.a bone file

fingers

Ada code of ethics has a power flow, Everyone should follow the code of ethics even non-ada member

first statement true,second false

Where do you attach a non-rigid retainer from a FPD? Indications of non-rigid connector The indications for the use of nonrigid connector in fixed prosthodontics are5 : 1. The existence of Pier abutment, which promote a fulcrum-like situation that can cause the weakest of the terminal abutments to fail and may cause the intrusion of the pier abutment. 2. The existence of the malaligned abutment, where parallel preparation might result in devitalisation. Such situation can be solved by the use of Intracoronal attachments as connectors. 3. The presence of mobile teeth, which need to be splinted together with fixed prosthesis 4. Long span, fixed partial dentures which can be distort due to shrinkage and pull of porcelain on thin sections of framework and thus, affect the fitting of the prosthesis on the teeth. The nonrigid connectors are mainly used to relieve stress on the abutment and to accommodate malaligned fixed partial denture. Types of Non-Rigid Connectors The four types of nonrigid connectors are the: 1. Dovetail(key-keyway) or Tenon-Mortise type connectors. 2. Cross-pin and wing type connector. 3. Split type connector. 4. Loop type connector

fixed prostheses, the "male" portion of a nonrigid attachment usually is located on the mesial aspect of the posterior pontic, whereas the "female" portion is in the distal aspect of the natural pier abutment tooth. This prevents mesial drift from unseating the attachment.

Then a fixed partial denture is loaded vertically and tissueWard on the pontic , what type of force is being a place on the FPd? 1) compression 2. Tension 3. Flexure 4. Torsion

flexure is bending force that is applied perpendicularly to the long axis of the restoration which in this case is the length of the fixed partial denture

Blade No for internal bevel crevicular

for crevicular- Beak-shaped #12D blade is used. for internal bevel-no 15

in pediatric dentistry to provide normal class 1 a-anterior movement of retromolar pad b-the mandibular teeth move distal to maxillary teeth c-backward growth of the tuberosity d-mesial movement of maxillary teeth during eruption of premolars

for me A- Because as i know mandibuler teeth are move mesial not the the upper one.. leeway spaces at lower arch = 2.5 more than the at upper which is equal about 1.5 mm so that permit more mesial movement of lower teeth.

What is inter maxillary Fixation (IMF) means?

fractures involving maxillomandibular complex-tooth mounted devices like arch bars, dental and interdental wiring, metallic and nonmetallic splints are used to achieve intermaxillary fixation

margins for PFM crown

generally supra gingivally placed subgingival in case of crown lengthening in a short clinical crown

which muscle influences the length of the flange during normal movements of the tongue?

genioglossus muscle

what is the intra oral finding in leukemic patient

gingival enlargement(defic of platelets is increased gingival hemorrhage)

TFO on implant, symptoms except. gingivitis pain loosening of implant breakage of abutment screw

gingivitis

If patient starts wheezing? Don't give oxygen; last thing you would do; other options give beta 2 blocker inhaler, corticosteroid inhaler, make patient more comfortable

give beta 2 blocker inhaler. breathlessness improve within 5-15 minutes with a beta-agonist inhaler. Steroid inhalers are only used in combination with a long-acting beta-agonist inhaler.

Patient has allergy to codeine what should be given in severe and mild pain ?

give synthetic opoids like meperidine, tramadol, buprenorphine for severe pain Mild pain aceta Moderate ibuprofen

At the preparation stage for a 4.6 DO restoration, a brown, non-cavitated lesion is observed on the 4.7 mesial proximal surface. There is no evidence on a bitewing radiograph of a radiolucency extending into the dentin. What is the most appropriate management of the mesial surface of tooth 4.7? Prepare and restore with a mesio-occlusal slot composite resin. Prepare and restore with a mesio-occlusal slot resin modified glass ionomer. Prepare the lesion with a ball diamond and restore with a flowable composite resin. Perform no operative treatment and apply fluoride.

given ans -Perform no operative treatment and apply fluoride?

How to differentiate ground glass appearance from cotton wool appearance ?

ground glass appearance is- orange peal appearance cotton wool- finger print appearance

Enamel pearls most often on which tooth?

he most common location of enamel pearl is the furcation areas of the maxillary and mandibular third molar roots

What condition of the psnoramic x ray shows smile line frown?

head tipped up- max incisors roots are cut,frown line head tipped down -mand inc roots cut ,v shaped line

Hemisection technique is most utilised in which teeth and root amputation mostly involve which teeth?

hemisection technique -Mandibular molars to treat Class II or III furcation invasions, Root amputation - removal of one root in a multi-rooted tooth(Root resection for maxillary grade 2 or 3)

most common type of hemophilia?

hemophilia A

One week following extraction of teeth 1.8 and 4.8, an 18 year old male returns to the dental office complaining of persistent bleeding from the extraction sites. The medical history is unremarkable, except for episodes of bruising and joint swelling as a child. Subsequent blood tests show normal bleeding time and a factor VIII level of 14%. The most likely cause of the bleeding is vitamin K deficiency. cirrhosis of the liver. ibuprofen intake. a dry socket hemophilia A.

hemophilia A.

Extrusion of canine which is used except Envelope Semilunar.. Apically positioned

here are 3 techniques for uncovering a labially impacted maxillary canine: excisional uncovering apically positioned flap , and closed eruption techniqueshere are 3 techniques ans-A

subtraction radio graph?

highlighting the differences between two radiographs by covering one film of a part with a diapositive film of the same part.mainly used in implant placement.

A rubber dam in which there is leakage interproximally most likely has too much tension on the rubber dam holder. too broad a rubber dam arch form holes punched too far apart holes punched too close together.

holes punched too close together.

which antihypertensive agent act directly on arterial smooth muscle to cause vasodilation?

hydralazine -It is a direct-acting smooth muscle relaxant and acts as a vasodilator primarily in arteries and arterioles. By relaxing vascular smooth muscle, vasodilators act to decrease peripheral resistance, thereby lowering blood pressure and decreasing afterload.

epinephrine not used in hyperthyroid hypothyroid hypoglycemia

hyperthyroid

Betadine contraindicated with which patients ?

hyperthyroidism, lithium therapy, renal failure,pregnancy, nursing, neonates younger than 6 months.

In smoker's soft palate, theres red points, wut could it be? erythroplakia, initial stages of SCC, nicotinic stomatitis (hard palate)

i think - c-because of plugging of the minor salivary glands

Which of the following are NOT as an adjunct to treatment of aggressive periodontitis : A. Amoxicillin . B. Metronidazole. C. Clindamycin. D. Ciprofloxacin. E. Augmentin.

i think clindamycin https://www.google.com/url?sa=t&rct=j&q=&esrc=s&source=web&cd=1&cad=rja&uact=8&ved=0ahUKEwiv_Yqj8JXSAhXphVQKHeKnDO0QFggaMAA&url=https%3A%2F%2Fwww.ncbi.nlm.nih.gov%2Fpmc%2Farticles%2FPMC3467876%2F&usg=AFQjCNGIzd9xSLUySGgx4cfhoS6c48iW5g&sig2=fZZ6GuWe2_dX5MTaUaxMTA

what is the best cement that you use under Amalgam & why ? what is the best lining that you use under composite & why ?

i think zinc phosphate cement is the best for amaglmam coz of its high strength .. Zinc polycarboxylate cement and glassionomer cenent arw best for composite coz of chemical bonding with tooth structure and composite.Glass ionnmer is the best under composite to gain advantages of sandwich techniqe.

ideal crown to root ratio ?

ideal=1:2 optimum=1:1.5 minimum=1:1

Diff btw regeneration and repair in perio attachment

if healing events involve regeneration -process by which the architecture and the function are completely renewed periodontal repair -healing of a wound by tissue that does not fully restore the architecture of a part. Regeneration occurred in parts of the roots only if some original cementum remained on the root surface after root planing. Repair was observed if peripheral dentin was removed by root planing as this layer was not reestablished.

cement used to inlay and cement onlay?

if inlay and onlay >2mm thickness - dual cure resin cement if >3mm- autopolymerising resin cement

Alot of translucency of pontic gingival 3rd what is the reason Wrong shade selection Wrong metal thickness Modified pontic

if metal is thickness is increased it should be dark and if the thickness is thin then the porcelain is opaque right? throw some light. B but not due to inadequate tooth preparation but less than required share of metal beneath. Porcelain has higher translucency than metal so higher proportion of porcelain will lead to greater translucency.

What do you do if after SRP there are 2 probing sites of 6mm?

if the pockets are still 5-6mm after the initial therapy, then the patient has two choices: a). SRP under local anesthesia every 3 months b). Pocket reduction surgery to bring the deep pocket back to the normal 2-3mm level.

If tooth has closed apex, immerse tooth in which solution ?

immerse tooth in 2.4% sodium fluoride solution with pH of 5.5

The principal reason for a cavosurface bevel on a gold inlay preparation is to increase resistance and retention forms. decrease marginal percolation. remove undermined enamel. improve marginal adaptation

improve marginal adaptation

The principal reason for a cavosurface bevel on an inlay preparation is to increase resistance and retention forms remove undermined enamel improve marginal adaptation. decrease marginal percolation

improve marginal adaptation.

pfm failure is due to surface micro-cracks in ceramic because of heating stresses T or F ?

in DD it says " micro cracks are due to COOLING STRESSES REALATED TO POOR THERMAL CONDUCTIVITY OF PORCELAIN "

difference bet idiopathic thrombocytopenic purpura and thrombotic thrombocytopenic purpura

in ITP-incresed megakaryocytes, auto imnune destruction of RBCs by the spleen,there is a failure of the blood to clot, there is a failure of the blood to clot, treatment-oral steriods in TTP--Schistocytes and Increased LDHand ALS , formation of too many blood clots which lead to overused platelets.HUS (hemolytic uremic syndrome) in kids. treatment- blood replacement therapy

Ameloblastoma teeth Vital Non vital

in ameloblastoma the tooth is vital , while at multilocular keratocyst it's non-vital... and the both are similar in radiographic appearance

Advantage of partially covered crown over full covered?

in fully covered anatomic shape is given in partially covered 3/4 crown leaving facial aspect. conservation of tooth structure

Porosity mostly caused by - inadequate condensation, underfiring

inadequate condensation

Patient with diabetes which finding is not consistent ? increase collegenase in crevicular fluid, increase glucose in crevicular fluid, increase gram negative in crevicular fluid, decrease in thickness of basilar lamina of blood vessels in periodontium.

increase gram negative in crevicular fluid,

increasing frequency of period recalls, what effect on patient compliance: increase pt compliance no effect on pt compliance if the dentist decides frequency of recalls carefully it should never be changed

increase pt compliance

increase water powder ratio leads to decrease setting expansion and increased strength?t/f

increase water powder ratio lead to dec setting expansion, decrease strength and increase setting time overtrituration lead to inc stregth , dec setting time and dec expansion

what is kissing lesion ?

infectious mononucleosis-Contact mucositis is also known as Chronic Ulcerative Paradental Syndrome (CUPS). It is characterized by "kissing lesions" - red, inflamed, ulcerative lesions at the inner cheek lining and sometimes tongue edges Answer is median rhomboid glossitis. Kissing disease is infectious mononucleosis caused by EBV. Kissing lesion is median rhomboid glossitis.

radiopacity seen inferior border of mandible and ramus?

inferior border of mandible (submandibular sialoliths) near ramus (parotid sialoliths)

Needle of injection? Needle of aspirations?

injections -25 to 27 gauge needle 3/8 to 5/8 inches long for adults and children aspiration-23-25 gauge

Occlusal isthmus width in a class 2 inlay or amalgam?

inlay-occlusal 1/3rd isthmus wall amalgam-1/4th

How would you treat necrotizing ulcerative gingivitis with no obvious systemic symptoms?

instrumentation and debridment under LA, 2- H202 mouthmouth, 3- OHI, if systemic invol--Penicilline, if no systemic involv--no abtibiotic.

peutz jeghers syndr is associated with what type of polyps: benign/ inflammatory/ hamartomatous?

intestinal hamartomatous polyps

If changed from 8 mm to 16 mm cone how much exposure time do you need to increase by 2,4,6,8

inverse square law going from 8->16 = doubled distance 2r->1/2)2=1/4 radiation exiting so increase exposure by 4.(from 8mm to 16 mm cone means cone/ target is LONGER .this is the PID (target film distance),if pid increased there is LESS magnification .if pid is shorter there is MORE magnification .also density increase when KA, MA and exposure are increased. thatmeans the x-ray looks darker.

conscious sedation

is a combination of medicines to help you relax (a sedative) and to block pain (an anesthetic) during a medical or dental procedure example - A short-acting benzodiazepine (eg, midazolam), either alone or in combination with an opioid analgesic (eg, fentanyl, morphine)

coin test in radiograph?

is a test for checking unsafe illumination - shut down all lights and put on safe lights, open film packet and palce the base film in the place where the films are usually unwrapped. place coin on the film for approx 1.5 minutes. process the film. fogging of the film due to safelight wiil be obvious when compared to clear area protected by the coin.

combination clasp for rpd?

it consists of a wrought wire retentive clasp arm and a cast reciprocal arm. cast reciprocal arm is a circumferential arm or bar clasp arm. wrought wire is a circumferential arm . it is indicated when undercut is present on the mesiobuccal side of abutment tooth adjacent to a distal extension space.

Zygomatic process X-ray feature? U J

it is J shape - Jughandles,elephant's trunk' appearance Zygomaticomaxillary complex (ZMC) fractures, also known as a tripod, tetrapod, quadripod, malar or trimalar fractures, are seen in the setting of traumatic injury to the face. They comprise fractures of the: zygomatic arch inferior orbital rim, and anterior and posterior maxillary sinus walls lateral orbital rim

Which of the following symptoms is the most distinct characteristic of morphine poisoning:: A. Comatose sleep B. pin point pupils C.Depressed respiration D. Deep, rapid respiration E.Widely dilated non responsive pupils

it is b- its dilated but responsive pupils

which of the injections is most likely to have positive inpspiration a. IA 10-15% b.akisino c.infraorbital d.gowgate 2% e.mental nerve

it is inferior alveolar nerve has 10-15% positive aspiration , next to that is psa with 3.1% positive aspiration

Child with rubber dam which hole u choose A.small B large C hole size doesnt matter

it says the large punch hole is used for the clamp bearing tooth and for nost permanent molars.The medium size punch hole generally for premolars and primary molars.The second smallest hole of maxillary permanent incisors ,whereas the SMALLEST hole is adequate for primary incisors and lower permanent incisors. I think A makes sence

In mandibular complete denture construction, which area is most difficult to manage? a. Mylohyoid area b. Retromylohyoid area

it's B....this area is limited by palatoglossus and superior constrictor muscles. Impingement on these muscles by the denture may case sore throat..

Lingual nerve resection after joining with cords tympani resulting in Loss of taste and muscle sensation Loss of sensory reception Loss of taste and tactile sensation Only tactile sensation Only taste sensation

it's because lingual nerve carries general sensation to anterior 2/3 of tongue and chords tympani carries taste sensation to same region, so if we resect the nerve after they join together then both the general and taste sensations are lost, meaning tactile sensation and taste sensation

Kuppa?

it's syphilis in latin

Displacement of odontoblastic nuclei caused by: A) mechanical. B) thermal. C) chemical

its 2--> the heat (thermal) produced by dental instrumentation when we use air coolant instead of water coolant-->this increases the intrapulpal pressure causes hyperemia n inflammation which leads to d displacement of odontablastic nuclei n cells into adjacent dentinal tubules..

Incipient interproximal caries are typically found in which of the following locations? Incisal of the contact At the contact Facial to the contact Gingival to the contact Lingual to the contact

its D Incipient interproximal caries are typically found gingival to the contact area. Incipient interproximal caries lesions extend up to half of the thickness of enamel and do not extend beyond enamel. Incipient caries do not necessarily require restoration unless the patient has a high caries risk index. Prescription of fluoride toothpaste/rinses or sealants are typically sufficient treatment for patients with low caries risk.

Where do you attach a non rigid retainer from a fpd?

key distal of pontic. hole mesial of abutment

most common face bow in FPD?

kinematic face bow is usually used for fixed partial dentures

which is contra indicated with local anesthetic Digoxin or Digitalis

la is indicated with digitalis in ventricular arrhythmias, it is contraindicated in atrial fibrillation with digitalis.

what causes of ectopic eruption of the tooth

large tooth size, deviated angulation of path of eruption, discrepancy in size or position of maxilla and delayed calcification of molars.

Which radiograph will give you direct view of TMJ?

lateral oblique is ideal MRI is the diagnostic study of choice for evaluation of disk position and internal derangement of the joint. CT scan for evaluation of TMJ is indicated if bony involvement is suspected .

A pregnant patient in second trimester falls into syncope during extraction of upper molars, she should be kept in Trendelenburg position Head down towards her feet Right lateral position Left lateral position

left lateral position

Radiologic damage is less with: A. more oxygen B. decreased are of exposure C. less oxygen 7 Comments

less oxygen- radioresistance is inc when radiation is given in reduced o2 state . the greater cell damage occurs in presence of o2 due to formation of h2o2 and hydroperoxyl free radicals more o2 after radiation Coz in hyperbaric you're giving the patient to a hundred percent oxygen that hasn't been exposed to radiation. This is the base principal of the use of hyperbaric oxygen to treat radiologically exposed patients. Oxygen reduces hypoxia caused by hypovascularization caused by irradiation

. Composite indirect casting has advantage on direct composite - less polymerization, more polymers

less polymerization shrinkage in the case of indirect composite casting.Superior esthetics, higher abrasion resistance, dimensional stability, color stability, strong bond between two adhesive surfaces, biocompatibility with oral tissues

What is the level of CD4 and nutrophil of HIV pt?

less than 200

pipe smoking thought to be one of the predisposing factor for the development of

leukoplakaia

Xerostomia causes all of the following except: A.smooth surface caries B.candiosis C.leukoplakia D.salivary gland infection

leukoplakia

mand incisors crowding 3mm at 8 years: A.grind distal of primary mand canines B.extract primary mand canines C.Lingual arch and observe

lingual arch and observe

Black triangle?

loss of papilla in between teeth -in anteriors

Which m divides submandibular space in two spaces ? 1) Buccinator 2) Mylohyoid

ludwig's angina This infection involves the submandibular space. The submandibular space can be divided into two spaces: submaxillary and sublingual space. These two spaces are separated from each other by the mylohyoid muscle. These two spaces are connected posteriorly through a cleft known as the mylohoid cleft

Best way to visualize sialolith in whartons duct?

mandibular occlusal view -but Sialography, and especially the digital subtraction sialography (DSS) is a recognised diagnostic method in suspicion of the salivary gland calculi, sialoadenitis, fistulas, as well as stenoses , and salivary gland dilatations, being helpful in their precise localization.

A cyst situated posterior to nasopalatine duct cyst

median palatal cyst

which is the most commonly exposed pulp horn in primary and permanent dentition?

mesiobuccal

What drugs often prescribed by dentists may affect taste or smell?

metronidazole, benzocaine ampicillin tetracycline codeine sodium lauryl sulfate toothpaste

Achondroplasia ? Mid face deficiency overgrowth of cranial base

mid face def-have broad forehead Achondroplasia is a common cause of dwarfism. It occurs as a sporadic mutation in ... The skull is large, with a narrow foramen magnum, and relatively small skull base.

what happens with intercanine distance after mixed dentition a. increased b. decreased c. stable, no change

mixed dentition- stable , no change primary to permanent- increase in permanent - decrease

Showing Child t/t on other sibling - modeling, sensitization, positive reinforcement, operant

modeling

monoplane occlusion?

monoplane occlusion is nothing but neutrocentric concept- plat planes in all direction with no inclination at all in respect to the underlying denture foundation, balance was consider unnecessary and undesirable , as the resukting inclines will create instability of the dentures. Indications of the monoplane occlusion (neutrocentric concept) 1) Flat ridge(s) 2) Class II jaw relation 3) Class III jaw relations 4) Maxillofacial patients 5) Handicapped patients 6) Cross bite 7) Doubtful or Without any perfect centric relation records.

monoplane occlusion or neutrocentric occlusion ?

monoplane occlusion or neutrocentric occlusion - they are cusp less teeth on a flat plane with 1.5 -2mm overjet, no cusp to fossa relation no overbite

Theory affinity receptor a) antagonist bind and intrinsical activity b) agonist bind no intrinsical activity c) more receptor bind better response

more receptor bind better response

codeine does all except: A.mydriasis B.miosis C.Tachycardia D.restlessness

mydriasis

Nasopalatine and Nasolabial Cyst is same?

nasopalatine cyst- common lesion, well circumscribed,occurs bet apices of maxillary central oincisors nasolabial cyst- soft tissue swelling ,lateral to ala of the nose

The concept of health is considered as feeling of wellbeing , able to fulfill role expectations, free of pain and distress is seen in a)native americans b)african americans

native americans

Most common solid tumors seen in children

neuroblastoma, rhabdomyosarcoma, Wilms' tumor, and osteosarcoma.

most common cell in gingival crevicular fluid: macrophage neutrophil plasma cell

neutrophils

on palate a diffuse,greyish-white ,thickened, multinodular papule appearence with a small red "SPOT"in the center of each tiny nodule . seen in

nicotine stomatitis

NICOTINIcSTOMATITIS due to MELANOCYTES O MeLANIN CAUSEin SMOKEr O SMOKELESS ?

nicotine stomatitis-increased production of melanin by the normal number of melanocytes(melanin 1q) A benign lesion on the hard palate typically associated with pipe and cigar smoking.(smoke 2q)

probing in implants?

normal probing with plastic probe. light pressure is needed to avoid damage to the delicate fibers that surround the implant.force of 0.25N

Autistic patient how will u deal: normal , tsd , some drug

normal,Repeatitive movements like flapping hands n sensitive 2 light and sound

Cl - 3 bevel - occlusal bevel & gingival butt joint, both occlusal & gingival bevel, gingival bevel & occlusal 90 degree butt joint, no bevel

occlusal bevel & gingival butt joint,better estthetics, better seal for margins, inc overalll retention, more E surface to be bonded, easier shade selection

max radiation dosage

occupational- 50msv or 5rem per year public exposure- 1msv or 0.1 rem per year Per year 50 msv( 5 rem) Per month 4 msv Per week 1 msv

A modified hinge non-adjustable articulator is limited in its accuracy to reproduce dynamic jaw movements because it can only reproduce large centric relation-maximum intercuspation slides. eccentric movements by multiple lateroprotrusive registrations. one hinge axis movement.

one hinge axis movement.

Which of the following are non- eugenol containing periodontal dressing? A) Periocare B) Coe-pak C) baricaid

one non-eugenol-containing periodontal dressing, Coe-pak (n = 76) and 2 eugenol-containing periodontal dressings, Wondrpak (n = 64) and Nobetec (n = 86).

amelogenesis imperfecta affect a-enamel b- dentin c- pulp d-cementum e- all

only enamel (dentinogenesis imperfecta affect all except enamel)

After the cementation of a crown, chronic gingivitis would most likely be the result of a/an subgingival finish line. supragingival finish line. undercontoured crown. overcontoured crown.

overcontoured crown.

hypercementosis which is a common feature that involves jaw is

paget's disease

what is the sign for pemohigus

painful blisters that start in the mouth or skin areas. skin blisters near the surface of the skin that come and go. oozing, crusting, or peeling at the blister site. and epithelial separation.

. Modeling - a-peer person+positive reinforcement, b-peer person+giving information

peer person +positive reinforsement

why do we use pen V for pulpal involvement and pen vk for endo?! whats the difference ? and is pen VK the number one choice for dental antibiotic?

penicillin VK is the drug of choice for periradicular abscesses is effective against facultative and anaerobic microorganisms associated with endodontic infections. Penicillin VK remains the antibiotic of choice because of its effectiveness, low toxicity

The drug which causes withdrawal symptoms in pt taking oxycodine?

pentazocine, nalbuphine

how to differentiate between chronic perio and suppurative perio?

percussion(electrical) and sinus tract. chronic suppurative means phoenix abscess.

The greatest risk to dental health care personnel for acquiring a blood-borne pathogen is through blood spatter on intact skin. salivary contamination of clinic jackets. manufacturing defects in gloves. percutaneous injury.

percutaneous injury.

Periodontal disease is associated with what systemic diseases?

periodontal disease and systemic conditions such as cardiovascular disease, type 2 diabetes mellitus, adverse pregnancy outcomes, and osteoporosis.

Probe angulation while measuring pockets is_____and force applied is how much

periodontal probe tip that was 0.8 mm in diameter with a 10 gram force. parallel tp tooth surface

Instrument to measure GCF(ging crivicular fluid)?

periotron (collected-intracrevicular washing, absorbing papre, micropipetes, twisted threads)

the first evidence of the internal resorption appear on the crown is

pink hue

How many hours until plaque accumulation (after brushing or eating?)

plaque accumulation begins immediately after brushing for example, 7-14 days matures 14-21 days-show gingivitis ref perio dd 126

pt have bald tongue and spoon shaped finger nails and predisposition to development of oral cancer what disease does pt have?

plummer-vinson syndrome

osler 's disease

polycythemia vera

how does PFM and acrylic over metal will be seen in radiograph?

porcelain-radiopaque acrylic-radiolucent

For complete dentures, the most important factor affecting speech is the reproduction of the palatal rugae. position of the molar teeth. palatal thickness. position of the anterior teeth.

position of the anterior teeth.

Pregnant lady notice hypertension why?

preeclampsia is a condition that pregnant women develop. It is marked by high blood pressure in women who have previously not experienced high blood pressure before. Preeclamptic women will have a high level of protein in their urine and often also have swelling in the feet, legs, and hands

The permanent teeth most frequently ankylosed are the Canines Incisors Molars Premolars

premolars- Ankylosis of deciduous teeth ("submerged teeth") may rarely occur. The most commonly affected tooth is the mandibular (lower) second deciduous molar.so it has to be premolars

most porcelain liable to fracture: a.PFM b.zirconia c. pressed leuc

pressed leuc

Primary and secondary support areas for Maxillary complete denture?

primary- residual ridge ,secondary - palatal rugae

Aspiration is not required in which situation? 1- anuerysmal bone cyst 2- simple bone cyst 3- hemangioma 4- primordial cyst 5- gaint cell granuloma

primordial cyst because you can see its surrounding the tooth you have a good differential that its a primordial cyst compared to the other cysts.

intra pulpal anesthesia?

produce anesthesia after 30 sec-true

Pt with generalized pdl widening and bilateral mandible angle resorption. So which d/s

progressive systemic sclerosis(scleroderma)

Administration of succinylcholine to a patient deficient in serum cholinesterase would most likely result in acute asthmatic attack convulsions prolonged apnea hypertension

prolonged apnea

25 yo female breast feeding 12m old child and currently pregnant-which sedative would you give

promethazine is an antihistamine used for conscious sedation - mosby

Which of the following drugs is most useful in treating or preventing angina pectoris? a.Digitalis b.Quinidine c.Propranolol d.Procainamide e.Pentobarbital

propranolol, and also nitroglycerin and verapamil. Long-acting forms of nitroglycerin are used to prevent angina from happening. Short-acting nitroglycerin, administered either as a sublingual tablet or spray, can complement anti-anginal therapy as part of optimal medical therapy in patients with refractory and recurrent angina either with or without myocardial revascularization, and is most commonly used to provide rapid therapeutic relief of acute recurrent angina attacks.

What drug adds iodine to thyroxine decreasing its level?

propylthiouracil prevents the binding of iodine to hormone

Interproximal reduction of the mesial of the lower primary cuspids is indicated to encourage mesial movement of the deciduous molars. decrease inter-cuspid arch width. provide space for alignment of the permanent incisors when crowding is 3-5mm. provide space for alignment of the permanent incisors when crowding is 1-3mm.

provide space for alignment of the permanent incisors when crowding is 1-3mm.

Granulation tissue which extends coronally from the pulp of a carious tooth is known as a/an fibroma. epulis granulomatosum. pyogenic granuloma. pulp polyp.

pulp polyp.

filler in composite disadvantage?

quartz filler- difficult to polish , potentially abrassiveto opposing teeth or restoration amorphous silica- less hard, not crystalline glass filler with heavy metals- weakens in acidic juices or fluids.more susceptible to wear, attacked by apf gel.

Struger-Weber syndrome a. mandibular retro b. midface ecto c. maxillary prog

ral manifestations include unilateral haemangiomatous lesions in the maxillary or mandibular gingiva, lips, tongue, palatine region, etc. In gingiva, these lesion present as unilateral hyperplasia due to an increase vascular component, and demonstrate bleeding following minimal traum

Pt with IV biphosphonate, what do you do with non restorable incisor??

rct

Bisphosphanate since 6 months t/t- coronectomy & seal the roots, extn

rct first

Radiographic examination of a 9 year old patient reveals the pre-eruptive position of a mandibular second premolar is tipped 20 degrees from vertical. The primary second molar exhibits no root resorption. The most appropriate management is to surgically upright the premolar. remove the primary second molar and place a space maintainer. recall the patient for reassessment in 6 months.

recall the patient for reassessment in 6 months.

Which of the following represents in the classic sign or symptom of an anterior displaced disc with reduction a. pain b. closed lock c. reciprocal click d. temporal headache e. decreased range of motion

reciprocal click

anti rotational effect in implant?

reducing the problem of screw loosening.

What is the most important etiologic factor in getting caries? Saliva ph Refined sugar fluoride tx saliva flow

refined sugar

ghost teeth

regional odontodysplasia

Child 6 year have abnormal enamel dentin and pulp in A quadrant you diagnosis is a. hypoplasia B. regional odontplasia c. Detogensis imperficta d. Amelogensis imperficta

regional odontplasia

Doing composite restoration , you have 1 mm of remaining dentin thickness . What to do? What is RDT?l

remaining dentin thickness iF RDT is 0.5 or less subbase and liner are given..mosby page 62

28 years old patient ..tooth #1 in lingual crossbite, 32 facially inclined, how to correct: cross elastics removable hawley appliance with finger spring fixed orthodontics extended to tooth #1 rapid palatal expansion

removable hawley appliance with finger spring for children elastics -adults

Clouded maxillary sinus shown on one side on X-ray indicate ?

retention cyst

Most feared rare complication of ZMC #

reterobulbar hemorrhage leads to blindness

least likely caused after radiation mucositis ridge resorption trismus

ridge resorption

which two major forces are used for routine tooth extraction?

rotation and luxation

Treatment of LAP?

scaling and root planning, if systemic-Tetracycline seems affecting Aa better (250 mg 3 times daily for 2 weeks).or Metronidazole 400 mg and Amoxicillin 250 mg 4 times daily for 1 week in severe cases.

Tb of submaxillary and cervical luymph node known as scarlatina scrofula lupus vulgaris military tb

scrofula

What is green stick fracture?

seen In children In this periosteum remains attached

which is the most frequently used pin type? friction locked cemented self threading self-tapping

self threading

In mucous membrane pemphigoid, a positive Nikolsky sign is the result of a/an intraepithelial bulla formation. separation of the lamina propria and submucosal. intraepithelial separation. separation at the basement membrane.

separation at the basement membrane.

Floating teeth in air?

severe periodontal disease (most common cause 1) eosinophilic granuloma hyperparathyroidism metastases multiple myeloma ameloblastoma osteomyelitis lymphoma alveolar squamous cell carcinoma nerve-based or primary bone tumours (e.g. Ewing sarcoma) of the mandible or maxilla

How to check occlusion of gold restoration?

shim stock

gene defect , in which valine is replaced with glutamic acid results in

sickle cell anemia (life span is 20 days)

Negative effect of chronic use glucocorticoids?

sleep disturbance acne edema neuropathy hypertension osteoporosis cataract growth retardation cushing disease dec immunity redistrbution of fat hyperglycemia ulcers moon face

replacement resorption of tooth best way to prevent it?

soaked in 2.4% acidulated sodium fluoride solution.

blood spills in hospital clean by? alchohol Quaternary AC hypochlorite Idodine

sodium hypochlorite

Which cranial suture is the last to close ?

sphenooccipital

1. The undesirable side-effect most commonly associated with use of a buccal coil spring to regain space for a mandibular second premolar is: a) pain b) gingival irritation c) severe mobility of the tooth *d) tendency for the first premolar to rotate 2. Slow progress in molar uprighting in an adult patient is usually due to: a) overextended bands b) an overcontoured spring c) lack of anchorage control *d) the occlusion not being relieved 3. The approximate age range for calcification of the fibrous tissue at the mandibular symphysis in humans is: a) 20 th to 30 th weeks in utero *b) 6-18 months c) 1.5 to 2.5 years d) 2.5 to 5 years 4. When an uncontrolled tipping force is applied to the crown of a single-rooted tooth, the fulcrum is usually located: a) at the apex b) at the cervical line c) 5mm beyond the apex *d) 1/3 of the root length from the apex 5. Bone tissue grows by: *a) appositional growth b) interstitial growth c) osteoclastic activity d) proliferation of mesenchymal tissue 15. When a loop is placed in a wire segment between two brackets, the range of action: *a) increases b) decreases c) remains the same d) loops do not affect the range of action 16. In determining a patient's skeletal pattern, the most important factor is: a) diet b) habits *c) heredity d) dental occlusion 17. A corrected anterior crossbite is best retained by: a) overcorrection *b) normal incisor relation c) placing a Hawley retainer d) placing an acrylic palatal appliance with no facial arch wire 18. To be effective in moving the roots of teeth, an orthodontic appliance must be: a) very gentle in action b) augmented with extraoral force *c) capable of exerting a torque or moment on the teeth d) capable of exterting positive, intermittent forces 9. Loops and helices are used in arch wires primarily to: a) align teeth b) increase force of the wire *c) increase flexibility of the wires d) hold soft tissues away from orthodontic brackets 20. How does mandibular growth in boys ordinarily compare with that in girls? a) Is sustained over a longer period of time in girls *b) Is sustained over a longer period of time in boys c) Occurs at the same chronologic age in both sexes d) Occurs two years earlier in boys than in girls 21. The cranial vault increases rapidly in size the first few years postnatally and completes approximately 90% of its growth by 6 years of age. This growth is typical of which of the following types of tissues? *a) Neural b) Dental c) Genital d) Lymphoid

star marks are answers

If there is insufficient space between the maxillary tuberosity and the retromolar pad, it is usually best to -open the articulator slightly. -avoid covering the pad with the lower base. -not cover the tuberosity with the upper denture. -surgically reduce the occlusal aspect of the tuberosity. -surgically reduce the retromolar pad.

surgically reduce the occlusal aspect of the tuberosity.

Eagle syndrome

symptomatic elongation of the styloid process or calcified stylohyoid ligament 1-2. It is often bilateral. sudden, sharp nerve-like pain in the jaw bone and joint, back of the throat, and base of the tongue, triggered by swallowing, moving the jaw

glass bead sterilizer temperature and time

temperature (217 oC -232oC) for brief exposure times (e.g., 45 seconds) to inactivate microorganisms

If pt doesnt get steroid treatment in time for temporal vasculitis what will happen Hearing loss Vision loss Retro ocular headache

temporal vasculitis is temporal arteritis. it involves the inflammation of superficial temporal arteries. which affects ophthalmic.. vision loss is one of the most significant cause of morbidity in this disease

A 19-year old college student develops photosensitivity after she starts taking a new prescribed antibiotic last week. What is the responsible medication? A- Penicillin B- Amoxicillin C- Gentamicin D- Isoniazid E- Tetracycline

tetracycline becoz it may lead to rashes when it is exposed to sun which in turn will result in photosensitivity

Stained teeth with cystic fibrosis,why?

tetracycline causes staining during odontogenesis

how much is the temperature of dry ice used in thermal testing of pulp vitality? -20deg -80deg -4deg _+4degr

the use of co2snow as cold test -77.7de/-108deg one is able to penetrate full coverage restoration and elicit a response from the underlying tooth.

Hyperventilation A. Tachycardia B. Tachypnea

they both can't happen together because : tachypnea is as any rapid breathing, hyperventilation is increased rate of breathing at rest, hyperventilation causes the concentration of carbon dioxide in the blood stream to fall and produces a state known as hypocapnia.

Acyclovir MOA?

this is from wiki: Aciclovir is converted by viral thymidine kinase to aciclovir monophosphate, which is then converted by host cell kinases to aciclovir triphosphate (ACV-TP).ACV-TP, in turn, competitively inhibits and inactivates HSV-specified DNA polymerases preventing further viral DNA synthesis without affecting the normal cellular processes.

Cushion hammock ligament?

this ligament is base of the socket passing from one bony wall to the other

Adv of rectangular orthodontic wires?

tightly fit within the bracket, esthetic, stabilization, maximum friction

The primary purpose(s) of relining a distal extension base of a removable cast framework partial denture is/are to improve 1. fit of the framework. 2. occlusion. 3. function. 4. tissue adaptation. A. (1) (2) (3) B. (1) and (3) C. (2) and (4) D. (4) only E. All of the above.

tissue adaptation.

In the context of informed consent, choice means the ability to accept recommended beneficial treatment voluntarily. to refuse recommended beneficial treatment voluntarily. to refuse recommmended beneficial treatment with an understanding of forseeable consequences.

to refuse recommmended beneficial treatment with an understanding of forseeable consequences.

Over drying after acid etching results in?

tooth sensitivity caused by over drying or over etching.

hypercementosis associated with which hormonal abnormality

toxic thyroid goiter, acromegaly, pituitary giagantism

Best treatment for erythroplakia?

treatment involves biopsy of the lesion to identify extent of dysplasia. Complete excision of the lesion is sometimes advised depending on the histopathology found in the biopsy. Even in these cases, recurrence of the erythroplakia is common and, thus, long-term monitoring is needed.

. Immunoglobin in myeloma - m spike

true

Angle cl - 1 - mesiobuccal cusp of first molar of max in buccal groove of mand

true

Avulsed tooth should be treated with what to reduce root resorption? 2.4% Sodium fluoride for 20 minutes at 5.5 pH

true

Celebrex (from list of NSAIDS)(cox 2) doesn't prolong bleeding time T/F

true

Distance from xray- 6 fee

true

Erythroplakia and leukoplakia have no histological connotations. They are purely diagnosed clinically. t/f

true

If the patient complains of sore gums and aching muscles at the bottom of the face after wearing dentures for several hours, opposing teeth of the denture have insufficient space u reduce verical dimension . false or true

true

If we have myelinated and unmyelinated nerves of equal sizes, the myelinated nerve will get blocked first. T/F

true

Mature plaque - 4-7 days

true

Rotation - articular disc & condyle

true

Trismus and injection of IA tru parotid can cause bell's Palsy? T/F

true

True OR False The following analgesic drugs should be avoided in patients with renal disease- ASPIRIN ACETAMENOPHEN NSAID's MEPERIDINE MORPHINE

true

Unfilled resin has a high thermal expansion and lowe thermal conductivity...f or t

true

Which order will sensation disappear? 1. pain, 2.temp, 3.touch, 4.pressure

true

clinically gemination and fusion look similar T/F

true

gingival recession in anterior region : pedicle graft multiple gingival recession: free gingival graft deep gingival recision;subepithelial connective tissue graft T OR F ?

true

number of teeth which calcification has begun at 30 weeks of utero stage is : 20 primary teeth T/F

true

sagital groove is made in the upper front of tongue with a small cross sectional area while producing lingualveolar sounds T/F

true

Drug agonist of folic acid Sulfa Trimethoprin Methotrexate T f

true folic acid antagonist. a compound such as trimethoprim or methotrexate which acts as an antimetabolite of folic acid, interfering with DNA replication and cell division by inhibiting the enzyme dihydrofolate reductase.

5yrs missing second molar missing - Distal shoe true / false

true -prevents mesial dripping

Ultrasonic scalers are contraindicated with composite restoration ... T/F ?

true( When scaling porcelain and composites, the use of an ultrasonic scaler is contraindicated. Porcelain may fracture or lose marginal integrity. Composites have shown surface alterations, and amalgams have shown a loss of marginal integrity and surface damage.)

Perpendicular to object not to film = elongation Perpendicular to film not to object = foreshortening Yes or no

true(Foreshort. starts with F (so perpendicular to Film and since its a long word F O R E S H o R T E n i N G... then its a too much angulation...)

Separated instruments technique that operate under microscope are : Cancelier Technique and Mounce extractor T/F

true-

elective dentoalvelar surgery is NOT contraindicated in patients on oral bisphosphanate treatment -T/F?

true- Elective dentoalveolar surgery is not contraindicated in these patients.

Cingulum helps in preventing caries T/f

true- cingulum ( Latin : girdle or belt ) refers to an anatomical feature of the ... fissure sealants ) are a dental treatment intended to prevent tooth decay

All ceramic FPD should cover how much of abutment? 270 degrees t/f?

true- for rpd 180 deg for full crown-360 degrees

Hard setting calcium hydroxide does not cause necrosis of superficial layers of pulp. T/F?

true- it also been shown to initiate healing with a hard tissue barrier. ref- cohens pg 766

rendu osler-weber syndrome is also know as hereditary hemorrhagic telangiectasia.?

true- this syndrome is also known as hereditary hemorrhagic telangiectasia. it is a inheritated disease tha causes abnormal blood vessels formation. most common sign sign of this syndrome is epistaxis .beelding from oral cavity is common reason. treatment- antifybrinolytics.

Anterior guidance in complete denture should be avoided. True or false

true- to prevent dislodgment of the denture base.

5 A's of cancer prevention Ask, Advise, Assess, Assist, and arrange

true-Ask - Identify and document tobacco use status for every patient at every visit. (You may wish to develop your own vital signs sticker, based on the sample below). Advise - In a clear, strong, and personalized manner, urge every tobacco user to quit. Assess - Is the tobacco user willing to make a quit attempt at this time? Assist - For the patient willing to make a quit attempt, use counseling and pharmacotherapy to help him or her quit. (See Counseling Patients To Quit and pharmacotherapy information in this packet). Arrange - Schedule followup contact, in person or by telephone, preferably within the first week after the quit date.

T/F Nitrous O is contraindicated in Cystic Fibrosis ptn

true-Conditions relatively contraindicating the use of nitrous oxide include: Sinus infections (sinusitis) Nasal blocking conditions such as colds, influenza or allergies. Cystic fibrosis or Chronic obstructive pulmonary diseases (COPD) such as emphysema and bronchitis.

vasoconstrictor Contraindic with bisphosphonate?

true-Decrease blood supply

Bevel at the gingival margin required for a sliding , lapfit adaptation of a metal casting is 30 degrees T/F what does Sliding lapfit adaptation means?

true-Gingival Margin : For amalgam in Class II restoration 15-20o bevel on the enamel portion of the wall in order to remove unsupported enamel rods (because of gingival orientation of enamel rods). For casting, 30o will result in sliding, lap fit that improves adaptation of metal to this margin

SSRIs = currently the first line treatment for depression ... T/F ?

true-The serotonin-norepinephrine reuptake inhibitor, or SNRI, class (venlafaxine and duloxetine) is notable for a dual mechanism of action: increasing the levels of the neurotransmitters serotonin and norepinephrine by inhibiting their reabsorption into cells in the brain. These medications are considered as effective as SSRIs, so they are also considered a first-line treatment

In acrodynia periodontal problem can be seen.T/F

true-from text book mc donald and Avery pg 48

Pigeon chest?

turner syndrome, marfan syndrome, scoliosis,ehlers-danlos syndrome.

rootless teeth

type 1 dentin dysplasia(residual fragments of pulp tissue appear typically as horizontal lucencies)

radiographically thistle tube appearance seen in

type 2 coronal dysplasia

Manic phase of unipolar disorder which drug?

unipolar- TCA, SSRI bipolar-with lithium, or with an antipsychotic drug to control mania and depression or prevent another episode. Some other anticonvulsant drugs -- such as gapabentin (Neurontin), oxcarbazenpine (Trileptal),

Unstimulated salivary flow rate Stimulated salivary flow rate

unstimulated salivary flow= resting salivary flow no external stimulus 0.2 ml -0.3ml per minute stimulated saliveary flow=response to stimulus , usally taste, chewing, medication. 1.5ml -2ml unstimulated =submandibular stimulated parotid

There are 3 basic tests to identify a vertical crown "green-stick" crack or fracture. Which of the below is NOT an appropriate test? a) use "shift-shot" radiography to obtain an image of the crack b) trans-illuminate the tooth with fiber optic light c) use "tooth sleuth" or "frac-finder" to demonstrate symptoms. d) periodontal probing to establish a "drop-off" pocket

use "shift-shot" radiography to obtain an image of the crack

Anesthesia done into the vein result in ? Anesthesia done into arteries result in?

vein-increased heart rate, decreased blood pressure. artery- pain in periphery

best way to fix diastema direct composite restoration veneer

veneer

. Vertical height of face divided into - 3rd, 5th

vertical -5, horizontal 3

complete denture imp distortion could be due its 2 mm short from what?

vestibule or mucobuccal fold or sulcus

if patient doesn't get steroid therapy in time for their temporal vasculitis what will happen?

vision loss

What is primary source of radiation to the operator when taking xrays: radiation left in the air, were scatter from the patient, scatter from the walls leakage from the xray head

were scatter from the patient,

oral cancer more common in: hispanic females asian females black males white males

white males

A 10 year old has diffuse bilateral white areas on the buccal mucosa, dorsum of the tongue and the floor of the mouth. The mother tells the dentist that she has noticed this appearance for at least 3 years, but she was not concerned since she has had a similar condition all of her life. The most likely diagnosis is allergic reaction to cinnamon. frictional keratosis. leukoplakia. white sponge nevus.

white sponge nevus.

Most radiation from nature is inhaling radon yes or no

yes


Conjuntos de estudio relacionados

Assignment: Exercise 4.1 (Practice)

View Set

Intro to Psychology Chapters 1-5

View Set

Exam 3 Chapter 17 and 18 Questions

View Set

Which of the following statements best describes successful therapeutic conversations with patients and families? Correct! Health care providers should encourage and allow families to ask questions. For better outcomes, healthcare providers should o

View Set

Pharmacology nurse achieve notes.

View Set